You are on page 1of 167

MedCosmos Surgery: MCQ : General Surgery http://medcosmossurgery.blogspot.

com/search/label/MCQ : General Surgery

More Next Blog» Create Blog Sign In

MedCosmos Surgery
Surgery Lecture Notes, Books, MCQ and Good Articles

MedCosmos Medical Blogs

.All MedCosmos Series


F R I D A Y, S E P T E M B E R 5 , 2 0 0 8

General Surgery MCQ


1. Skeletal muscle breakdown produces predominantly liberation of
which two amino acids?
A. Lysine.
B. Tyrosine.
C. Alanine.
D. Glutamine.
E. Arginine.
Answer: CD Labels

History Taking (6)


DISCUSSION: Alanine is released from skeletal muscle and extracted by
Important Eponyms (1)
the liver, where it is converted to new glucose. Glutamine is also
released from muscle and participates in renal acid-base homeostasis MCQ : Breast (1)
and serves as fuel for rapidly growing cells such as enterocytes, MCQ : Cardiac Surgery (1)
stimulated macrophages, and fibroblasts. Together, these two amino
MCQ : Endocrine (1)
acids account for approximately two thirds of the nitrogen released
from skeletal muscle. MCQ : General Surgery (1)
MCQ : Hernia-Acute Abdomen (1)
MCQ : Large Intenstine (1)
2. In “catabolic” surgical patients, which of the following changes in
MCQ : Liver-Pancreas (1)
body composition do not occur?
A. Lean body mass increases. MCQ : NeuroSurgery (1)
B. Total body water increases. MCQ : Oncology (1)
C. Adipose tissue decreases.
MCQ : Pediatric Surgery (1)
D. Body weight decreases.
Answer: A MCQ : Small Intenstine (1)
MCQ : Spleen (1)
DISCUSSION: Lean body mass represents the body compartment that MCQ : Stomach-Esophagus (1)
contains protein. Because critical illness stimulates proteolysis and
MCQ : Thoracic Surgery (1)
increased excretion of body nitrogen, this compartment is consistently
reduced, not increased. The change in body composition is associated MCQ : Thyroid Gland (1)
with a loss of body weight, an increase in total body water, and a MCQ : Transplantation (1)
decrease in body fat.
MCQ : Trauma and Burns (1)
MCQ : Urology (1)
3. The hormonal alterations that follow operation and injury favor MCQ : Vascular Surgery (1)
accelerated gluconeogenesis. This new glucose is consumed by which of Surgery Books (7)
the following tissues?

1 of 167 11-Aug-15 10:57 PM


MedCosmos Surgery: MCQ : General Surgery http://medcosmossurgery.blogspot.com/search/label/MCQ : General Surgery

A. Central nervous system. Surgery Lectures (1)


B. Skeletal muscle. Surgery Must Read (1)
C. Bone.
Surgery Pamphlets (2)
D. Kidney.
E. Tissue in the healing wound.
Blog Archive
Answer: ADE
▼ 2008 (48)
DISCUSSION: Glucose is produced in increased amounts to satisfy the ▼ September (48)
fuel requirements of the healing wound. In addition, nerve tissue and Paradoxical Aciduria
the renal medulla also utilize this substrate. Skeletal muscle primarily Thoracic Surgery MCQ
utilizes fatty acids, and bone utilizes mineral substrate.
Cardiac Surgery MCQ
NeuroSurgery MCQ
4. Cytokines are endogenous signals that stimulate: Urology MCQ
A. Local cell proliferation within the wound. Pediatric Surgery MCQ
B. The central nervous system to initiate fever.
Vascular Surgery MCQ
C. The production of “acute-phase proteins.”
D. Hypoferremia. Liver and Pancreas MCQ
E. Septic shock. Large Intenstine MCQ
Answer:ABCD Stomach and Esophagus MCQ
Hernia and Acute Abdomen
DISCUSSION: Although cytokines exert primarily autocrine and paracrine
MCQ
effects, they may also cause systemic effects.
Trauma and Burns MCQ
General Surgery MCQ
5. The characteristic changes that follow a major operation or moderate Thyroid Gland MCQ
to severe injury do not include the following:
Small Intenstine MCQ
A. Hypermetabolism.
B. Fever. Oncology MCQ
C. Tachypnea. Transplantation MCQ
D. Hyperphagia. Enocrine Surgery MCQ
E. Negative nitrogen balance.
Breast MCQ
Answer: D
Spleen MCQ
DISCUSSION: The characteristic metabolic response to injury includes Technical Basis Of Radiation
hypermetabolism, fever, accelerated gluconeogenesis, and increased Therapy
proteolysis (creating a negative nitrogen balance). Food intake is Metastasis of Prostate Cancer
generally impossible because of abdominal injury or ileus. With time,
Schwartz Manual Surgery 8th
food intake increases, but the patient generally experiences anorexia,
Edition
not hyperphagia.
Bailey & Love's Short Practice of
Surgery

6. Shock can best be defined as: Sabiston Textbook of Surgery


A. Hypotension. A History of Plastic Surgery
B. Hypoperfusion of tissues.
Endoscopic Surgery of Potential
C. Hypoxemia.
anatomical spaces
D. All of the above.
1000 Eponyms in Surgery
Answer: B
Sentinel Lymph Node
DISCUSSION: Shock, no matter what the cause, is a syndrome associated History & Examination in
with tissue hypoperfusion. Tissue hypoperfusion leads to tissue hypoxia, Rheumatoid Arthritis
which may or may not be due to hypoxemia. Hypotension is a late sign

2 of 167 11-Aug-15 10:57 PM


MedCosmos Surgery: MCQ : General Surgery http://medcosmossurgery.blogspot.com/search/label/MCQ : General Surgery

of shock and, therefore, is not a good clinical indicator of the presence Varicose veins examination
of tissue hypoperfusion. Examination for a diabetic foot
Thyroid Examination

7. Which of the following statements about continuous cardiac output Lump History & Examination
monitoring are true? Named Hernia
A. Continuous cardiac output monitoring may unmask events not Deep Vein Thrombosis
detected by intermittent cardiac output measurements.
Post Operative Fever
B. Continuous cardiac output monitoring by the thermodilution method
requires continuous infusion of fluid injectate at a constant rate and Staging of breast cancer
temperature. PERIPHERAL VASCULAR DISEASE
C. The major advantage of the Fick method over the thermodilution History
method of calculating cardiac output is that it is noninvasive, requiring Q-A Scrotal Swelling
only the determination of oxygen consumption by respiratory gas
Management of Differentiated
analysis.
Thyroid Carcinoma
D. The technique of thoracic electrical bioimpedance utilizes sensors to
Top 100 Secrets in Surgery
determine stroke volume by detecting changes in resistance to a small,
applied alternating current. Burns
Answer: AD Sister Mary Joseph Nodule
DD of rectal bleeding
DISCUSSION: Various techniques are available to measure cardiac output
continuously. The advantages of continuous cardiac output monitoring, DD of scrotal swelling
as compared with intermittent methods, are (1) previously undetected DD of breast lumps
events may be unmasked; (2) more prompt recognition of adverse Lots of Surgery Pamphlets
events may be achieved; and (3) earlier therapeutic intervention may
be possible. Continuous cardiac output monitoring using the
thermodilution method appears to be as accurate as the “standard”
intermittent bolus method, but it does not require fluid injectates. In
this method, a modified pulmonary artery catheter incorporating a
thermal filament heats blood in the right ventricle at pulsed intervals,
and a distal thermistor detects the temperature change, which can be
related mathematically to cardiac output. The Fick method combines
respiratory gas analysis with oximetery to determine oxygen
consumption (V(overdot)O 2) and to estimate mixed venous and arterial
oxygen content differences, respectively. Cardiac output (CO) is then
determined from the formula: CO = V(overdot)O 2/ {C(a-v)O 2 × 10} @
V(overdot)O 2/ {SaO 2 - SvO 2) × (Hb) × (1.39) × 10}. Thoracic electrical
bioimpedance is a technique by which the resistance to a small-
amplitude alternating current (i.e., the impedance) is measured using
various electrodes. The impedance change induced by each cardiac
ejection is a function of the stroke volume, which then can be used to
calculate the cardiac output.

8. Which of the following statements regarding cytokines is incorrect?


A. Cytokines act directly on target cells and may potentiate the actions
of one another.
B. Interleukin 1 (IL-1) is a major proinflammatory mediator with
multiple effects, including regulation of skeletal muscle proteolysis in
patients with sepsis or significant injury.
C. Platelet-activating factor (PAF) is a major cytokine that results in
platelet aggregation, bronchoconstriction, and increased vascular

3 of 167 11-Aug-15 10:57 PM


MedCosmos Surgery: MCQ : General Surgery http://medcosmossurgery.blogspot.com/search/label/MCQ : General Surgery

permeability.
D. Tumor necrosis factor alpha (TNF-a), despite its short plasma
half-life, appears to be a principal mediator in the evolution of sepsis
and the multiple organ dysfunction syndrome because of its multiple
actions and the secondary cascades that it stimulates.
Answer: C

DISCUSSION: Cytokines are soluble peptide molecules that are


synthesized and secreted by a number of cell types in response to
injury, inflammation, and infection. Cytokines, which include the
interleukins, tumor necrosis factor, colony-stimulating factors, and the
interferons, comprise only one category of inflammatory mediators
involved in the host response. Endotoxin, complement fragments,
eicosanoids, kinins, nitric oxide, oxidants, and PAF are noncytokine
mediators that also have important roles in the systemic inflammatory
response. IL-1 and TNF-a, like other cytokines, have multiple effects on
target cells and potentiate the actions of other mediators to produce an
amplified inflammatory response. TNF-a is thought to play a central role
in the stress response, particularly in response to endotoxemia.

9. True statements concerning hypoadrenal shock include which of the


following?
A. Adrenocortical insufficiency may manifest itself as severe shock
refractory to volume and pressor therapy.
B. The presence of hyperglycemia and hypotension may suggest the
diagnosis of shock due to adrenocortical insufficiency.
C. Hydrocortisone does not interfere with the serum cortisol assay and
should be given to hemodynamically unstable patients suspected of
having hypoadrenal shock.
D. The rapid adrenocorticotropic hormone (ACTH) stimulation test
should be performed to help establish the diagnosis of acute
adrenocortical insufficiency.
Answer: AD

DISCUSSION: Shock due to acute adrenocortical insufficiency is relatively


uncommon but must be considered when shock refractory to volume
replacement and pressor therapy is present. Hypoglycemia may be
present. Patients with high metabolic stress may exhibit adrenal
insufficiency only under conditions of severe stress; thus, a history of
adrenal insufficiency or steroid dependency need not be elicited. When
adrenocortical insufficiency is suspected, the rapid ACTH (cosyntropin)
stimulation test should be performed. Serum cortisol levels should be
drawn before intravenous administration of 250 mg. of cosyntropin, and
30 and 60 minutes afterward. A peak cortisol level of less than 20
mg./100 ml. suggests abnormal adrenal function. In a hemodynamically
unstable patient therapy should be instituted before the test results
become available. Dexamethasone does not interfere with the cortisol
assay, and it is the corticosteroid of choice while the ACTH stimulation
test is being performed.

4 of 167 11-Aug-15 10:57 PM


MedCosmos Surgery: MCQ : General Surgery http://medcosmossurgery.blogspot.com/search/label/MCQ : General Surgery

10. All of the following are true about neurogenic shock except:
A. There is a decrease in systemic vascular resistance and an increase in
venous capacitance.
B. Tachycardia or bradycardia may be observed, along with hypotension.
C. The use of an alpha agonist such as phenylephrine is the mainstay of
treatment.
D. Severe head injury, spinal cord injury, and high spinal anesthesia may
all cause neurogenic shock.
Answer: C

DISCUSSION: Neurogenic shock occurs when severe head injury, spinal


cord injury, or pharmacologic sympathetic blockade leads to
sympathetic denervation and loss of vasomotor tone. Both arteriolar and
venous vessels dilate, causing reduced systemic vascular resistance and
a great increase in venous capacitance. The patient's extremities appear
warm and dry, in contrast to those of a patient in cardiogenic or
hypovolemic shock. Tachycardia is frequently observed, though the
classic description of neurogenic shock includes bradycardia and
hypotension. Volume administration to fill the expanded intravascular
compartment is the mainstay of treatment. The use of alpha-adrenergic
agonist is infrequently necessary to treat neurogenic shock.

11. True statements regarding eicosanoids include which of the


following?
A. Prostaglandins and thromboxanes are synthesized via the cyclo-
oxygenase pathway.
B. The vasoconstricting, platelet-aggregating, and bronchoconstricting
effects of thromboxane A 2 are balanced by the actions of prostacyclin,
which produces the opposite effects.
C. Leukotriene synthesis is inhibited by the action of nonsteroidal
anti-inflammatory drugs (NSAIDs).
D. The principal prostaglandins have a short circulation half-life and
exert most of their effects locally.
Answer: ABD

DISCUSSION: The eicosanoids are a group of compounds arising from the


metabolism of arachidonic acid. The prostaglandins and thromboxanes
are synthesized via the cyclo-oxygenase pathway; thus, their synthesis is
blocked by NSAIDs. Leukotrienes, on the other hand, are synthesized via
the lipoxygenase pathway. Prostacyclin, produced largely by vascular
endothelium, inhibits platelet aggregation and causes vasodilatation as
well as bronchodilatation. Its effects are balanced by those of
thromboxane A 2, which is produced by platelets and also local actions,
including platelet aggregation, vasoconstriction, and
bronchoconstriction. The leukotrienes also have pulmonary and
hemodynamic effects and may be involved in the physiologic responses
associated with anaphylactic and septic shock.

12. Which of the following statements about delivery-dependent oxygen


consumption are true?

5 of 167 11-Aug-15 10:57 PM


MedCosmos Surgery: MCQ : General Surgery http://medcosmossurgery.blogspot.com/search/label/MCQ : General Surgery

A. Below the critical oxygen delivery (D(overdot)O 2crit), one would


expect to see a decrease in the lactate-pyruvate ratio.
B. D(overdot)O 2crit may be increased in patients with sepsis.
C. A desirable goal in the treatment of shock is to achieve delivery-
independent oxygen consumption.
D. The oxygen extraction ratio remains constant as long as oxygen
delivery remains above D(overdot)O 2crit.
Answer: BC

DISCUSSION: Oxygen consumption is said to be delivery dependent below


a critical point, D(overdot)O 2crit, at which anaerobic metabolism
supervenes. Above this point, oxygen consumption is relatively
independent of oxygen delivery because the body's cells can compensate
for falls in oxygen delivery by extracting more oxygen. In the delivery-
dependent region, if cellular hypoxia is present, the lactate-pyruvate
ratio rises, owing to the switch to anaerobic metabolism. Generally, it is
desirable to achieve delivery-independent oxygen consumption, to avoid
ongoing tissue hypoxia. There is considerable debate, however, about
the nature of the oxygen consumption–oxygen delivery relationship in
cases of established sepsis or multiple organ dysfunction syndrome. In
such cases, D(overdot)O 2crit may be increased, although the
therapeutic benefit of trying to achieve “supranormal” oxygen delivery
has not been firmly established.

13. All of the following may be useful in the treatment of cardiogenic


shock except:
A. Dobutamine.
B. Sodium nitroprusside.
C. Pneumatic antishock garment.
D. Intra-aortic balloon pump.
Answer: C

DISCUSSION: Cardiogenic shock occurs when the heart fails to generate


adequate cardiac output to maintain tissue perfusion. Intrinsic causes
such as myocardial dysfunction secondary to coronary artery disease, or
extrinsic causes such as pulmonary embolism, tension pneumothorax,
and pericardial tamponade, may produce cardiogenic shock. Principles
of treatment of cardiogenic shock are aimed at optimizing preload,
cardiac contractility, and afterload. Preload is usually adequate or high
in cardiogenic shock. Dobutamine is a useful inotropic agent,
particularly when filling pressures are high, because of its mild
vasodilatory effect, as well as its effect to enhance cardiac
contractility. Afterload-reducing agents, such as sodium nitroprusside,
may be beneficial in cardiogenic shock in the setting of elevated filling
pressures, low cardiac output, and elevated systemic vascular
resistance. Cardiac output may improve with use of afterload-reducing
agents by decreasing myocardial wall tension and optimizing the
myocardial oxygen supply-demand ratio. The intra-aortic balloon pump
(IABP), by providing diastolic augmentation, reducing left ventricular
afterload, and reducing myocardial oxygen consumption, is sometimes
useful in the treatment of cardiogenic shock. The IABP is especially

6 of 167 11-Aug-15 10:57 PM


MedCosmos Surgery: MCQ : General Surgery http://medcosmossurgery.blogspot.com/search/label/MCQ : General Surgery

useful in low–cardiac output postcardiotomy patients, in patients


awaiting revascularization, and in patients with acute myocardial
infarction complicated by mitral insufficiency or ventricular septal
defect. The pneumatic antishock garment (PASG), which causes an
increase in systemic vascular resistance, is contraindicated in
cardiogenic shock.

14. Which of the following statements concerning monitoring techniques


in the intensive care unit are true?
A. Pulmonary artery and pulmonary capillary wedge pressure readings
should be made at end inspiration, to minimize ventilatory artifacts.
B. Continuous SvO 2 monitoring based on the technique of reflectance
spectrophotometry has been shown to be accurate and reliable.
C. Direct measurement of gastric intramucosal pH can be provided by
gastrointestinal tonometry.
D. Hyperlactatemia may be seen in a number of clinical conditions not
associated with tissue hypoxia, including liver disease and
hypermetabolic states.
Answer: BD

DISCUSSION: Many different monitoring techniques may be used to


assess the adequacy of therapy for shock. The pulmonary artery
catheter can provide important hemodynamic and oxygen transport data
that are very useful in directing therapy aimed at optimizing cardiac
function and oxygen delivery. Pulmonary artery and pulmonary capillary
wedge pressure readings should be made at end-expiration to minimize
ventilatory artifacts. Continuous SvO 2 monitoring, an accurate, reliable
method that combines pulmonary artery catheterization with the
technique of reflectance spectrophotometry, may provide early warning
signs of hemodynamic compromise or inadequate oxygen delivery.
Gastrointestinal tonometry provides information that allows one to infer
the adequacy of splanchnic tissue perfusion. In this technique,
intramucosal pH is calculated using the Henderson-Hasselbalch equation
and measurements of gut intraluminal PCO 2 and arterial bicarbonate
concentration. Serum lactate concentration may be monitored in shock
to detect metabolic acidosis associated with anaerobic metabolism;
however, mild to moderate hyperlactatemia may also be seen with liver
disease, toxin ingestion, and hypermetabolic states not associated with
shock.

15. An 18-year-old man shot once in the left chest has a blood pressure
of 80/50 mm. Hg, a heart rate of 130 beats per minute, and distended
neck veins. Immediate treatment might include:
A. Administration of one liter of Ringer's lactate solution.
B. Subxiphoid pericardiotomy.
C. Needle decompression of the left chest in the second intercostal
space.
D. Emergency thoracotomy to cross-clamp the aorta.
Answer: AC

7 of 167 11-Aug-15 10:57 PM


MedCosmos Surgery: MCQ : General Surgery http://medcosmossurgery.blogspot.com/search/label/MCQ : General Surgery

DISCUSSION: The finding of distended neck veins in conjunction with


hypotension should suggest tension pneumothorax or pericardial
tamponade. Absent ipsilateral breath sounds and a trachea deviated to
the contralateral side may provide additional evidence for a tension
pneumothorax, the immediate treatment of which is needle
decompression of the chest in the second or third intercostal space in
the midclavicular line. Pericardial tamponade may initially respond to
volume administration by enhancing preload. Pericardiocentesis may
need to be performed emergently if hemodynamic instability persists
after an initial fluid bolus when signs of compressive cardiogenic shock
are present. Subxiphoid pericardiotomy should be performed only in the
operating room by experienced persons who are trained to deal with
penetrating cardiac injuries. There is no role for aortic cross-clamping
in this scenario of cardiogenic shock.

16. Which of the following statements are true of the multiple organ
dysfunction syndrome (MODS)?
A. The “two-hit” model proposes that secondary MODS may be produced
when even a relatively minor second insult reactivates, in a more
amplified form, the systemic inflammatory response that was primed by
an initial insult to the host.
B. The systemic inflammatory response syndrome (SIRS), shock due to
sepsis or SIRS, and MODS may be regarded as a continuum of illness
severity.
C. Prolonged stimulation or activation of Kupffer cells in the liver is
thought to be a critical factor in the sustained, uncontrolled release of
inflammatory mediators.
D. The incidence of MODS in intensive care units has decreased owing to
increased awareness, prevention, and treatment of the syndrome.
Answer: ABC

DISCUSSION: MODS is part of a clinical continuum that begins with the


systemic inflammatory response syndrome, which is the host's stress
response to any major insult such as injury or infection. MODS may
develop as a result of the initial insult, but more commonly, it develops
following a second or subsequent insult to the host. The two-hit theory
holds that the systemic inflammatory response is amplified following the
second hit, such as nosocomial pneumonia, leading to exaggerated,
persistent release of inflammatory mediators that contribute to the
pathogenesis of MODS. The liver appears to be a pivotal organ in the
progression and outcome of MODS, partly because of the activation and
prolonged stimulation of the Kupffer cells, which comprise the majority
of the body's macrophage population. Macrophages are known to play a
critical role in the elaboration of numerous inflammatory mediators.
Despite advances in critical care and in the understanding of the
pathogenesis of MODS, the incidence of MODS continues to increase
without a significant improvement in outcome.

17. All of the following statements about hemorrhagic shock are true
except:

8 of 167 11-Aug-15 10:57 PM


MedCosmos Surgery: MCQ : General Surgery http://medcosmossurgery.blogspot.com/search/label/MCQ : General Surgery

A. Following hemorrhagic shock, there is an initial interstitial fluid


volume contraction.
B. Dopamine, or a similar inotropic agent, should be given immediately
for resuscitation from hemorrhagic shock, to increase cardiac output
and improve oxygen delivery to hypoperfused tissues.
C. The use of colloid solutions or hypertonic saline solutions is
contraindicated for treatment of hemorrhagic shock.
D. In hemorrhagic shock, a narrowed pulse pressure is commonly seen
before a fall in systolic blood pressure.
Answer: BC

DISCUSSION: Hemorrhagic shock is associated with a contraction of the


interstitial fluid compartment because of precapillary vasoconstriction
and reabsorption of interstitial fluid into the vascular compartment
along hydrostatic pressure gradients. Systolic hypotension may not be
evident in hemorrhagic shock until at least 30% or more of blood volume
is exsanguinated. A decrease in the pulse pressure (the difference
between systolic and diastolic pressures) may be observed with losses of
15% to 30% of blood volume. Treatment of hemorrhagic shock includes
intravascular fluid administration and definitive control of the source of
the hemorrhage. Inotropic agents should not be started before volume
resuscitation but may be added to improve oxygen delivery to hypoxic
tissues if volume administration alone does not produce resuscitative
goals. Colloid or hypertonic saline solutions are not contraindicated in
the treatment of hemorrhagic shock; however, definitive evidence that
such solutions are better than standard crystalloid solutions is lacking.

18. Which of the following statements about septic shock are true?
A. A circulating myocardial depressant factor may account for the
cardiac dysfunction sometimes seen with shock due to sepsis or SIRS.
B. A cardiac index (CI) of 6 liters per minute per square meter of body
surface, a pulmonary capillary wedge pressure of 15 mm. Hg, and a
systemic vascular resistance index (SVRI) of 800 dynes-sec/(cm 5-m 2) is
a hemodynamic profile consistent with septic shock.
C. An increase in SvO 2 in septic patients may be explained by the
finding of anatomic arteriovenous shunts.
D. Results of human trials employing antimediator therapy, such as
antiendotoxin antibodies, IL-1 receptor antagonist, and tumor necrosis
factor (TNF) antibodies, have confirmed animal studies that
demonstrate a significant improvement in survival with the use of such
agents.
Answer: AB

DISCUSSION: Shock due to sepsis or SIRS frequently manifests as a


hyperdynamic cardiovascular response, consisting of an elevated CI and
a decreased SVR or SVRI. Occasionally, myocardial depression may be
seen, characterized by increased ventricular volumes and decreased
ejection fractions. A circulating myocardial depressant factor, possibly
TNF, may be responsible for the cardiac dysfunction in such instances.
The cause of the increased SvO 2 frequently observed in septic patients
is unclear, but it may be secondary to bioenergetic failure, metabolic

9 of 167 11-Aug-15 10:57 PM


MedCosmos Surgery: MCQ : General Surgery http://medcosmossurgery.blogspot.com/search/label/MCQ : General Surgery

downregulation, or microcirculatory maldistribution leading to


physiologic shunting. True anatomic arteriovenous shunting has not been
demonstrated in humans in septic shock. Treatment of septic shock
consists of appropriate antibiotic use and supportive therapy.
Experimental antimediator therapies have not been encouraging thus far
in human clinical trials, despite the promising results from many animal
studies.

19. Which of the following statements are true of oxidants?


A. In addition to their pathophysiologic roles in inflammation, injury,
and infection, oxidants also have physiologic roles.
B. Oxidants may be generated from activated neutrophils and during
reperfusion following a period of ischemia.
C. The deleterious effects of oxidants include lipid peroxidation and cell
membrane damage, oxidative damage to DNA, and inhibition of
adenosine triphosphate (ATP) synthesis.
D. The mechanism of ischemia-reperfusion injury involved the catalytic
production of superoxide anion (O 2•) by the enzyme xanthine oxidase.
Answer: ABCD

DISCUSSION: Oxidants are reactive oxygen metabolites that have both


physiologic and pathophysiologic roles. As potent oxidizing agents,
oxidants are involved in cytochrome P 450–mediated oxidations, for
example. In pathophysiologic processes associated with inflammation,
injury, and infection, oxidants may be generated by activated
neutrophils and in ischemia-reperfusion injury. During ischemia, the
enzyme xanthine oxidase accumulates. When oxygen availability
increases during reperfusion, O 2• is formed in a reaction catalyzed by
xanthine oxidase. Further oxidant formation ensues, causing the
production of H 2O 2 and the extremely reactive hydroxyl ion (OH•).
Oxidants may cause direct cell damage by the mechanisms of lipid
peroxidation and cell membrane disruption, inhibition of ATP synthesis,
reduction of cellular nicotinamide adenine dinucleotide (NAD), and
oxidative damage to DNA and amino acids. In addition, oxidants may
have a chemotactic role, leading to leukocyte infiltration and
activation, causing further tissue damage by the release of cytotoxic
proteases.

20. Which of the following statements about the role of the gut in shock
and sepsis are true?
A. Selective decontamination of the digestive tract with the use of oral
antibiotics has been shown to reduce nosocomial pneumonias and to
improve mortality rates.
B. Enteral nutrition, as compared with parenteral nutrition, preserves
the villus architecture of the gut.
C. Gut dysfunction may be an effect of shock, but it may also contribute
to the development of MODS by the mechanism of bacterial
translocation.
D. As compared with parenteral nutrition, enteral nutrition is associated
with a reduction in septic morbidity.

10 of 167 11-Aug-15 10:57 PM


MedCosmos Surgery: MCQ : General Surgery http://medcosmossurgery.blogspot.com/search/label/MCQ : General Surgery

Answer: BCD

DISCUSSION: The gut has a vital role in the pathophysiology of shock.


The splanchnic circulation is very vulnerable to the circulatory
redistribution that occurs in shock, thus, gut ischemia may occur early
in the various shock syndromes. Gut injury, as a result of ischemia or
reperfusion injury, leads to disruption in the intestinal mucosal barrier
and increased gut permeability. Translocation of enteric flora or
bacterial toxins across the gut wall may then occur, resulting in
amplification of the systemic inflammatory response and the
development of multiple organ dysfunction. Gut dysfunction, therefore,
may perpetuate the inflammatory process. Various methods have been
tried to modulate the deleterious effects of gut dysfunction. Selective
decontamination of the digestive tract by oral antibiotics has been
shown to reduce the incidence of nosocomial pneumonias, but no
improvement in mortality has been demonstrated thus far with this
controversial technique. Early enteral nutrition probably has the biggest
impact on the preservation of gut architecture and function. When
compared to parenteral nutrition, enteral feeding is more cost effective
and is associated with a lower rate of septic morbidity.

21. Which of the following statements about head injury and


concomitant hyponatremia are true?
A. There are no primary alterations in cardiovascular signs.
B. Signs of increased intracranial pressure may be masked by the
hyponatremia.
C. Oliguric renal failure is an unlikely complication.
D. Rapid correction of the hyponatremia may prevent central pontine
injury.
E. This patient is best treated by restriction of water intake.
Answer: A

DISCUSSION: Acute symptomatic hyponatremia is characterized by


central nervous system signs of increased intracranial pressure. Changes
in blood pressure and pulse are secondary to increased intracranial
pressure. In the absence of hypovolemia, asymptomatic patients may be
treated by restriction of water intake; however, in such patients,
hyponatremia should be partially corrected by parenteral sodium
administration. Rapid correction, particularly to hypernatremia, may
lead to central pontine myelinolysis. Oliguric renal failure may rapidly
develop in severe hyponatremia.

22. Which of the following statements about total body water


composition are correct?
A. Females and obese persons have an increased percentage of body
water.
B. Increased muscle mass is associated with decreased total body water.
C. Newborn infants have the greatest proportion of total body water.
D. Total body water decreases steadily with age.
E. Any person's percentage of body water is subject to wide physiologic

11 of 167 11-Aug-15 10:57 PM


MedCosmos Surgery: MCQ : General Surgery http://medcosmossurgery.blogspot.com/search/label/MCQ : General Surgery

variation.
Answer: CD

DISCUSSION: Since fat contains little water, lean persons with a


proportionately greater muscle mass have a greater than expected
volume of total body water. Likewise, the female body habitus and
obesity contribute to decreased total body water percentage. The
highest proportion of total body water is found in newborn infants, and
total body water decreases steadily and significantly with age. The
actual figure for a healthy person is remarkably constant.

23. Which of the following statements about extracellular fluid are true?
A. The total extracellular fluid volume represents 40% of the body
weight.
B. The plasma volume constitutes one fourth of the total extracellular
fluid volume.
C. Potassium is the principal cation in extracellular fluid.
D. The protein content of the plasma produces a lower concentration of
cations than in the interstitial fluid.
E. The interstitial fluid equilibrates slowly with the other body
compartments.
Answer: B

DISCUSSION: The total extracellular fluid volume represents 20% of body


weight. The plasma volume is approximately 5% of body weight. Sodium
is the principal cation. The Gibbs-Donan equilibrium equation explains
the higher total concentration of cations in plasma. Except for joint
fluid and cerebrospinal fluid, the majority of the interstitial fluid exists
as a rapidly equilibrating component.

24. Which of the following statements are true of a patient with


hyperglycemia and hyponatremia?
A. The sodium concentration must be corrected by 5 mEq. per 100 mg.
per 100 ml. elevation in blood glucose.
B. With normal renal function, this patient is likely to be volume
overloaded.
C. Proper fluid therapy would be unlikely to include potassium
administration.
D. Insulin administration will increase the potassium content of cells.
E. Early in treatment adequate urine output is a reliable measure of
adequate volume resuscitation.
Answer: D

DISCUSSION: Each 100-mg. per 100 ml. elevation in blood glucose causes
a fall in serum sodium concentration of approximately 2 mEq. per liter.
Excess serum glucose acts as an osmotic diuretic, producing increased
urine flow, which can lead to volume depletion. Insulin therapy and the
correction of the patient's associated acidosis produce movement of
potassium ions into the intracellular compartment.

12 of 167 11-Aug-15 10:57 PM


MedCosmos Surgery: MCQ : General Surgery http://medcosmossurgery.blogspot.com/search/label/MCQ : General Surgery

25. Which of the following statements about respiratory acidosis are


true?
A. Compensation occurs by a shift of chloride out of the red blood cells.
B. Renal compensation occurs rapidly.
C. Retention of bicarbonate and increased ammonia formation are
normal compensatory mechanisms.
D. Narcotic administration is an unusual cause of respiratory acidosis.
E. The ratio of bicarbonate to carbonic acid is less than 20:1.
Answer: CE

DISCUSSION: Renal compensation for acute hypoventilation is relatively


slow. Depression of the respiratory center by morphine can lead to
respiratory acidosis. Renal retention of bicarbonate, ammonia
formation, and shift of chloride into red cells combine to increase the
ratio of bicarbonate to carbonic acid to 20:1.

26. Which of the following statements are true of elevated–anion gap


metabolic acidosis?
A. Hypoperfusion from the shock state rarely produces an elevated
anion gap.
B. Retention of sulfuric and phosphoric acids may lead to this form of
acidosis.
C. Copious diarrhea does not produce this condition.
D. Rapid volume expansion may produce this form of acidosis.
E. Use of lactated Ringer's solution is inappropriate in the treatment of
lactic acidosis.
Answer: BC

DISCUSSION: An elevated anion gap may be produced by lactic acidosis


from shock or by retention of inorganic acids from uremia. Lactated
Ringer's solution rapidly corrects the lactic acidosis from hypovolemia,
as lactate is converted to bicarbonate with hepatic reperfusion.
Bicarbonate loss from diarrhea and “dilutional acidosis” are non–anion
gap types of metabolic acidosis.

27. Which of the following is true of loss of gastrointestinal secretions?


A. Gastric losses are best replaced with a balanced salt solution.
B. Potassium supplementation is unnecessary in replacement of gastric
secretions.
C. Bicarbonate wasting is an unusual complication of a high-volume
pancreatic fistula.
D. Balanced salt solution is a reasonable replacement fluid for a small
bowel fistula.
E. A patient with persistent vomiting usually requires hyperchloremic
replacement fluids.
Answer: DE

DISCUSSION: Gastric secretions are relatively high in chloride and


potassium. Other than an isolated pancreatic fistula, gastrointestinal

13 of 167 11-Aug-15 10:57 PM


MedCosmos Surgery: MCQ : General Surgery http://medcosmossurgery.blogspot.com/search/label/MCQ : General Surgery

tract losses below the pylorus are best replaced by a balanced salt
solution. Although potassium concentrations are low, copious losses
require potassium supplementation to prevent hypokalemia.

28. Which of the following statements regarding hypercalcemia are


true?
A. The symptoms of hypercalcemia may mimic some symptoms of
hyperglycemia.
B. Metastatic breast cancer is an unusual cause of hypercalcemia.
C. Calcitonin is a satisfactory long-term therapy for hypercalcemia.
D. Severely hypercalcemic patients exhibit the signs of extracellular
fluid volume deficit.
E. Urinary calcium excretion may be increased by vigorous volume
repletion.
Answer: ADE

DISCUSSION: Markedly elevated serum calcium levels produce


polydipsia, polyuria, and thirst. Vigorous volume repletion and saline
diuresis correct the extracellular fluid volume deficit and promote the
urinary excretion of calcium. Metastatic breast cancer is the most
common cause of hypercalcemia, from bony metastasis. The calcitonin
effect on calcium is diminished with repeat administrations.

29. Which of the following statements about normal salt and water
balance are true?
A. The products of catabolism may be excreted by as little as 300 ml. of
urine per day.
B. The lungs represent the primary source of insensible water loss.
C. The normal daily insensible water loss is 600 to 900 ml.
D. Excessive cell catabolism causes significant loss of total body water.
E. In normal humans, urine represents the greatest source of daily water
loss.
Answer: CE

DISCUSSION: The skin is the primary source of insensible water loss.


Including losses from the lungs, this averages 600 to 900 ml. per day.
Catabolism liberates “water of solution.” In normal humans, urine
represents the greatest source of water loss. The patient deprived of
external access to water must still excrete a minimum of 500 to 800 ml.
of urine per day to expel the products of catabolism.

30. Which of the following is/are not associated with increased


likelihood of infection after major elective surgery?
A. Age over 70 years.
B. Chronic malnutrition.
C. Controlled diabetes mellitus.
D. Long-term steroid use.
E. Infection at a remote body site.
Answer: C

14 of 167 11-Aug-15 10:57 PM


MedCosmos Surgery: MCQ : General Surgery http://medcosmossurgery.blogspot.com/search/label/MCQ : General Surgery

DISCUSSION: Controlled diabetes mellitus has been shown repeatedly


not to be associated with increased likelihood of incisional infection
provided one avoids operations on body parts that may be ischemic or
neuropathic. Uncontrolled diabetes mellitus, such as ketoacidosis, is
associated with a dramatic increase in surgical infection. The other
parameters noted—age over 70, chronic malnutrition, regular steroid
use, and an infection at a remote body site—are well-recognized
adverse predictive factors and are identified in tables within the
chapter.

31. Which of the following are not determinants of a postoperative


cardiac complication?
A. Myocardial infarct 4 months previously.
B. Clinical evidence of congestive heart failure in a patient with 8.5 gm.
per dl. hemoglobin.
C. Premature atrial or ventricular contractions on electrocardiogram.
D. A harsh aortic systolic murmur.
E. Age over 70 years.
Answer: B

DISCUSSION: Clinical evidence of congestive heart failure in a patient


with 8.5 gm. per dl. hemoglobin concentration is a misleading sign.
Evidence of congestive failure is ordinarily a major risk factor, but in
this particular patient the anemia lends itself to correction by
preoperative transfusion with packed red blood cells, and often it is
found that congestive failure and the associated increased risks
disappear when the hemoglobin concentration is returned to the 12 gm.
per dl. or higher ratio. All other factors are overt signs of increased
likelihood of a postoperative cardiac event, the most ominous being a
myocardial infarction 4 months preoperatively or the presence of a
harsh aortic systolic murmur suggesting the presence of aortic stenosis.
Age over 70 years and the presence of premature atrial or ventricular
contractions on the electrocardiogram are less strong determinants of a
postoperative cardiac complication.

32. Rank the clinical scenarios in order of greatest likelihood of serious


postoperative pulmonary complications.
A. Transabdominal hysterectomy in an obese woman that requires 3
hours of anesthesia time.
B. Right middle lobectomy for bronchogenic cancer in a 65-year-old
smoker.
C. Vagotomy and pyloroplasty for chronic duodenal ulcer disease in a
50-year-old who had chest film findings of old, healed tuberculosis.
D. Right hemicolectomy in an obese 60-year-old smoker.
E. Modified radical mastectomy in a 58-year-old woman who is obese.
Answer: BDCAE

DISCUSSION: If one considers the constellation of risk factors for


pulmonary complications that is provided in tabular form in the

15 of 167 11-Aug-15 10:57 PM


MedCosmos Surgery: MCQ : General Surgery http://medcosmossurgery.blogspot.com/search/label/MCQ : General Surgery

accompanying chapter, one should readily recognize B, right middle


lobectomy for bronchogenic cancer in a 65-year-old smoker, as the
highest risk of a clinical situation for the likelihood of serious pulmonary
complications. The next in rank may be properly debated between
answer D and answer C. D, right hemicolectomy, is judged to have
somewhat greater likelihood of complications since the patient is older,
smokes, and is obese, although the procedure may be done through a
transverse or lower abdominal incision. C, vagotomy and pyloroplasty, is
viewed as being somewhat less serious since it is an upper abdominal
operation on an elective basis in a 50-year-old whose only abnormalities
include old, healed tuberculosis on a chest film. A very low risk of
pulmonary complication should follow a transabdominal hysterectomy
done through a lower abdominal incision in a woman whose only risk
factors are obesity and a 3-hour anesthesia time. The lowest risk
probably resides with the younger patient undergoing modified radical
mastectomy, whose only risk factor is obesity. This is particularly true
since this operation is conducted on the surface of the body, is
associated with relatively little postoperative pain, and provides free
and unrestricted respiratory function.

33. Rank the following laboratory tests and procedures in terms of their
relative value to a 65-year-old woman who is to undergo elective
resection of a sigmoid cancer.
A. Carcinoembryonic antigen (CEA).
B. Blood urea nitrogen (BUN).
C. Electrocardiogram (ECG).
D. Hemoglobin concentration (Hgb).
E. Serum creatinine (Cr).
F. Arterial blood oxygen tension (PaO 2) on room air.
G. Serum sodium concentration (Na+).
Answer: CDFEBAG

DISCUSSION: The most important test by far is the electrocardiogram,


with its capacity to indicate signs of occult heart disease. The second
most important evaluation is the hemoglobin concentration, which in
this patient may show an anemia related to chronic alimentary tract
blood loss that would require correction prior to safe induction of a
general anesthetic. Arterial blood gases vary from individual to
individual depending primarily on smoking habits and age. Accordingly,
each older person should have a resting baseline determination prior to
operation. Serum creatinine may show evidence of occult renal disease
and is substantially more useful than blood urea nitrogen, which is more
vulnerable to transient volume changes. Carcinoembryonic antigen is
important to know in many patients with cancer with respect to
postoperative follow-up since in some cases it may be an early herald of
recurrent disease. However, it has little to do with the patient's
preoperative assessment in terms of risk and preparation for an elective
operation. The presence of liver metastases, for example, can be
discovered with significant accuracy by palpation at the time of
operation, and an elevated carcinoembryonic antigen in no set of
circumstances would lead one to withhold colon resection with its relief

16 of 167 11-Aug-15 10:57 PM


MedCosmos Surgery: MCQ : General Surgery http://medcosmossurgery.blogspot.com/search/label/MCQ : General Surgery

of potential obstruction and bleeding. Finally, serum sodium


concentration in a 65-year-old woman who is admitted electively for
resection of the colon is always normal and would be of least value
among these tests.

34. Which of the following statements regarding whole blood transfusion


is/are correct?
A. Whole blood is the most commonly used red cell preparation for
transfusion in the United States.
B. Whole blood is effective in the replacement of acute blood loss.
C. Most blood banks in the United States have large supplies of whole
blood available.
D. The use of whole blood produces higher rates of disease transmission
than the use of individual component therapies.
Answer: B

DISCUSSION: Whole blood is effective as a replacement fluid for acute


blood loss because it provides both volume and oxygen-carrying capacity
(red blood cells). It is rarely used in the United States nowadays, and
most blood banks do not provide whole blood transfusions. It is
significantly more efficient to separate donated blood into its
components. In this manner, the red blood cell mass can be used to
provide oxygen-carrying capacity, the plasma can be used for factor
replacement, and the platelets and white cells can be used for patients
deficient in these components. The use of whole blood to replace acute
blood loss is associated with lower disease transmission rates than the
use of packed red blood cells, fresh frozen plasma, and platelets, each
from a different donor.

35. Which of the following statements about the preparation and


storage of blood components is/are true?
A. Solutions containing citrate prevent coagulation by binding calcium.
B. The shelf life of packed red blood cells preserved with CPDA-1 is
approximately 35 days at 1‫ ؛‬to 6‫ ؛‬C.
C. There are normal numbers of platelets in packed red blood cells
stored at 1‫ ؛‬to 6‫ ؛‬C for more than 2 days.
D. The storage lesion affecting refrigerated packed red blood cells
includes development of acidosis, hyperkalemia, and decreased
intracellular 2,3DPG (diphosphoglycerate).
Answer: ABD

DISCUSSION: After blood has been collected from a donor, it is


anticoagulated with a solution containing citrate, which acts by binding
calcium. Blood is then separated into its components. Packed red blood
cells stored at 1‫ ؛‬to 6‫ ؛‬C using CPDA-1 preservative have a shelf life of
35 days. There are essentially no functional platelets in refrigerated
blood stored at 1‫ ؛‬to 6‫ ؛‬C after approximately 48 hours in storage.
Refrigerated packed red blood cells undergo progressive changes termed
a storage lesion. Such changes include acidosis, hyperkalemia, and
decreased levels of 2,3-DPG, which are reversed after transfusion or

17 of 167 11-Aug-15 10:57 PM


MedCosmos Surgery: MCQ : General Surgery http://medcosmossurgery.blogspot.com/search/label/MCQ : General Surgery

produce effects other than those predicted based on the content of the
unit of blood.

36. Which of the following is/are acceptable reasons for the transfusion
of red blood cells based on currently available data?
A. Rapid, acute blood loss with unstable vital signs but no available
hematocrit or hemoglobin determination.
B. Symptomatic anemia: orthostatic hypotension, severe tachycardia
(greater than 120 beats per minute), evidence of myocardial ischemia,
including angina.
C. To increase wound healing.
D. A hematocrit of 26% in an otherwise stable, asymptomatic patient.
Answer: AB

DISCUSSION: Currently accepted guidelines for the transfusion of packed


red blood cells include acute ongoing blood loss, as might occur in an
injured patient, and the development of symptomatic anemia with
manifestations of decreased tissue perfusion related to decreased
oxygen-carrying capacity of the blood. This includes situations in which
the patient is unable to compensate for a decreased oxygen-carrying
capacity by the usual mechanisms, such as increased cardiac output.
Such patients develop myocardial dysfunction if an excessive demand is
placed on the heart. The patient should be transfused with packed red
blood cells, which afford added oxygen-carrying capacity. This decreases
the workload on the myocardium while providing the necessary oxygen-
delivery capability. The use of packed red blood cells to improve wound
healing or to improve the patient's sense of well-being is highly
questionable. No data support such a practice. In general, the use of a
transfusion trigger such as a hematocrit of 30% or hemoglobin of 10 gm.
per dl. constitutes a questionable indication for transfusion. If a patient
is asymptomatic and stable and has no risk of myocardial ischemia,
packed red blood cell transfusion should not be given based solely or
predominantly on a numerical value such as a hematocrit of 28%.

37. The transfusion of fresh frozen plasma (FFP) is indicated for which
of the following reasons?
A. Volume replacement.
B. As a nutritional supplement.
C. Specific coagulation factor deficiency with an abnormal prothrombin
time (PT) and/or an abnormal activated partial thromboplastin time
(APTT).
D. For the correction of abnormal PT secondary to warfarin therapy,
vitamin K deficiency, or liver disease.
Answer: CD

DISCUSSION: The use of FFP as a volume expander is not indicated.


There are currently several preparations (both crystalloid and colloid)
that are equally effective and do not carry the infectious and other risks
associated with the use of FFP. The use of FFP as a “nutritional”
supplement is to be condemned. Patients with specific deficiencies of

18 of 167 11-Aug-15 10:57 PM


MedCosmos Surgery: MCQ : General Surgery http://medcosmossurgery.blogspot.com/search/label/MCQ : General Surgery

coagulation factors generally benefit greatly from the infusion of FFP. In


cases of specific factor deficiency, other preparations may be more
appropriate, but FFP is generally immediately available and is effective
in most patients. Patients receiving warfarin therapy, those who have
vitamin K deficiency, and those with liver dysfunction have
abnormalities of the vitamin K–dependent factors II, VII, IX, and X, as
well as protein C and protein S.

38. In patients receiving massive blood transfusion for acute blood loss,
which of the following is/are correct?
A. Packed red blood cells and crystalloid solution should be infused to
restore oxygen-carrying capacity and intravascular volume.
B. Two units of FFP should be given with every 5 units of packed red
blood cells in most cases.
C. A “six pack” of platelets should be administered with every 10 units
of packed red blood cells in most cases.
D. One to two ampules of sodium bicarbonate should be administered
with every 5 units of packed red blood cells to avoid acidosis.
E. One ampule of calcium chloride should be administered with every 5
units of packed red blood cells to avoid hypocalcemia.
Answer: A

DISCUSSION: Patients who are suffering from acute blood loss require
crystalloid resuscitation as the initial maneuver to restore intravascular
volume and re-establish vital signs. If 2 to 3 liters of crystalloid solution
is inadequate to restore intravascular volume status, packed red blood
cells should be infused as soon as possible. There is no role for
“prophylactic infusion” of FFP, platelets, bicarbonate, or calcium to
patients receiving massive blood transfusion. If specific indications exist
patients should receive these supplemental components. In particular,
patients who have abnormal coagulation tests and have ongoing
bleeding should receive FFP. Patients who have depressed platelet
counts along with clinical evidence of oozing (microvascular bleeding)
benefit from platelet infusion. Sodium bicarbonate is not necessary,
since most patients who receive blood transfusion ultimately develop
alkalosis from the citrate contained in stored red blood cells. The use of
calcium chloride is usually unnecessary unless the patient has depressed
liver function, ongoing prolonged shock associated with hypothermia, or,
rarely, when the infusion of blood proceeds at a rate exceeding 1 to 2
units every 5 minutes.

39. Hemostasis and the cessation of bleeding require which of the


following processes?
A. Adherence of platelets to exposed subendothelial glycoproteins and
collagen with subsequent aggregation of platelets and formation of a
hemostatic plug.
B. Interaction of tissue factor with factor VII circulating in the plasma.
C. The production of thrombin via the coagulation cascade with
conversion of fibrinogen to fibrin.
D. Cross-linking of fibrin by factor XIII.

19 of 167 11-Aug-15 10:57 PM


MedCosmos Surgery: MCQ : General Surgery http://medcosmossurgery.blogspot.com/search/label/MCQ : General Surgery

Answer: ABCD

DISCUSSION: Hemostasis requires the interaction of platelets with the


exposed subendothelial structures at the site of injury followed by
aggregation of more platelets in that area. Interactions between
endothelial cell and subendothelial tissue factor with factor VII activate
the coagulation cascade. The end product is large amounts of thrombin
that catalyze the conversion of fibrinogen into fibrin. Fibrin thus formed
is cross-linked by factor XIII to form a stable clot that incorporates the
platelet plug and fibrin thrombus into a stable clot.

40. Which of the statements listed below about bleeding disorders


is/are correct?
A. Acquired bleeding disorders are more common than congenital
defects.
B. Deficiencies of vitamin K decrease production of factors II, VII, IX,
and X, protein C, and protein S.
C. Hypothermia below 32‫؛‬C rarely causes a bleeding disorder.
D. Von Willebrand's disease is a very uncommon congenital bleeding
disorder.
Answer: AB

DISCUSSION: Acquired bleeding disorders are significantly more common


than congenital bleeding defects. Vitamin K deficiency may be related
to malnutrition or competitive inhibition of the production of the
vitamin K–dependent factors II, VII, IX, X, protein C, and protein S by
warfarin (Coumadin). Hypothermia causes significant platelet
dysfunction with a significant bleeding disorder in many patients. It is
among the least recognized causes of altered coagulation in surgical
patients. Von Willebrand's disease is a relatively common disorder of
bleeding and is generally undetectable by routine screening methods.

41. The evaluation of a patient scheduled for elective surgery should


always include the following as tests of hemostasis and coagulation:
A. History and physical examination.
B. Complete blood count (CBC), including platelet count.
C. Prothrombin time (PT) and activated partial thromboplastin time
(APTT).
D. Studies of platelet aggregation with adenosine diphosphate (ADP) and
epinephrine.
Answer: A

DISCUSSION: The evaluation of most patients scheduled for elective


surgery who do not have a history of significant bleeding disorders is
somewhat controversial. An adequate history and physical examination
screen out most patients with bleeding problems. For patients who are
scheduled to undergo a major surgical procedure, it is advisable to
obtain a CBC and platelet count, as well as a PT and APTT level. This
detects a large number of bleeding disorders but does not rule out all
possible causes of perioperative bleeding. Studies of platelet

20 of 167 11-Aug-15 10:57 PM


MedCosmos Surgery: MCQ : General Surgery http://medcosmossurgery.blogspot.com/search/label/MCQ : General Surgery

aggregation are indicated only for patients who are suspected of having
qualitative defects of platelet function (e.g., von Willebrand's disease).

42. Which of the following statements regarding the transmission of


infectious agents through blood transfusions is/are true?
A. The transmission rates for human immunodeficiency virus (HIV) have
been decreasing progressively since the early 1980s.
B. The transmission rates of hepatitis have been decreasing steadily
since the 1980s.
C. Cytomegalovirus (CMV) is the infectious agent most commonly
transmitted in blood.
D. Severely immunocompromised patients (such as patients undergoing
transplantation) should receive specially screened blood products.
Answer: ABCD

DISCUSSION: The incidence of both HIV and hepatitis transmitted via


blood transfusions has been steadily decreasing since the 1980s. This is
related to improved methods for detection and increased awareness of
surrogate markers of disease. The currently available techniques for the
detection of HIV are highly effective, provided the donor is not in the
“window” before the formation of specific antibody. The surrogate
markers for hepatitis C, as well as the specific assays for the organism,
are now sufficiently refined to allow the detection of a large percentage
of hepatitis C infection in donated blood. Screening for hepatitis B
surface antigen has effectively eliminated the transmission of hepatitis
B through blood products in most cases. CMV is the most commonly
transmitted infectious agent in blood. Since a large percentage of the
population carry the virus, routine screening is not performed for this
organism; however, severely compromised patients such as those
undergoing transplantation should receive CMV-negative blood products.

43. The most common cause of fatal transfusion reactions is:


A. An allergic reaction.
B. An anaphylactoid reaction.
C. A clerical error.
D. An acute bacterial infection transmitted in blood.
Answer: C

DISCUSSION: The most common cause of fatalities related to transfusion


reactions result from ABO-incompatible transfusion related to clerical
error. Most such reactions occur if a type O person receives type A red
cells owing to a clerical error that occurs either at the time the blood
sample was drawn, during processing in the laboratory, or at the time a
unit is administered. The importance of extremely careful labeling,
transfer, and handling of specimens and of cross-matched blood
products cannot be overemphasized. Allergic and other reactions are
common but rarely fatal. The transmission of bacterial organisms (e.g.,
Staphylococcus aureus) has been reported especially with platelet
concentrates maintained at or near room temperature. Fortunately,
such reactions are rare.

21 of 167 11-Aug-15 10:57 PM


MedCosmos Surgery: MCQ : General Surgery http://medcosmossurgery.blogspot.com/search/label/MCQ : General Surgery

44. Which of the following statements about the coagulation cascade


is/are true?
A. The intrinsic pathway of coagulation is the predominant pathway in
vivo for hemostasis and coagulation.
B. The intrinsic pathway beginning with the activation of factor XII is
the predominant in vivo mechanism for activation of the coagulation
cascade.
C. Deficiencies of factor VIII and IX cause highly significant coagulation
abnormalities.
D. Deficiencies of factor XII cause severe clinical bleeding syndromes.
Answer: AC

DISCUSSION: Although it was previously held that two somewhat distinct


pathways existed for the activation of the coagulation cascade, it is now
recognized that the predominant mechanism for coagulation in vivo is
the “extrinsic pathway.” Tissue factor is exposed in the subendothelial
tissues when endothelial cell injury occurs. Tissue factor then tightly
binds factor VII circulating in the plasma and activates the coagulation
cascade. Factor VIII and factor IX deficiency cause the clinical
syndromes of hemophilia A and hemophilia B, respectively. Both of these
disorders involve very severe clinical bleeding disorders, whereas
deficiencies of factor XII do not generally cause clinically significant
bleeding. This further emphasizes the secondary role that the “intrinsic
pathway” plays in coagulation.

45. A major problem in nutritional support is identifying patients at risk.


Recent studies suggest that these patients can be identified. Which of
the following findings identify the patient at risk?
A. Weight loss of greater than 10% over 2 to 4 months.
B. Serum albumin of less than 3 gm. per 100 ml. in the hydrated state.
C. Malnutrition as identified by global assessment.
D. Serum transferrin of less than 220 mg. per 100 ml.
E. Functional impairment by history.
Answer: ABCDE

DISCUSSION: All of these are at least partially correct. It is not clear


whether weight loss of 10% or 15% is the required threshold, but it
certainly is close. Serum albumin of less than 3 gm per 100 ml. remains
the most constant identifier of patients at risk in the literature and has
been so for years. Global assessment in the hands of an experienced
investigator is quite efficacious at identifying persons at risk. Serum
transferrin is certainly a confirmatory identifier of patients with
malnutrition—and may be even a primary one. Graham Hill and his
co-workers have pioneered the concept of global assessment using
functional parameters, and in the hands of an experienced observer is
quite a reasonable way of approaching and identifying patients at risk.

46. Essential fatty acid deficiency may complicate total parenteral

22 of 167 11-Aug-15 10:57 PM


MedCosmos Surgery: MCQ : General Surgery http://medcosmossurgery.blogspot.com/search/label/MCQ : General Surgery

nutrition (TPN). Which of the following statements are true?


A. Essential fatty acid deficiency may be prevented by the
administration of 1% to 2% of total calories as fat emulsion.
B. Fat-free parenteral nutrition results in the appearance of plasma
abnormalities, indicating essential fatty acid deficiency, within 7 to 10
days of initiation.
C. An abnormal plasma eicosatrienoic-arachidonic acid ratio is always
associated with essential fatty acid deficiency.
D. Following initiation of fat-free parenteral nutrition, dry, scaly skin
associated with a maculopapular rash indicates essential fatty acid
deficiency.
Answer: BD

DISCUSSION: Biochemical evidence of essential fatty acid deficiency may


occur as early as 7 to 10 days following initiation of fat-free parenteral
nutrition. The decrease in arachidonic acid in plasma and the
appearance of the abnormal eicosatrienoic acid may yield the earliest
indication of prostaglandin deficiency; it is not absolute. Decreased
intraocular pressure, another early indication of prostaglandin
deficiency, may appear as soon as 7 days following initiation of fat-free
parenteral nutrition. While my current practice is to give at least 500
ml. of 10% lipid emulsion daily to provide 20% to 25% of total calories to
support hepatic protein synthesis, as little as 4% to 6% of total daily
calories as fat prevents essential fatty acid deficiency. Practically, this
may be undertaken by the administration of 500 ml. of 10% lipid three
times weekly. The appearance of eicosatrienoic acid and a decrease in
arachidonic acid, and a change in ratio, is not essential to the diagnosis
of essential fatty acid deficiency, but this plasma abnormality is often
present.

47. It is stated that enteral nutrition is safer than parenteral nutrition.


Which of the following may be complications of enteral nutrition?
A. Hyperosmolar, nonketotic coma.
B. Vomiting and aspiration.
C. Pneumatosis cystoides intestinalis.
D. Perforation and peritonitis.
Answer: ABCD

DISCUSSION: It is not necessarily true that enteral nutrition is safer than


parenteral nutrition, and it may in fact be associated with a higher risk
of death than parenteral nutrition. Specifically, a well-run parenteral
nutrition service should not be associated with significant mortality,
except for the occasional death due to undetected yeast infection. On
the other hand, enteral nutrition, especially if not carried out safely,
can result in significant mortality. The most common of the severe
complications of enteral nutrition result from the gastrostomy, or tube
feedings into the stomach. Sudden changes in gastric motility, such as
those associated with sepsis, may result in aspiration. Nasoenteric or
nasoduodenal tubes help prevent this complication, as does shutting off
enteral feedings between the hours of 11 P.M. and 7 A.M. It is also
essential to keep the patient's head elevated 30 degrees. Also necessary

23 of 167 11-Aug-15 10:57 PM


MedCosmos Surgery: MCQ : General Surgery http://medcosmossurgery.blogspot.com/search/label/MCQ : General Surgery

is the use of extreme care when initiating enteral nutrition. If


hypertonic material is given into the stomach, one can increase
osmolality followed by an increase in volume. If, however, the material
is given into the small bowel, volume must be increased first and then
tonicity, with the expectation that osmolality greater than 400 or 500
mOsm per liter may never be achieved without provoking severe
diarrhea. If care is not taken with the initiation of enteral nutrition,
massive diarrhea may result, including fluid loss, the absorption of
enormous amounts of carbohydrate into the circulation with inadequate
fluid to support it, and the development of hyperosmolar, nonketotic
coma. Alternatively, severe unremitting diarrhea may result in necrosis
of the intestinal wall, the appearance of pneumatosis cystoides
intestinalis, and, finally, perforation and death. All of these
complications may be prevented by judicious use of enteral nutrition
with the same care one uses for parenteral nutrition.

48. It has been suggested that enterocyte-specific fuels be utilized for


all patients receiving parenteral nutrition. Theoretically, the benefits of
such fuels include:
A. Glutamine increases gut mucosal protein content and wall thickness.
B. Butyrate increases jejunal mucosal protein content and wall
thickness.
C. The short-chain fatty acids—butyrate, propionate, and acetate—are
useful in supporting ileal mucosal protein content and thickness.
D. The use of glutamine-enriched solutions for parenteral nutrition for
patients with chemotherapy toxicity or radiation enteritis is without
hazards.
Answer: NONE IS ENTIRELY TRUE

DISCUSSION: The use of enterocyte-specific fuels is part of a new and


potentially exciting phase of “nutritional pharmacology” in parenteral
nutrition; however, exciting as the research may be, the use of such
fuels is by no means acceptable for indiscriminate use at present.
Though some studies have shown that the provision of glutamine in
amounts up to 2% in standard parenteral nutrition solutions increases
both jejunal and ileal mucosal protein content, cell wall thickness, and
DNA content, this has not been the case in all studies, and this reported
effect seems very dependent on experimental design. In many of the
studies that have shown such an effect, 2% glutamine has been used to
replace virtually all nonessential amino acids, probably initiating a
deficiency state. The beneficial effects seen with glutamine are far less
impressive than those seen with epidermal growth factor, for example,
and disappear entirely when a different experimental design is used in
which 2% glutamine is added to an adequate amino acid formulation in
which glutamine does not replace nonessential amino acids but is added
to them. Nonetheless, the use of enterocyte-specific fuels, specifically
glutamine, is potentially exciting and should be carefully investigated.
More striking are the results that follow massive bowel resection,
radiation enteritis, and chemotherapy toxicity. Glutamine may help the
small bowel regenerate more quickly, enabling more rapid use of the
small bowel for nutrition. It should be pointed out, however, that

24 of 167 11-Aug-15 10:57 PM


MedCosmos Surgery: MCQ : General Surgery http://medcosmossurgery.blogspot.com/search/label/MCQ : General Surgery

glutamine is a fuel utilized by many tumors and, thus, one runs the risk
of stimulating the growth of the tumor with excessive glutamine. The
short-chain fatty acids, produced from bacterial fermentation of soluble
pectin, may be useful in both the maintenance of colonocyte-specific
nutrition and, in the case of butyrate, ileal enterocyte nutrition.

49. Essential amino acids have been advocated as standard therapy for
renal failure. Which of the following statements are true?
A. Increased survival from acute renal failure has been reported with
both essential and nonessential amino acid therapy of patients in renal
failure.
B. Essential amino acids retard the rise of blood urea nitrogen (BUN)
secondary to decreased urea appearance.
C. Essential amino acids and hypertonic dextrose are a convenient form
of therapy for hyperkalemia.
D. Essential amino acids decrease BUN and creatinine to the same
degree as solutions containing excessive nonessential amino acids.
Answer: BC

DISCUSSION: Essential amino acids and hypertonic dextrose, as opposed


to hypertonic dextrose alone, was reported by Abel and co-workers to
be associated with a decreased mortality rate in mostly surgical patients
with acute tubular necrosis. The most significant improvement in
mortality, as compared with the control group receiving hypertonic
dextrose, was among patients who required dialysis (i.e., the more
severely affected patients). Another group responding favorably to
treatment includes patients with nonoliguric renal failure whose need
for dialysis is not clearly established. The effect of essential amino acids
in preventing a rise in BUN, as well as its beneficial effect in preventing
hyperkalemia, may obviate dialysis in such patients. With increasing
amounts of nonessential amino acids, BUN increases, and thus, dialysis
is required. Prospective randomized studies comparing the use of
essential versus nonessential amino acids in patients with acute renal
failure have not been carried out in sufficient numbers to yield answers
to this question.

50. A modified amino acid solution with increased equimolar


branched-chain amino acids and decreased aromatic amino acids has
been proposed for patients with hepatic insufficiency. Which of the
following statements is/are true?
A. This formulation is proposed for the use of patients with fulminant
hepatitis.
B. Nitrogen balance is achieved in such patients with amounts of 40 gm.
of amino acids per 24 hours.
C. The use of 80 to 100 gm. of such solutions is associated with hepatic
encephalopathy.
D. In some studies of surgical patients, improvements in mortality have
been reported.
Answer: D

25 of 167 11-Aug-15 10:57 PM


MedCosmos Surgery: MCQ : General Surgery http://medcosmossurgery.blogspot.com/search/label/MCQ : General Surgery

DISCUSSION: The use of modified amino acid solutions is based on the


false neurotransmitter hypothesis of the cause of hepatic coma.
According to this hypothesis, the imbalance between aromatic and
branched-chain amino acids in the plasma results in abnormally high
levels of the toxic aromatic amino acids in the brain, thus provoking
hepatic encephalopathy. The use of modified amino acid mixtures, with
glucose as the calorie base, has been associated in a number of studies
with improvement in encephalopathy. Meta-analysis has concluded that
the use of such solutions is indicated as therapy for hepatic
encephalopathy but has been proposed only for hepatic encephalopathy
complicating acute exacerbation of chronic liver disease. Although there
are a few anecdotal reports of beneficial effects on hepatic
encephalopathy of acute fulminant hepatitis, the use of such a solution
has not been advocated, but such a modified solution is tolerated better
than standard amino acid mixtures in patients requiring TPN. In some
studies, particularly in complicated surgical cases, the use of a
high–branched-chain, low–aromatic amino acid solution has been
associated with lower mortality. These statements are true only for
studies in which the modified solutions are given with hypertonic
glucose as a calorie base. Studies in which lipid was the principal calorie
source have not revealed such improvements in mortality. In recent
studies, giving an aromatic amino acid–deficient, branched-chain amino
acid–enriched solution to patients about to undergo resection of the
liver has proved particularly efficacious in a group of patients with
cirrhosis, decreasing morbidity and showing a trend toward decreased
mortality.

51. In the nutritional support of patients with cancer, which of the


following statements is/are true?
A. Nutritional support benefits the patient's lean body mass but does not
enable the tumor to grow.
B. In experimental animals, the growth of implanted tumors is directly
proportional to the amount of calories and protein supplied.
C. Prospective randomized trials of nutritional support utilizing
chemotherapy and radiation therapy have revealed benefits to patients
receiving total parenteral nutrition.
D. Studies of nutritional support for patients with cancer about to
undergo surgery revealed decreased morbidity and mortality, especially
morbidity from sepsis.
Answer: B

DISCUSSION: The problem with the patient with cancer is a very vexing
one. Clearly, one of the metabolic effects of cancer, cachexia, affects
patients in the last quartile of their disease and makes such patients
intolerant of chemotherapy, radiation therapy, and, in many cases,
operative procedures. Total parenteral nutrition (TPN) has been
proposed as a means of reversing cachexia and enabling patients to
better tolerate surgery, chemotherapy, and radiation therapy. In
experimental animals, it is clear that the provision of calories and
protein, especially in excessive amounts, is associated with the more
rapid growth of tumors and decreased survival, especially in the group

26 of 167 11-Aug-15 10:57 PM


MedCosmos Surgery: MCQ : General Surgery http://medcosmossurgery.blogspot.com/search/label/MCQ : General Surgery

that is overfed in the extreme. There is also evidence suggesting that


overfeeding, or at least TPN, may result in increased growth (or at least
change cell kinetics) in patients who are overnourished with TPN. Of the
randomized prospective trials that have been carried out, no trial
utilizing chemotherapy or radiation therapy has revealed a survival
advantage for patients receiving TPN. Indeed, in Shamberger's study,
there is a suggestion that the tumor-free interval following treatment of
lymphoma may be shorter in patients receiving TPN. In patients
undergoing surgery, however, especially those who are severely
malnourished (as recently revealed in the VA study) or in patients with
major procedures such as esophagogastrectomy (as in Muller's study),
evidence suggests that TPN is beneficial. In a late follow-up in Muller's
study, there was no apparent increase in recurrence, and the survival
rate was the same, despite much higher mortality in the non-TPN group.
This suggests that any improved survival following operation may have
been offset by an increased late recurrence rate, although it is difficult
to reach this conclusion. In summary, for patients with cancer TPN
probably nourishes the tumor as well as the host. Nonetheless, in
severely malnourished patients provision of TPN from 5 to 10 days
preoperatively may increase survival and decrease morbidity.
Overfeeding must be avoided. Future studies will undoubtedly reveal
that there are certain nutrients that tumors require, which probably
should be best avoided.

52. Glucose overload results in increased CO 2 production. Which of the


following statements are true?
A. In patients with respiratory insufficiency, administration of glucose as
a principal calorie source is contraindicated.
B. In patients with pulmonary infection and sepsis, calorie support
should consist of 95% fat and 5% glucose.
C. In Askanazi's study, increased CO 2 production and difficulty in
weaning was associated only with pronounced overfeeding.
D. CO 2 production should be measured in most patients who are
supported by respirators in intensive care units and are receiving
nutritional support.
Answer: C

DISCUSSION: Few papers have excited as much interest as that by


Askanazi, Kinney, and co-workers, which demonstrated that glucose
calories given to patients with severe respiratory impairment may result
in difficulty in weaning from a respirator. Subsequent research has
suggested, however, that this occurs only with severe overfeeding, when
the respiratory quotient is greater than 1 and calories are excessive. If
one examines the conditions under which Askanazi's patients were
studied, these were a group of septic, depleted patients who were
taken from almost no nutritional support to a caloric supply of 2.25
times their caloric requirement, most of the calories consisting of
glucose. Suffice it to say that, in patients with impaired respiratory
function, one should measure VCO2 and, when VCO2 is significantly
elevated and appears to interfere with weaning, decrease the amount
of glucose calories and increase the amount of fat. If one measures or

27 of 167 11-Aug-15 10:57 PM


MedCosmos Surgery: MCQ : General Surgery http://medcosmossurgery.blogspot.com/search/label/MCQ : General Surgery

estimates calorie requirements and does not overfeed, lipid can be


utilized for 25% of the caloric requirement and glucose for the
remainder, without much fear of excessive CO 2 production.

53. Hepatic abnormalities have been noted in adults since the beginning
of hyperalimentation. Which of the following statements are true?
A. Hepatic steatosis appears to be associated with an overload of
glucose.
B. Hepatic steatosis is usually associated with abnormalities in hepatic
enzymes.
C. Hyperbilirubinemia is inevitably associated with hepatic steatosis.
D. Abnormalities in the portal insulin-glucagon ratio are thought to be
causative of hepatic steatosis in experimental animals.
Answer: AD

DISCUSSION: The most common metabolic complication of TPN in adults


is hepatic steatosis. Unlike the hepatic abnormalities in children, which
may progress to cholestasis, liver damage, and in some cases death,
hepatic steatosis, or fatty infiltration of the liver with triglycerides,
appears to be a rather benign complication. It may be, but is not
necessarily, associated with hepatic enzymatic abnormalities, which
usually occur in the first week, peak at the third week, and generally
disappear by the sixth week of parenteral nutrition. Abnormalities in
the transaminases are most common, with alkaline phosphatase also
being elevated, but there is no correlation between the degree of fatty
infiltration and enzymatic abnormalities. Fatty infiltration appears to be
largely vacuolization with increased storage of triglycerides. Hepatic
steatosis is almost always associated with an overload of glucose.
Recent studies in experimental animals have suggested that the portal
insulin-glucagon ratio, which is elevated under these circumstances,
may be causally related to hepatic steatosis. Insulin is the leading
storage enzyme and is responsible for lipogenesis. The presence of
insulin inhibits lipolysis. Glucagon, on the other hand, results in the
mobilization of hepatic lipid. The liver “sees” the portal vein insulin-
glucagon ratio. Excesses of insulin elicited by hypertonic dextrose
increase lipid deposition in the liver, whereas glucagon, which is elicited
by certain amino acids, results in the mobilization of hepatic lipid.

54. Which of the following statements about the presence of gallstones


in diabetes patients is/are correct?
A. Gallstones occur with the same frequency in diabetes patients as in
the healthy population.
B. The presence of gallstones, regardless of the presence of symptoms,
is an indication for cholecystectomy in a diabetes patient.
C. Diabetes patients with gallstones and chronic biliary pain should be
managed nonoperatively with chemical dissolution and/or lithotripsy
because of severe complicating medical conditions and a high operative
risk.
D. The presence of diabetes and gallstones places the patient at high
risk for pancreatic cancer.

28 of 167 11-Aug-15 10:57 PM


MedCosmos Surgery: MCQ : General Surgery http://medcosmossurgery.blogspot.com/search/label/MCQ : General Surgery

E. Diabetes patients with symptomatic gallstones should have prompt


elective cholecystectomy, to avoid the complications of acute
cholecystitis and gallbladder necrosis.
Answer: E

DISCUSSION: Gallstones have been found to be very prevalent in patients


with type II (non–insulin-dependent) diabetes mellitus, perhaps related
to the dyslipoproteinemia in such patients. Although the complications
of acute cholecystitis (infection, sepsis, gangrene of the gallbladder) are
more common in diabetics, a decision-analysis study has shown that
prophylactic cholecystectomy cannot be justified since the risk of
morbidity and/or mortality from the cholecystectomy procedure is as
great as that of complications or death from acute cholecystitis.
Patients who become symptomatic should be promptly prepared and
should undergo elective cholecystectomy, because an emergency
operation in these patients with comorbid conditions, especially
coronary artery disease, has substantial added mortality associated with
it. There is no causal relationship between diabetes and pancreatic
cancer.

55. Intensive insulin therapy:


A. Prevents the aggressive development of atherosclerosis in diabetic
patients.
B. Is not associated with unawareness of hypoglycemia.
C. Improves peripheral neuropathy.
D. Improves established retinopathy and nephropathy.
E. Is indicated in all patients with non–insulin-dependent diabetes
mellitus (NIDDM).
Answer: C

DISCUSSION: Intensive insulin therapy is indicated in patients with IDDM


who can actively participate in their own management and the
attainment of the goals set for their blood glucose and glycosylated
hemoglobin (HgA1 c) levels. Because the main complication of intensive
therapy is iatrogenic hypoglycemia, this mode of treatment is not
indicated for patients with NIDDM, who often have coexisting medical
conditions such as coronary artery disease and who tolerate
hypoglycemia poorly. There is little or no evidence that macrovascular
disease is affected by intensive insulin therapy, and the added weight
gain and hyperinsulinemia associated with the therapy may worsen
atherosclerosis. Unawareness of hypoglycemia is directly related to a
recent hypoglycemia episode, so patients treated intensively are often
unaware of the problem. Intensive therapy does not improve established
retinopathy or nephropathy but slows or prevents progression of these
complications; however, better glucose control may improve peripheral
neuropathy.

56. Which of the following statements about hypertension in diabetes


patients is/are correct?
A. Hypertension worsens the macrovascular disease of diabetes

29 of 167 11-Aug-15 10:57 PM


MedCosmos Surgery: MCQ : General Surgery http://medcosmossurgery.blogspot.com/search/label/MCQ : General Surgery

patients.
B. Hypertension accelerates the progression of diabetic nephropathy.
C. Hypertension is associated with sodium retention in diabetes
patients.
D. Angiotensin-converting enzyme (ACE) inhibitors should be used in all
patients with chronic hyperglycemia, regardless of the presence of
hypertension.
E. Diuretics, as single-drug therapy, are not indicated in the treatment
of hypertension in diabetes patients.
Answer: ABCDE

DISCUSSION: All of the answers listed are correct. By damaging


endothelial cells, hypertension worsens macrovascular disease in all
patients but especially in diabetics. Hypertension dramatically
accelerates the onset and progression of diabetic renal disease and
proteinuria, and this phenomenon can be slowed or prevented by a
combination of treatment modalities, including ACE inhibitors, which
dilate efferent glomerular vessels and lower intraglomerular pressure.
Despite sodium retention in diabetes patients, single-drug therapy with
a diuretic is not indicated because the chronic state of dehydration in
such patients may become worse.

57. What is the major determinant in an individual patient's risk for


perioperative complications?
A. The surgical procedure.
B. The length of the surgical procedure.
C. The anesthetic technique (e.g., general, regional).
D. The length of anesthesia.
E. All of the above.
Answer: A

DISCUSSION: The planned surgical procedure is the major determining


factor in assessing an individual patient's risk for perioperative
complications and in deciding which anesthetic technique will be most
appropriate. Good communication between the surgeon and the
anesthesiologist is vital, as the surgeon knows better than anyone else
the extent of the operation and the length of time it will require.

58. Which of the following are considered routine intraoperative


monitors?
A. Temperature probe.
B. Electrocardiogram.
C. Capnograph.
D. Blood pressure cuff.
E. Foley catheter.
Answer: ABD

DISCUSSION: The American Society of Anesthesiologists requires that the


patient's ventilation, circulation, oxygenation, and temperature be
continually monitored during all anesthetics. Routine monitors are

30 of 167 11-Aug-15 10:57 PM


MedCosmos Surgery: MCQ : General Surgery http://medcosmossurgery.blogspot.com/search/label/MCQ : General Surgery

considered to be a temperature probe, electrocardiogram, pulse


oximetry, and blood pressure cuff.

59. Muscle relaxants can be used for which of the following?


A. To facilitate intubation.
B. To provide optimal surgical conditions.
C. To optimize ventilator support.
D. To provide sedation.
Answer: ABC

DISCUSSION: Muscle relaxants are administered to facilitate


endotracheal intubation, to provide the surgeon with optimal working
conditions during anesthesia, and to optimize mechanical ventilator
support in some patients. They do not produce analgesia, sedation, or
amnesia. Therefore, muscle paralysis should not be performed without
sedation or general anesthesia.

60. Local anesthetics:


A. Inhibit transmission of nerve impulses by increasing sodium
membrane permeability and the displacement of ionized calcium.
B. Are classified as amides or esters.
C. Produce peripheral vasodilation.
D. Are weak acids.
Answer: BC

DISCUSSION: Local anesthetics act within the nerve membrane, where


they inhibit transmission of nerve impulses by reducing sodium
membrane permeability and the displacement of ionized calcium. All
local anesthetics consist of a hydrophilic region and a hydrophobic
region separated by an alkyl chain. The bond of the alkyl chain is either
an ester or an amide, and these drugs are classified based on this bond.
All local anesthetics except cocaine produce vasodilatation and are
weak bases.

61. Absolute indications for a double-lumen endotracheal tube during


thoracic surgery are:
A. Massive hemorrhage from one lung.
B. Unilateral lung infection.
C. Facilitation of surgical exposure.
D. Unilateral bronchopulmonary lavage.
E. All of the above.
Answer: ABD

DISCUSSION: The absolute indications for a double-lumen tube are for


the purposes of protecting one lung from the other. These indications
include ventilation with a bronchopleural fistula, massive hemorrhage
from one lung, pulmonary air cyst resection, unilateral lung infection,
and unilateral bronchopulmonary lavage. Relative indications include
facilitation of surgical exposure, for pneumonectomy, upper lobectomy,

31 of 167 11-Aug-15 10:57 PM


MedCosmos Surgery: MCQ : General Surgery http://medcosmossurgery.blogspot.com/search/label/MCQ : General Surgery

and thoracic aneurysm repair.

62. Determinants of cerebral blood flow include:


A. Preoperative neurologic dysfunction.
B. Arterial CO 2 tension.
C. Arterial O 2 tension.
D. Systemic arterial pressure.
E. All of the above.
Answer: BCD

DISCUSSION: Determinants of cerebral blood flow include arterial CO 2


and O 2 tensions, systemic arterial pressure, and temperature. Other
factors that may affect cerebral blood flow and intracranial pressure are
head position, jugular venous obstruction, and positive end-expiratory
pressure.

63. Discharge criteria following ambulatory surgery include:


A. Ability to eat solid food.
B. Stable vital signs.
C. Ability to ambulate.
D. Ability to have protective airway reflexes.
Answer: BCD

DISCUSSION: Discharge criteria following ambulatory surgery include the


patient's being fully awake and oriented, the ability to have protective
airway reflexes, stable vital signs, adequate hydration with the ability
to hold down oral intake, the ability to ambulate, and adequate pain
control. All patients must have a competent person with them to
transport them—and ideally to stay with them on the first postoperative
night.

64. Advantages of patient-controlled analgesia (PCA) include:


A. Immediate medication delivery.
B. Less contact with nursing staff.
C. Rapid onset of analgesia.
D. Patient control over pain medication.
E. All of the above.
Answer: ACD

DISCUSSION: Advantages of PCA are immediate medication delivery,


rapid onset of analgesia, and patient control over pain medication.
Disadvantages of PCA are less contact with nursing staff and patients'
fears that they could inadvertently administer an overdose or possibly
become addicted to the opioid.

65. Advantages of epidural analgesia include:


A. Earlier mobilization after surgery.
B. Earlier return of bowel function.

32 of 167 11-Aug-15 10:57 PM


MedCosmos Surgery: MCQ : General Surgery http://medcosmossurgery.blogspot.com/search/label/MCQ : General Surgery

C. Shorter hospitalizations.
D. Decreased stress response to surgery.
E. All of the above.
Answer: E

DISCUSSION: Epidural analgesia include excellent pain relief, decreased


sedation with more rapid recovery to presurgical levels of
consciousness, earlier mobilization after surgery with increased ability
to co-operate with respiratory therapy and physical therapy. Following
vascular surgery epidural analgesia may also improve graft flow through
mild sympathetic blockade. Earlier return of bowel function, decreased
stress response, shorter hospitalizations, and decreased morbidity have
all been associated with epidural analgesia.

66. Ketorolac:
A. Is a nonsteroidal anti-inflammatory drug (NSAID) approved for
intravenous, intramuscular, and oral administration.
B. Can be used indefinitely for postoperative analgesia.
C. Can cause renal dysfunction.
D. May decrease surgical blood loss.
Answer: AC

DISCUSSION: Ketorolac tromethamine, an NSAID, is approved by the FDA


for intravenous, intramuscular, and oral administration. The agent is an
effective analgesic with minimal side effects; however, ketorolac, like
all NSAIDs, can enhance surgical bleeding and cause renal and platelet
dysfunction. Additionally, it is recommended that ketorolac should not
be used for more than 5 consecutive days.

67. Factors that decrease collagen synthesis include all of the following
except:
A. Protein depletion.
B. Infection.
C. Anemia.
D. Advanced age.
E. Hypoxia.
Answer: C

DISCUSSION: Collagen synthesis, an integral part of wound healing, is


affected by many local and systemic factors. Protein depletion impairs
fibroplasia. Hypoproteinemia leads to diminution of fibroblast
proliferation, proteoglycan and collagen synthesis, angiogenesis, and
wound remodeling. Although anemia was once believed to be a
significant cause of wound disruption, studies have shown that, in the
absence of malnutrition or hypovolemia, anemia with a hematocrit
greater than 15% does not interfere with wound healing. In contrast,
molecular oxygen is critical for collagen synthesis because it is one of
the factors required for the hydroxylation of lysine and proline. Also,
hypoxia favors wound infection. The role of age in collagen synthesis is
not clear, but the incidence of wound failure and incisional hernias is

33 of 167 11-Aug-15 10:57 PM


MedCosmos Surgery: MCQ : General Surgery http://medcosmossurgery.blogspot.com/search/label/MCQ : General Surgery

greater in patients older than 60. Fibroplasia occurs at a slower rate in


older animals. Perhaps more than any other factor, wound infection is
associated with the risk of wound failure.

68. Wound contraction and ultimate contracture may be controlled by


which of the following drugs?
A. Colchicine.
B. D-Penicillamine.
C. Thiphenamil (Trocinate).
D. Glucocorticoids.
E. Ibuprofen (Motrin).
Answer: AC

DISCUSSION: Wound contraction is carried out by highly specialized cells


called myofibroblasts, which, as their name implies, have histologic
characteristics of fibroblasts and smooth muscle cells. The activity of
these cells, and therefore wound contraction, can be influenced by
topical application of smooth muscle inhibitors such as thiphenamil.
Inhibitors of microtubule formation in myofibroblasts, such as colchicine
and vinblastine, also inhibit wound contraction under experimental
conditions. Glucocorticoids and NSAIDs do not affect the wound
contraction process.

69. Which of the following is/are true of the actions of transforming


growth factor beta (TGF-b) during wound repair?
A. Increased matrix and proteoglycan synthesis.
B. Inhibition of proteases.
C. Stimulation of plasminogen inhibitor.
D. Chemotaxis for fibroblasts and macrophages.
E. Autoinduction of TGF-b.
Answer: ABDE

DISCUSSION: Through autocrine and paracrine mechanisms TGF-b


stimulates the deposition of collagen and other matrix components by
fibroblasts, inhibits proteases, blocks plasminogen inhibitor, enhances
angiogenesis, and is chemotactic for fibroblasts, monocytes, and
macrophages. TGF-b modulates the expression of cell-surface integrins
in a manner that enhances cell-matrix interaction and matrix assembly.
TGF-b also induces cell production by cells, thus amplifying its biologic
effects. The sustained production of TGF-b at the wound site leads to
tissue fibrosis.

70. In contrast to adult wound healing with scar formation, which of the
following are characteristic of scarless fetal skin repair?
A. Matrix rich in hyaluronic acid.
B. Increased inflammatory response.
C. Increased production of TGF-b.
D. No collagen deposition.
E. Minimal angiogenesis.

34 of 167 11-Aug-15 10:57 PM


MedCosmos Surgery: MCQ : General Surgery http://medcosmossurgery.blogspot.com/search/label/MCQ : General Surgery

Answer: AE

DISCUSSION: The ability of a fetus to heal without scar formation


depends on its gestational age at the time of injury and the size of the
wound defect. In general, linear incisions heal without scar until late in
gestation, whereas excisional wounds heal with scar at an earlier
gestational age. The profiles of fetal proteoglycans, collagens, and
growth factors are different from those in adult wounds. The less
differentiated state of fetal skin is probably an important characteristic
responsible for scarless repair. There is minimal inflammation and
angiogenesis in fetal wounds. Fetal wounds are characterized by high
levels of hyaluronic acid and its stimulator(s) with more rapid, highly
organized collagen deposition. The roles of peptide growth factors such
as TGF-b and basic fibroblast growth factor are less prominent in fetal
than in adult wound healing. An understanding of scarless tissue repair
has possible clinical applications in the modulation of adult fibrotic
diseases and abnormal scar-forming conditions.

71. Which of the following cell types are not crucial for healing a clean,
incisional wound?
A. Macrophage.
B. Platelet.
C. Fibroblast.
D. Polymorphonuclear leukocyte.
E. Myofibroblast.
Answer: DE

DISCUSSION: Experimental studies have shown that healing may progress


normally in the absence of polymorphonuclear leukocytes in an
uninfected wound. In contrast, depletion of monocytes and
macrophages causes a severe alteration in wound healing with poor
débridement, delayed fibroblast proliferation, and inadequate
angiogenesis. Platelets carry a cadre of biologically active substances
that are important for wound repair, including peptide growth factors
like platelet-derived growth factor (PDGF) and TGF-b. Fibroblasts are
the principal cell for matrix synthesis and deposition. Myofibroblasts are
important for wound contraction in open defects but have little if any
role in clean, incisional wounds.

72. Which of the following is/are not a substrate or cofactor for prolyl
hydroxylase?
A. Alpha-ketoglutarate.
B. Ascorbate.
C. Biotin.
D. Oxygen.
E. Copper.
Answer: CE

DISCUSSION: Prolyl hydroxylase is one of the rate-limiting enzymes in


collagen synthesis. Substrates and cofactors such as iron, alpha-

35 of 167 11-Aug-15 10:57 PM


MedCosmos Surgery: MCQ : General Surgery http://medcosmossurgery.blogspot.com/search/label/MCQ : General Surgery

ketoglutarate, ascorbate, and oxygen are important participants in this


process. If insufficient prolines are hydroxylated, then the alpha-
peptide collagen chains cannot assume a stable triple helix, the
collagen cannot be exported from the fibroblasts, and the incomplete,
unassociated alpha chains are broken down. Thus, ascorbate deficiency
(scurvy) and hypoxia have similar effects on collagen synthesis.

73. Which of the following is an adhesion glycoprotein?


A. Fibronectin.
B. Tenascin.
C. Laminin.
D. Hyaluronic acid.
E. Collagen type IV.
Answer: ABC

DISCUSSION: Cell adhesion glycoproteins such as fibronectin,


vitronectin, laminin, and tenascin provide a “railroad track” to
facilitate epithelial and mesenchymal cell migration over the wound
matrix. Hyaluronic acid is a glycosaminoglycan, and collagen type IV is a
protein that is a crucial component of basement membrane.

74. Which of the following is/are true concerning wound fibroblasts?


A. Fibroblasts synthesize and secrete collagen molecules.
B. Wound fibroblasts are derived from blood-borne precursor cells.
C. Fibroblasts migrate to a wound along fibrin strands, which are used
as a scaffold.
D. Large amounts of fibrin or blood clot can act as a physical barrier to
fibroblast penetration, which delays normal wound healing.
Answer: ACD

DISCUSSION: Fibroblasts appear in the wound on about the third day of


healing and begin to synthesize and secrete collagen molecules. Wound
fibroblasts arrive from cells surrounding the wound (e.g., the adventitia
of blood vessels), change their phenotype(s), and become mobile during
the process of replication. Fibroblasts migrate into a wound using the
provisional fibronectin and fibrin matrix as a scaffold. Fibroblasts do not
have fibrinolytic enzymes, and large amounts of fibrin and blood clot
prevent fibroblasts from entering the wound.

75. Which of the following is/are true?


A. Because of its thickness, the tensile strength of a healing wound on
the eyelid is much less than one on the thick skin of the back.
B. By 2 days, the experimental burst strength of skin is minimal since
collagen has been formed in the wound but has not yet cross-linked.
C. Wound strength reaches a plateau by 3 weeks.
D. Wounds rarely, if ever, regain the strength of intact tissues.
Answer: BD

DISCUSSION: Tensile strength measures load per cross-section area at

36 of 167 11-Aug-15 10:57 PM


MedCosmos Surgery: MCQ : General Surgery http://medcosmossurgery.blogspot.com/search/label/MCQ : General Surgery

rupture, whereas burst strength measures load required to break a


wound, regardless of dimension. Therefore, skin wounds have
comparable tensile strength, regardless of thickness. Collagen appears
in the wound by 3 to 4 days. Minimal wound strength on day 2 is due to
fibrin polymerization and adhesion of globular proteins. Wounds rapidly
gain strength for about 4 months and then continue to gain strength at a
slower rate for more than a year. Wounds do not regain the strength of
normal tissue.

76. Which of the following interfere with normal collagen formation or


cross-linking?
A. Beta-aminopropionitrile.
B. Iron chelators.
C. Vitamin C depletion.
D. Proline analogs (e.g., cis-hydroxyproline).
E. D-Penicillamine.
Answer: ABCDE

DISCUSSION: Intramolecular and intermolecular cross-links are crucial


for collagen structural stability. Formation of cross-links can be
inhibited by two pharmacologic agents: beta-aminopropionitrile inhibits
the enzyme lysyl oxidase, and D-penicillamine binds to collagen
substrate directly to prevent collagen cross-link formation. Iron is a
cofactor for prolyl hydroxylase, which is important for collagen
synthesis. In high enough concentration, proline analogs prevent
collagen formation with minimal effects on noncollagenase protein
synthesis.

77 Which of the following statement(s) is/are true concerning the cell


plasma membrane?
a. The plasma membrane is composed of amphipathic molecules
b. The hydrophobic core of the lipid bilayer of the cell membrane
contains specialized transport proteins which maintain the intracellular
ionic milieu different from the extracellular fluid
c. Plasma membrane proteins extend externally and bear phospholipid
moieties which contribute to the cell coat
d. The membrane proteins of nerve cells are highly voltage-dependent
Answer: a, b, d

The plasma membrane defines the boundary of the cell and serves to
contain and concentrate enzymes and other macromolecule
constituents. The plasma membrane is composed of amphipathic
molecules, mainly phospholipids and proteins that contain distinct
regions that are either insoluble in water (hydrophobic) or soluble in
water (hydrophilic). The plasma membrane forms a continuous barrier
between the aqueous extracellular and intracellular fluids. Transport
proteins in the membrane act as regulated channels or transporters to
maintain the intracellular ionic milieu that is clearly different from the
extracellular milieu. In some cells, membrane proteins are diversified
such as in nerve cells where the ion channels are highly voltage-

37 of 167 11-Aug-15 10:57 PM


MedCosmos Surgery: MCQ : General Surgery http://medcosmossurgery.blogspot.com/search/label/MCQ : General Surgery

dependent, providing the basis for information transmission in the form


of electrical impulses. Most plasma membrane proteins extend
externally and bear carbohydrate moieties primarily as oligosaccharide
chains that contribute to the cell coat or glycocalyx.

78 Which of the following statement(s) is/are true concerning water


movement across cell membranes?
a. Water moves only actively through cell membrane transport proteins
b. For most cells of the body, the transmembrane hydrostatic pressure is
0
c. Water distribution is determined entirely by solute distribution
d. Specialized cells such as the glomerulus of the kidney actively
transport water to maintain hydrostatic pressure
Answer: b, c

The energetics of water transport across cell membranes is simplified by


the fact that water moves only passively due to gradients of hydrostatic
pressure or water concentration. Hydrostatic pressure is an important
driving force only for certain specialized cells—the capillary
endothelium and the glomerulus of the kidney. For most cells of the
body, the transmembrane hydrostatic pressure is 0 and water moves
only in response to water concentration gradients. Because the
concentration of water is determined by the amount of dissolved solute,
the difference in water concentration is typically expressed as a
function of the difference in solute concentration or osmotic pressure
difference. Because there are no specialized, energy-converting
transport mechanisms for water, water is distributed at equilibrium.
Water distribution is determined entirely by solute to solute
distribution.

79 The transport of proteins out of the cell is termed exocytosis. Which


of the following statement(s) is/are true concerning this process?
a. Secretory vesicles fuse with the plasma membrane
b. The process can occur in either a constitutive or regulated process
c. A regulated secretion is triggered by a stimulus, most likely a
hormone or a neurotransmitter
d. A decrease in cytoplasmic calcium occurs as part of the secretion
process
Answer: a, b, c

Transport vesicles that bud off the Golgi network carry both material to
be secreted from the cell and protein destined to become components
of the plasma membrane. These vesicles can fuse with the plasma
membrane in a process termed exocytosis. Vesicular transport to the
cell surface can be divided into two components, constitutive and
regulated secretion. Regulated secretion occurs in cells secreting
digestive enzymes, hormones and other regulatory molecules, and
neurotransmitters. In regulated secretion, the material to be secreted is
sorted in a storage vesicle or granule; fusion with the plasma membrane
in exocytosis then takes place in response to external stimulation.
Regulated secretion is triggered in most cases by a hormone or

38 of 167 11-Aug-15 10:57 PM


MedCosmos Surgery: MCQ : General Surgery http://medcosmossurgery.blogspot.com/search/label/MCQ : General Surgery

neurotransmitter. The ensuing process is termed stimulus-secretion


coupling. In most cases the coupling involves an increase in cytoplasmic
concentration of Ca++, but may also involve generation of diacylglycerol
or production of cyclic AMP which activate kinases or phosphatases.

80 Which of the following statement(s) is/are true concerning the cell


function of phagocytosis?
a. Phagocytosis is a mechanistically distinct process of endocytosis
performed by special cells to take up larger particles such as bacteria or
erythrocytes
b. Lymphocytes are the primary blood cell involved with this process
c. The process involves a coating of the cytoplasmic surface known as
clathrin
d. Phagocytosis is performed only by white blood cells and tissue
macrophages
Answer: a

Phagocytosis is a specialized form of endocytosis by which large


particles are internalized by specialized cells primarily macrophages and
neutrophils. To be phagocytosed, particles must bind to the surface of
the phagocytic cell, usually as the result of specific antibody coating the
particle. The phagocytic cell then extends pseudopods which engulf the
particle. This event is followed by membrane fusion and a pinching off.
As opposed to endocytosis, this process does not involve the membrane
protein, clathrin, but rather actin. A physiologically relevant site of
phagocytosis is the thyroid gland, where thyroid follicular cells
phagocytose and digest thyroglobulin from the lumen of the thyroid
follicle, thereby releasing the thyroid hormones, thyroxine
triiodothyronine.

81 A striking feature of living cells is a marked difference between the


composition of the cytosol and the extracellular milieu. Which of the
following statement(s) concerning the mechanisms of maintenance of
these differences is/are true?
a. The cell membrane is able to maintain a 10,000 fold gradient
between the extracellular concentration of ionized calcium and the
intracellular concentration
b. The key to these differences is the fact that the plasma membrane is
normally impermeable to sodium, potassium and calcium
c. The selectivity of biologic membranes is highly consistent and seldom
changes
d. The selectivity of cell membranes relates only to ions and not organic
compounds

Answer: a

The survival of the cell requires that cytosolic composition be


maintained within narrow limits, despite the constant influx of nutrients
and the simultaneous outflow of waste. A familiar example of the
distribution of ions across the cell membrane is that of sodium and
potassium. Cells are typically rich in potassium and contain very little
sodium. Despite the fact that they are constantly bathed by fluid that is

39 of 167 11-Aug-15 10:57 PM


MedCosmos Surgery: MCQ : General Surgery http://medcosmossurgery.blogspot.com/search/label/MCQ : General Surgery

precisely the opposite composition. Even more impressive is the


distribution of ionized calcium. The extracellular concentration of this
ion is typically of the order of 10–3M, whereas that of cytosol is typically
10–7M, a 10,000-fold gradient. Such nonequilibrium ion distributions are
even more remarkable in light of the fact that the plasma membrane is,
to varying degrees, leaky to ions such as sodium, potassium and
calcium. The plasma membrane is leaky to a variety of substances, but
it exhibits an astonishing ability to discriminate or select one substance
over another. This selectivity relates to not only ions but also for organic
compounds such as glucose. Finally, the selectivity of biologic
membranes can be altered drastically as a result of regulatory or
signaling processes that occur within the cell.

82 Which of the following statement(s) is/are true concerning DNA?


a. DNA is contained only in the nucleus of the cell
b. DNA strands are encoded by the sequence of four bases—adenine,
guanine, cytosine and uridine
c. The basic unit of information of DNA is the intron, a sequence of
three bases
d. There are an infinite number of possible codons
Answer: a

The genetic blueprint of an organism is carried in the nucleus of every


cell, encoded by the sequence of four bases—adenine, guanine, cytosine
and thymine, which together make up two long chains bound together
by hydrogen bonds to form a DNA double helix. A gene is a segment of
DNA that is transcribed into a corresponding RNA molecule that either
codes for a protein or forms a structural RNA molecule. Genes are
commonly between 10,000 and 100,000 base pairs in length and include,
in addition to the coding sequence, flanking regions and intervening
sequences, termed introns. Introns are removed from the primary RNA
transcript by a process called splicing. The basic unit of information is
the codon, a sequence of three bases or triplet. The four nucleotide
bases arranged as triplets lead to 64 possible codons. Sixty-one of these
code for amino acids and three are termination signals called stop
codons.

83 Which of the following statement(s) is/are true concerning cell


membrane receptors?
a. The largest family of cell surface receptors are the G-protein-linked
receptors
b. Activities of the G-protein involve binding and hydrolysis of ATP
c. The G protein receptor generates an intracellular messenger
commonly through the use adenylate cyclase
d. Tyrosine kinase receptors are considered G-protein-linked receptors
Answer: a, c

All water-soluble regulatory molecules bind to the cell surface receptor


proteins. Binding of the appropriate ligand evokes an intracellular signal
which usually regulates enzyme activity, membrane transport, or in
some cases gene expression. Most cell surface receptors belong to one

40 of 167 11-Aug-15 10:57 PM


MedCosmos Surgery: MCQ : General Surgery http://medcosmossurgery.blogspot.com/search/label/MCQ : General Surgery

of three functional classes—these are ion channel receptors, catalytic


receptors, and G-protein-linked receptors. Ion channel receptors are
multisubunit assemblies which, with each subunit, have a multiple
membrane spanning segment. Together these subunits form an
ion-selected pore that can be gated by a change in transmembrane
electrical potential or binding of a ligand to one of the subunits.
Catalytic receptors are membrane proteins that possess enzymatic
activity. The best understood receptors of this class are the tyrosine
kinases. The largest family of cell surface receptors are the G-protein-
linked receptors. G-proteins are a family of proteins that bind and
hydrolyze GTP. The final component of single transduction by G-protein-
linked cell surface receptors is the effector that generates the
intracellular messenger. The two best understood effectors are
adenylate cyclase, which converts ATP to cAMP, and the
polyphosphoinositide-specific phospholipase C.

84 Which of the following statement(s) is/are true concerning cellular


ion channels?
a. Ion channels are transmembrane proteins that form pores that can
conduct ions across the plasma membrane
b. Ion channels are formed by membrane-spanning peptides that are
arranged so that polar moieties line a central core
c. Ion channel proteins undergo conformational changes between open
states and closed states
d. Ion channels can be blocked
Answer: a, b, c, d

Ion channels are transmembrane proteins that form pores that can
conduct ions across the plasma membrane. Ion channels are formed by
membrane-spanning peptides that are arranged so that polar moieties
line a central pore. These polar groups take the place of the water of
hydration, which stabilizes an ion in an aqueous solution creating, in
essence, a water-like environment into which the ion can partition and
move in the presence of the appropriate driving force. Ion channels are
permissive transport elements. Ions flow through a channel only through
the presence of an appropriate driving force. Ion channels do not
conduct all the time, rather the channel protein undergoes
conformational changes between a conducting (open) state and
nonconducting (closed) state. These conformational changes are
collectively referred to as gating. The conduction process can also be
blocked by ions or organic compounds that enter the channel, bind
there, and occlude the pore.

85 Which of the following statement(s) is/are true concerning carrier


proteins?
a. Carrier proteins are distinguished by three types of mechanisms:
carrier-type, channel-type, and conduction-type
b. Conformational changes in the membrane protein occur between the
conducting and the nonconducting states
c. A channel-type carrier protein has two states—closed and open
d. Carrier-type transport proteins are equally accessible from either

41 of 167 11-Aug-15 10:57 PM


MedCosmos Surgery: MCQ : General Surgery http://medcosmossurgery.blogspot.com/search/label/MCQ : General Surgery

side of the membrane


Answer: b, c

Most transport proteins appear to function as carriers, rather than


channels. Important distinctions can be made between types of carrier
proteins on the basis of transport kinetics. Two primary types can be
distinctly identified based on carrier-type and channel-type
mechanisms. The most important difference between the channel
mechanism and the carrier mechanism is the role in the transport event
played by conformational changes in the membrane protein. The
channel is depicted as having two states, closed and open, so that it
operates like a switch. In contrast, carrier transport is envisioned as
requiring a cycle of conformational changes. The transport of one
molecule of substrate requires one complete cycle of the protein. In a
channel mechanism, binding sites within the open pore are equally
accessible from either side of the membrane, whereas in a carrier
mechanism, the binding site is available only one side of the membrane
at any instant.

86 Which of the following statement(s) is/are true concerning


translation of the mRNA message to protein synthesis?
a. An adaptor molecule, tRNA, recognizes specific nucleic acid bases
and unites them with specific amino acids
b. Covalent attachment of tRNA to amino acids is energy dependent
c. The formation of a peptide bond between the growing peptide chain
and the free amino acid occurs in the free cytoplasm
d. Complete protein synthesis takes hours
Answer: a, b

The synthesis of protein involves conversion from a four-letter


nucleotide language to one of 20 chemically distinct amino acids. This
process is referred to as translation. There is no mechanism for direct
chemical recognition between specific nucleic acid bases and specific
amino acids. Instead, an adaptor molecule, tRNA, is used. Each tRNA
carries only one amino acid and must be recognized by a distinct
enzyme which catalyzes the covalent attachment of the carboxyl end of
the amino acid to the end of the tRNA in a process using ATP. Protein
synthesis occurs by the formation of a peptide bond between the
carboxyl terminal of the growing peptide chain and the free amino acid
of deactivated amino acid tRNA. This event does not occur in free
solution, but within ribosomes. Ribosomes are protein synthesizing
machines that bring all of the necessary components together in the
correct sequence and spacial orientation. Protein synthesis consumes a
great deal of energy because four high-energy phosphate bonds must be
split to make each peptide bond. Complete synthesis of a single protein
takes 30 seconds to a few minutes, but multiple ribosomes can initiate
translation and be moving down the mRNA molecules simultaneously,
thus increasing the rate of protein synthesis.

87 Cell regulation can be thought of as the effector side of cell


communication. Most commonly cell regulation occurs by means of
extracellular chemical messengers. Which of the following statement(s)

42 of 167 11-Aug-15 10:57 PM


MedCosmos Surgery: MCQ : General Surgery http://medcosmossurgery.blogspot.com/search/label/MCQ : General Surgery

is/are true concerning these messengers?


a. Paracrine regulation involves a messenger which is produced and acts
systemically
b. The extracellular signal or stimulus is received by a receptor on or in
the target cell
c. Neurocrine regulation depends on a physical connection between the
neuron and the target cell
d. Most hormones, local mediators, and neurotransmitters readily cross
the cell plasma membrane
Answer: b, c

Depending on how the extracellular messenger arrives, cell regulation


can be classified as paracrine, endocrine, or neurocrine. In paracrine
regulation, a chemical messenger or mediator is produced and acts
locally. In endocrine regulation, the extracellular messengers
(hormones) are released into the blood and act on target cells anywhere
on the body that has appropriate receptors. In neurocrine regulation,
neurons secrete transmitters into highly localized regions, the synaptic
cleft, so that the regulation depends on a physical connection between
the neuron and the target cell as well as the presence of a specific
receptor. In almost all cases of cell regulation, the extracellular signal
or stimulus is restricted to being an informational molecule. This
information is received by receptor on or in the target cell, which
generally has an affinity for the signal molecule. Most hormones, local
mediators, and neurotransmitters are water-soluble and cannot readily
cross the plasma membrane.

88 Proteins that are destined to be secreted from the cells must pass
through a series of organelles. These organelles include:
a. Endoplasmic reticulum
b. Golgi apparatus
c. Mitochondria
d. Lysosomes
Answer: a, b, d

Proteins targeted for the secretory pathway most commonly begin with
translocation from the cytoplasm across the lipid bilayer into the lumen
of the endoplasmic reticulum. It must then pass through a number of
compartments including the Golgi apparatus where they are further
processed and sorted and end up in a secretory vesicle or lysosome.

89 The best understood intracellular messenger is cyclic AMP (cAMP).


Which of the following statement(s) concerning this intracellular
messenger is/are correct?
a. Intracellular cyclic AMP is constantly degraded by a specific enzyme,
cAMP phosphodiesterase
b. Most of the actions of cAMP are mediated by activation of protein
kinase A
c. Intracellular levels of cAMP are relatively stable and change solely in
response to activation of adenylate cyclase
d. cAMP is the only cyclic nucleotide active as an intracellular
messenger

43 of 167 11-Aug-15 10:57 PM


MedCosmos Surgery: MCQ : General Surgery http://medcosmossurgery.blogspot.com/search/label/MCQ : General Surgery

Answer: a, b

The prototypic intracellular messenger is cAMP. To function as a


mediator, the concentration of cAMP must change rapidly. In resting
cells, cAMP is continuously being degraded by a specific enzyme, cAMP
phosphodiesterase. cAMP levels can increase 10-fold or more within
seconds of receptor binding through activation of adenylate cyclase.
cAMP acts as an allosteric regulator, and most, if not all, of its actions
are mediated by activation of cAMP-dependent protein kinase A. cAMP is
not the only cyclic nucleotide active as an intracellular messenger. Most
animal cells also produce cGMP. Intracellular calcium ions also serve as
second messengers in a large number of cells.

90 The activities of the cytoskeleton is dependent on which of the


following types of filaments?
a. Microtubules
b. Intermediate filaments
c. Actin filaments
d. None of the above
Answer: a, b, c

The cytoskeleton is a collection of filamentous protein structures that


allow cells to assume and maintain a variety of shapes, to produce
directed movement of organelles within the cell, and to affect
movement of the entire cell relative to other cells. These multiple
activities depend upon three main types of filaments: actin filaments,
intermediate filaments, and microtubules.

91 Intracellular organelles involved with protein synthesis include:


a. Mitochondria
b. Endoplasmic reticulum
c. Golgi complex
d. Lysosomes
Answer: b, c

Mitochondria are the major source of energy production in eukaryotic


cells. The endoplasmic reticulum is the network of interconnected
membranes forming closed vesicles, tubules, and saccules. The
endoplasmic reticulum has a number of functions and is primarily
involved in the synthesis of proteins and lipids. Adjacent to the rough
endoplasmic reticulum and functionally involved in the sorting and
package of secreted protein is the Golgi complex. Lysosomes are
membrane-limited organelles containing acid hydrolytic enzymes that
degrade polymers such as proteins, carbohydrates, and nucleic acids.

92 An important step in protein synthesis is transcription. Which of the


following statement(s) is/are true concerning this process?
a. The first step in gene transcription involves separating the double
helix of DNA by an enzyme known as DNA polymerase
b. The initial product of DNA transcription is called heterogeneous
nuclear RNA which codes directly for proteins

44 of 167 11-Aug-15 10:57 PM


MedCosmos Surgery: MCQ : General Surgery http://medcosmossurgery.blogspot.com/search/label/MCQ : General Surgery

c. After processing is complete, the mRNA is exported from the nucleus


to the cytoplasm
d. Only one protein can be produced from an initial mRNA strand
Answer: c

Transcription of a gene begins at an initiation site associated with a


specific DNA sequence, termed a promoter region. After binding to DNA,
the RNA polymerase opens up a short region of the double helix to
expose the nucleotides. Once the two strands of DNA are separated, the
strand containing the promoter acts as a template to which
ribonucleoside triphosphates base pair by hydrogen bonds. The initial
products of transcription are known as heterogeneous nuclear RNA
because of their large size variation. These primary transcripts are then
processed to form mRNA. RNA splicing accounts for mature RNA being
much shorter than nuclear RNA. Moreover, alternative splicing can lead
to the production of different mRNA molecules and in some cases
different proteins from the same gene. mRNA is exported from the
nucleus only after processing is complete.

93 There are two properties of the cell necessary to maintain


nonequilibrium cellular composition; the first is selectivity and the
second is energy conversion. Which of the following statement(s) is/are
true concerning energy converting transport?
a. The site of energy conversion and transport in the plasma membrane
involves the phospholipid component
b. The Na+-K++-ATPase derives energy from hydrolysis of extracellular
ATP
c. In some systems, energy inherent in the transmembrane ion gradient
can be used to drive transport of a second species
d. Examples of species transported via secondary active transport
include hydrogen ions, calcium, amino acids and glucose
Answer: c, d

The selectivity of the plasma membrane, although impressive, cannot


account for the nonequilibrium composition of living cells. A cell can be
maintained in a nonequilibrium state only by continual expenditure of
energy. The maintenance of a steady-state, nonequilibrium cellular
composition is possible because the plasma membrane is the site of
energy converters, membrane proteins that function as biologic
transport machines using energy derived from metabolic processes to
perform transport work. The archetype for the biologic transport
machine is the Na+-K+-ATPase, a membrane protein that hydrolyses
cytosolic ATP and couples the resulting free energy to transport of Na+
and K+. A second equally important type of energy-converting
transporter is one in which the energy inherent in a transmembrane ion
gradient, usually that of Na+ can be used to drive the transport of a
second species such as protons, calcium, amino acids, or glucose.

94 Which of the following statement(s) is/are correct concerning cell


junctions?
a. The major occluding junction is the tight junction or zonula
occludens

45 of 167 11-Aug-15 10:57 PM


MedCosmos Surgery: MCQ : General Surgery http://medcosmossurgery.blogspot.com/search/label/MCQ : General Surgery

b. Tight junctions are usually located near the basal pole of the cell
c. Desmosomes are button-like points of attachment which serve to
weld together adjacent cells
d. Gap junctions are a type of cell junction specialized for cell
communication
Answer: a, c, d

Cell junctions are classified as occluding, anchoring, and


communicating. The major occluding junction is the tight junction or
zonula occludens which connects cells in epithelia and thereby allows
epithelia to serve as selective permeability barriers. Tight junctions are
normally located near the apical pool of the cell and form a belt that
completely encircles the cell. Anchoring junctions connect the
cytoskeleton of the cell to the extracellular matrix or neighboring cells.
Morphologically these are adherens junctions or desmosomes.
Desmosomes are button-like points of attachment with a prominent
intracellular plaque that serves to weld together adjacent cells by
serving as anchoring sites for intermediate filaments within the cell.
The third functional type of cell junction is a gap junction which is
specialized for communication. This junction mediates both electrical
and chemical coupling.

95 Examples of ion channel blockers include:


a. Tetrodotoxin
b. Amiloride
c. Xylocaine
d. None of the above
Answer: a, b, c

Channel blockade is an important mechanism of action for toxins and


some therapeutic agents. The deadly toxin of the puffer fish,
tetrodotoxin, acts by blocking the Na+ channels that are responsible for
the conduction of nerve impulse. The diuretic, amiloride, acts by
blocking the Na+ channels that inhabit the apical membrane of the
epithelial cells of the distal nephron. Local anesthetics such xylocaine
also act by blocking ion channels.

96 Most hormone receptors are localized on the cell membrane and


transduce hormone binding into altered levels of intracellular
messengers. A limited number of intracellular receptors do exist. Which
of the following statement(s) is/are true concerning intracellular
receptors?
a. The messengers or hormones must by lipophilic
b. These intracellular receptors generally regulate protein synthesis
c. The intracellular receptors are located entirely in the nucleus of the
cell
d. A heat-shock protein serves as an inhibitor protein blocking the
DNA-binding domain of the steroid receptor
Answer: a, d

Although most hormone and other messenger receptors are


extracellular, intracellular receptors have been identified. The hormone

46 of 167 11-Aug-15 10:57 PM


MedCosmos Surgery: MCQ : General Surgery http://medcosmossurgery.blogspot.com/search/label/MCQ : General Surgery

messengers involved for these receptors are primarily steroid and


thyroid hormones and are lipophilic. By virtue of their hydrophobic
nature, they are able to readily penetrate the lipid portion of the cell
membrane. Receptors for these hormones exist intracellularly in the
cytoplasm or in the nucleus and generally act as regulators of gene
expression. These hydrophobic signaling molecules exist in plasma
bound to protein, so that the concentration of this class of regulators
does not fluctuate rapidly in plasma and their actions are generally
slower in onset and more prolonged than those of water-soluble class.
Some types of steroid receptors, particularly for glucocorticoids, are
located in the cytosol in the inactive state. Once the ligand binds, the
receptor undergoes a conformational change, termed activation. This
allows cytoplasmic receptors to move into the nucleus and bind to DNA.
Receptors already in the nucleus increase their affinity for DNA. In the
case of glucocorticoid receptors and probably others of this class, the
inactive receptor is associated with another protein, the heat-shock
protein. They block the DNA-binding domain of the receptor. Activation
involves the dissociation of the inhibitor protein.

97 Altering the amino acid profile in total parenteral nutrition solutions


can be of benefit in certain conditions. Which of the following
conditions are associated with a benefit by supplementation with the
amino acid type listed?
a. Acute renal failure and essential amino acids
b. Hepatic failure and aromatic amino acids
c. Short gut syndrome and glutamine
d. Chronic renal failure and essential amino acids
Answer: a, c

In a number of conditions, altering the amino acid profile of the total


parenteral nutrition solution can be of benefit. TPN with amino acids of
high biologic value may decrease the mortality in patients with acute
renal failure. These solutions, containing high quality amino acids, can
improve nitrogen balance and diminish urea nitrogen. Provision of
essential amino acids only allows the body to maximally utilize nitrogen
for the synthesis of non-essential amino acids and thereby helps prevent
rapid rises in blood urea nitrogen. There appears to be no advantages to
using essential amino acids if the patient is already being dialyzed every
other day and therefore a balanced standard amino acid solution is
recommended. Because of liver damage and portasystemic shunting,
patients with hepatic failure develop derangements in circulating levels
of amino acids. The plasma aromatic/branch chain amino acid ratio is
increased favoring the transport of aromatic amino acids across the
blood brain barrier. These amino acids are precursors of false
transmitters which contribute to lethargy and encephalopathy.
Treatment of individuals with liver failure with solutions enriched in
branch chain amino acids and deficient in aromatic amino acids results
in improved tolerance to administration of protein and clinical
improvement in encephalopathic states. Glutamine-enriched TPN
partially attenuates villous atrophy and may be useful in treatment of
short gut syndrome.

47 of 167 11-Aug-15 10:57 PM


MedCosmos Surgery: MCQ : General Surgery http://medcosmossurgery.blogspot.com/search/label/MCQ : General Surgery

98 Under certain circumstances, the gut may become a source of sepsis


and serve as the motor of systemic inflammatory response syndrome.
Microbial translocation is the process by which microorganisms migrate
across the mucosal barrier to invade the host. Which of the following
mechanisms can promote bacterial translocation?
a. An increased number of gut bacteria
b. Altered intestinal mucosal permeability
c. Decreased host defense mechanisms
d. Lack of enteral feeding

Answer: a, b, c, d

99 Translocation is promoted in three general ways: 1) altered


permeability of the intestinal mucosa as caused by shock, sepsis, distant
injury, or cell toxins; 2) decreased host defense (secondary to
glucocorticoid administration, immunosuppression, or protein depletion;
and 3) an increased number of bacteria within the intestine. Because
many factors that facilitate bacteria translocation occur simultaneously
in surgical patients, these effects may be either additive or cumulative.
In addition, many patients in Surgical Intensive Care Units do not
generally receive enteral feedings and therefore current parenteral
therapy results in gut atrophy which further promotes translocation.
Which of the following statement(s) is/are true concerning nutritional
support of the injured patient?
a. The goal of nutritional support is maintenance of body cell mass and
limitation of weight loss to less than 25% of preinjury weight
b. Under-nutrition may compromise the patient’s available defense
mechanisms
c. Nutritional support is an immediate priority for the trauma patient
d. Fifty percent of non-nitrogen caloric requirements should be provided
in the form of fat
Answer: b

Metabolic response to injury results in increased energy expenditure. If


energy intake is less than expenditure, oxidation of body fat stores and
erosion of lean body mass will occur with resultant loss of weight. When
weight loss exceeds 10–15% of body weight, the complications of
malnutrition interact with disease processes, with increased morbidity
and mortality rates. The goal of nutritional support is maintenance of
body cell mass and limitation of weight loss to less than 10% preinjury.
The major impact of nutritional support in the trauma patient is to aid
host defense. Under-nutrition may compromise the available host
defense mechanism and may thus increase the likelihood of invasive
sepsis, multiple organ system failure, and death. Resuscitation,
oxygenation and arrest of hemorrhage are immediate priorities for
survival. Nutritional support is an essential part of the metabolic care of
the critically ill patient and should be instituted after resuscitation
before significant weight loss occurs. The nutritional requirements of a
trauma patient can be determined by determining basal metabolic rate
with appropriate increases based on extent of injury and hospital
activity. After initial determination of nitrogen requirements, caloric
requirements should be distributed at a ratio of 70% as glucose and 30%

48 of 167 11-Aug-15 10:57 PM


MedCosmos Surgery: MCQ : General Surgery http://medcosmossurgery.blogspot.com/search/label/MCQ : General Surgery

as fat.

100 Which of the following statement(s) is/are true concerning body


fuel reserves?
a. The largest fuel reserve in the body is skeletal muscle
b. Fat provides about 9 calories/gram
c. Free glucose and glycogen stores are a trivial fuel reserve
d. Body protein is a valuable storage form of energy
Answer: b, c

The body contains fuel reserves which it can mobilize and utilize during
times of starvation or stress. By far the greatest energy component is
fat, which is calorically dense since it provides about 9 calories/gram.
Body protein comprises the next largest mass of utilizable energy, but
amino acids yield only about 4 kcal/gram. Unlike fat reserves, body
protein is not a storage form of energy but rather serves as a structural
functional component of the body; loss of body protein, if severe, is
associated with functional consequences. Glycogen stored in muscle and
liver and free glucose have a trivial caloric value of less than 1000 kcal
for a 70 kg male.
101 Which of the following statement(s) is/are true concerning the
indications and administration of nutritional support to cancer patients?
a. Preoperative nutritional support should be provided to all patients
with cancer
b. To be effective, preoperative nutrition must be given for at least two
weeks preoperatively
c. Parenteral nutrition is the preferred route of feeding for all cancer
patients
d. Standard total parenteral nutrition solutions maintain integrity of the
small bowel
e. None of the above
Answer: e

The role of nutritional support in the cancer patient remains an


important component of overall therapy. Preoperative nutritional
support should be given only to those patients who do not require an
emergency operation and who have severe weight loss (> 15% of
pre-illness body weight) and a serum albumen < 2.9 mg%. Preoperative
nutrition (enteral or parenteral) should not be given for longer than 7 to
10 days. Enteral nutrition is always the preferred route of feeding
cancer patients if the GI tract is functional. There are several benefits
of using the bowel lumen for nutrient delivery. The trophic effects of
enteral feeding on small bowel mucosa have been well described. The
integrity of the mucosal lining is maintained and it may provide an
effective barrier to intraluminal enteric organisms which might
otherwise translocate into the systemic circulation. Atrophic changes
may be seen in the intestinal epithelium after several days of bowel
rest; this atrophy is not reversed by currently available total parenteral
nutrition solutions.

102 Which of the following hormones can be expected to be released as


part of the stress response?

49 of 167 11-Aug-15 10:57 PM


MedCosmos Surgery: MCQ : General Surgery http://medcosmossurgery.blogspot.com/search/label/MCQ : General Surgery

a. Antidiuretic hormone (ADH)


b. Aldosterone
c. Insulin
d. Epinephrine
nswer: a, b, d

Several important responses occur in response to stress. The body


immediately attempts to compensate for a reduction in circulating
blood volume in order to maintain adequate organ perfusion. Afferent
nerve signals are also initiated which stimulate the release of both
antidiuretic hormone (ADH) and aldosterone. The pain and fear
associated with the stress response lead to excessive production to
catecholamines which also increase metabolic rate, stimulate lipolysis,
hepatic glycolysis, and gluconeogenesis. Glucagon, which has a potent
glycogenolytic and gluconeogenic effect in the liver, is also released.
This hormone has the exact opposite effect of insulin, which promotes
glucose storage and uptake by the cells.

103 Cytokines which play an important role in the metabolic response to


injury include:
a. Tumor necrosis factor—a (TNF)
b. Interleukin-1 (IL-1)
c. Interleukin-6 (IL-6)
d. Interferon-g
Answer: a, b, c, d

TNF or cachetin is considered the primary mediator of the systemic


effects of endotoxin, producing anorexia, fever, tachypnea, and
tachycardia at low doses and hypotension, organ failure, and death at
higher doses. TNF is produced primarily by macrophages, but
lymphocytes, Kupffer cells, and a number of other cell types have been
identified as sources of TNF. IL-1, like TNF, has a variety of
pro-inflammatory activities. IL-6 is now recognized as a primary
mediator of altered hepatic protein synthesis known as acute-phase
protein synthetic response. Glucocorticoid hormones augment the
cytokine effects on acute phase protein synthesis. Interferons are a
family of proteins which are readily identified for their ability to inhibit
viral replication in infected sells. IFN-g has the ability to upregulate the
number of TNF receptors on various cell types.

104 A 16-year-old boy suffers a mid-gut volvulus with massive loss of


small intestine. Which of the following statement(s) is/are true
concerning his nutritional requirements and management?
a. If at least 18 inches of residual small intestine survives, the patient
may tolerate some form of enteral nutrition
b. A nutritional regimen consisting of supplemental glutamine, growth
hormone, and a modified high carbohydrate, low fat diet may be
beneficial in this patient
c. The regimen described above may decrease the cost of care
d. TPN needs will increase after discontinuation of growth hormone
Answer: a, b, c

50 of 167 11-Aug-15 10:57 PM


MedCosmos Surgery: MCQ : General Surgery http://medcosmossurgery.blogspot.com/search/label/MCQ : General Surgery

Prior to the availability of TPN, most patients developing short bowel


syndrome from either surgery or catastrophic event died. In selected
patients, however, with residual small intestine (at least 18 inches),
post-resectional hyperplasia may develop with time such that they can
tolerate enteral feeds. Recent studies have demonstrated the
requirement for TPN could be decreased or even eliminated in patients
with short-gut syndrome by providing a nutritional regimen consisting of
supplemental glutamine, growth hormone, and a modified high
carbohydrate, low fat diet. There was a marked improvement in
absorption of nutrients with this combination of therapy and a decrease
in stool output. In addition, TPN requirements were reduced by 50% as
were costs associated with the care of these individuals. Discontinuation
of the growth hormone did not increase TPN needs in patients once they
had undergone successful gut rehabilitation.

105 A number of changes in trace mineral metabolism are noted during


sepsis. Which of the following change(s) may be observed in a septic or
trauma patient?
a. Plasma iron levels are noted to decrease
b. Plasma copper levels are noted to decrease
c. Plasma serum zinc levels may decrease
d. Administration of iron is appropriate
Answer: a, c

Changes in the balance of magnesium, inorganic phosphate, zinc, and


potassium generally follow alterations in nitrogen balance. Although the
iron-binding capacity of transferrin is usually unchanged in early
infection, iron disappears from the plasma, especially during severe
pyogenic infection; similar alterations are observed in serum zinc levels.
The administration of iron to the infected host, especially early into the
disease, is contraindicated, however, because increased serum iron
concentrations may impair resistance. Unlike iron and zinc, copper
levels generally rise, and the increased plasma concentrations can be
ascribed almost entirely to the levels of the ceruloplasmin produced by
the liver.

106 A 17-year-old patient involved in an automobile accident is


paralyzed with multiple peripheral extremity injuries. Nutritional
support is instituted with a transnasal feeding catheter. Which of the
following statement(s) is/are true concerning the patient’s
management?
a. Feeding into the stomach results in stimulation of the
biliary/pancreatic axis which is probably trophic for small bowel
b. Gastric secretions will dilute the feedings increasing the risk of
diarrhea
c. The major risk in this patient is tracheobronchial aspiration
d. Placement of the feeding catheter through the pylorus into the first
portion of the duodenum reduces the risk of regurgitation and aspiration
Answer: a, c, d

The use of transnasal feeding catheters for intragastric feeding or for


duodenal intubation are popular adjuncts for providing nutritional

51 of 167 11-Aug-15 10:57 PM


MedCosmos Surgery: MCQ : General Surgery http://medcosmossurgery.blogspot.com/search/label/MCQ : General Surgery

support by the enteral route. The stomach is easily accessed by passage


of a soft flexible feeding tube. Intragastric feeding provides several
advantages for the patient. The stomach has the capacity and reservoir
for bolus feedings. Feeding into the stomach results in stimulation of
the biliary/pancreatic axis which is probably trophic for the small
bowel. Gastric secretions will have a dilutional effect on the osmolarity
of the feedings, reducing the risk of diarrhea. The major risk of
intragastric feeding is the regurgitation of gastric contents resulting in
aspiration into the tracheobronchial tree. This risk is highest in patients
who have an altered sensorium or who are paralyzed. The placement of
the feeding tube through the pylorus into the fourth portion of the
duodenum reduces the risk of regurgitation and aspiration of feeding
formulas.

107 Although TPN has major beneficial effects to the patient and
specific organ systems, TPN has a downside which is related to intestinal
disuse. Which of the following statement(s) is/are true concerning the
effects of TPN on the GI tract?
a. Patients receiving TPN have an accentuated systemic response to
endotoxin challenge compared to enterally fed volunteers
b. TPN can result in disruption of intestinal microflora
c. In experimental models, bacterial translocation from the gut is
increased
d. Effects of TPN on the gut may lead to multiple organ failure
nswer: a, b, c, d

A number of studies have examined the effects of TPN on intestinal


function and immunity. Although most of these studies have been done
in animal models, TPN has consistently been shown to have some
detrimental effects. In rats, TPN results in significant disruption of the
intestinal microflora and bacterial translocation of the gut to the
mesenteric lymph nodes. In addition, when stresses such as a burn
injury, chemotherapy, or radiation are introduced into these models,
animals on TPN have a much higher mortality. The body of literature
suggests that TPN under certain circumstances may predispose patients
to an increase in gut-derived infectious complications. In a study in
human volunteers, individuals receiving TPN had an accentuated
systemic response to endotoxin challenge compared to enterally fed
volunteers. This study is consistent with impairment of gut barrier
function during parenteral feedings which may promote the release of
bacteria and/or cytokines leading to pronounced systemic responses and
possibly multiple organ failure.

108 Total body mass is composed of an aqueous component and a


nonaqueous component. The nonaqueous component is made up of
which of the following?

a. Liver
b. Tendons
c. Skeletal muscle
d. Extracellular fluid
e. Adipose tissue

52 of 167 11-Aug-15 10:57 PM


MedCosmos Surgery: MCQ : General Surgery http://medcosmossurgery.blogspot.com/search/label/MCQ : General Surgery

Answer: b, e

The nonaqueous portion of total body mass is made up of bones,


tendons, and mineral mass as well as adipose tissue. The aqueous
component contains the body cell mass which is made up of skeletal
muscle, intraabdominal and intrathoracic organs, skin, and circulating
blood cells. Also contributing to the aqueous portion is the interstitial
fluid and intravascular volume.

109 Fatty acids are a major energy source for the body. Which of the
following statement(s) is/are true concerning the use of fatty acids as
an energy source?
a. Fatty acids are stored in adipocytes as triglycerides
b. Hormone-sensitive lipase is present only in adipose tissue
c. Fatty acids are released into the circulation traveling freely in plasma
d. Approximately 25% of total nonprotein caloric needs supplied via
total parenteral nutrition should be in the form of fat
Answer: a, b, d

In most tissues, fatty acids are readily oxidized for energy. They are
especially important energy sources for the heart, liver and skeletal
muscle. In adipose tissue, fatty acids may be re-esterified with glycerol
and stored as triglycerides in adipocytes. Stored fat is mobilized during
starvation and stress. Hormone-sensitive lipase, present only in adipose
tissue, catalyzes the breakdown of stored triglycerides into glycerol and
fatty acids. The fatty acids that are produced are released in the
circulation. The major lipids in plasma do not circulate in a free form,
thus free fatty acids must be bound to albumin. During stress, the
activity of hormone-sensitive lipase is increased which leads to
mobilization of fat stores. However, fat remains an important fuel
source for critically ill patients and as a rule the amount of fat
administered to patients receiving total parenteral nutrition should
comprise about 5–30% of total nonprotein caloric needs.

110 Which of the following metabolic effects may be observed in


patients with sepsis?
a. Increased gluconeogenesis
b. Accelerated proteolysis
c. Increased lipolysis
d. Impaired gut metabolism of glutamine
Answer: a, b, c, d

A number of metabolic responses to sepsis have been defined. Glucose


production is increased in infected patients which appears to be
additive to the augmented gluconeogenesis that occurs following injury.
Accelerated proteolysis, increased nitrogen excretion and prolonged
negative nitrogen balance also occur following infection with a response
pattern similar to that described with injury. Severe infection is often
associated with a hypercatabolic state that initiates marked changes in
interorgan glutamine metabolism. This process results in accelerated
muscle proteolysis and net skeletal muscle glutamine release. The bulk
of glutamine is taken up by the liver at the expense of the gut. It

53 of 167 11-Aug-15 10:57 PM


MedCosmos Surgery: MCQ : General Surgery http://medcosmossurgery.blogspot.com/search/label/MCQ : General Surgery

appears that sepsis can impair gut metabolism of glutamine. Fat is a


major fuel oxidized in infected patients, and increased metabolism of
lipids from peripheral fat stores is especially prominent during a period
of inadequate nutritional support.

111 Which of the following statement(s) is/are true concerning


protein/amino acid metabolism in man?
a. The major source of amino acids is breakdown of circulating proteins
b. The recommended daily allowance for protein may triple in critically
ill patients
c. Urinary nitrogen losses will approach 0 in the face of protein
starvation
d. Negative nitrogen balance refers to a decrease in nitrogen taken into
the body versus the amount of nitrogen lost
Answer: b, d

About 15% of the total body weight is made up of proteins, about half of
which are intracellular and half extracellular. In man and other animals,
dietary protein is the source of most amino acids. Intestinal absorption
is the only physiological pathway by which the body obtains exogenous
amino acids. Digestion of ingested protein provides free amino acids
that are absorbed by the small intestine and transported to the liver
where they can be incorporated into new proteins or other biosynthetic
products. Excess amino acids are degraded and their carbon skeleton is
oxidized to produce energy or it is incorporated into glycogen or into
free fatty acids. In addition to the metabolism of dietary amino acids,
the existing proteins in the cell are continuously recycled, such that
total protein turnover in the body is about 300 g/day. Vertebrates
cannot reutilize nitrogen with 100% efficiency; therefore, obligatory
nitrogen losses occur, mainly in the urine. Urinary nitrogen losses will
diminish when individuals are fed a protein-free diet, but will never
become 0 because of the body’s inability to completely reutilize
nitrogen. In stressed patients, this ability to adapt to starvation is
compromised such that proteolysis of body proteins continues at a
substantial rate. This increases the amount of obligatory nitrogen losses
which are accentuated by the catabolic disease states. This results in a
negative nitrogen balance in which the amount of nitrogen taken in by
the patient is exceeded by the amount of nitrogen lost in the urine,
stool, skin, wounds, and fistula drainage.

112 Which of the following statement(s) concerning intravenous


nutritional support is/are true?
a. Concentrations of glucose no higher than 5% should be used to avoid
peripheral vein sclerosis
b. A major disadvantage of the peripheral technique is limited caloric
delivery
c. If total parenteral nutrition is required, access to the superior vena
cava via the external jugular vein is the most suitable site
d. Venous thrombosis is an uncommon complication for long-term
central vein catheterization
Answer: b

54 of 167 11-Aug-15 10:57 PM


MedCosmos Surgery: MCQ : General Surgery http://medcosmossurgery.blogspot.com/search/label/MCQ : General Surgery

Although peripheral access can be used for intravenous nutrition, the


major disadvantage of this technique is limited caloric delivery to meet
catabolic demands within tolerated fluid limits. Infusion of glucose (up
to 10%), amino acid solutions, and fat emulsions can be administered
peripherally but these solutions must be nearly isotonic to avoid
peripheral vein sclerosis. The preferred method of access for total
parenteral nutrition is into the superior vena cava by cutaneous
cannulation of the subclavian vein. Alternative sites include the internal
and external jugular vein but the catheter exiting from the neck region
makes it more difficult to secure and maintain a sterile dressing.
Complications from long-term central venous catheterization include
venous thrombosis and venous catheter-related infection. Thrombosis of
central vessels is a complication which is often overlooked. The clinical
suspicion of subclavian vein thrombosis is only about 3%, whereas
studies that use phlebography or radionucleotide venography indicate
the incidence is as high as 35%.

113 Sepsis causes a marked metabolic response. Which of the following


statement(s) is/are true concerning the metabolic response to sepsis?
a. Oxygen consumption is increased in the face of infection
b. In a patient with a maximal metabolic rate secondary to trauma, the
presence of infection will increase the rate further
c. Metabolic rate increases at a rate of approximately 10% for each
increase of 1°C in central temperature
d. The extent of increase in oxygen consumption relates to the severity
of the infection
Answer: a, c, d

Oxygen consumption is usually elevated in the infected patient. The


extent of this increase is related to the severity of the infection, with
peak elevations reaching 50% to 60% above normal. If the patient’s
metabolic rate is already elevated to a maximal extent because of
severe injury, no further increase will be observed. In patients with only
a slightly accelerated rate of oxygen consumption, the presence of
infection will cause a rise in metabolic rate added to the preexisting
state. A portion of the increase in metabolism may be ascribed to
increase in reaction rate associated with fever. Calculations suggest that
the metabolic rate increases 10% to 13% for each elevation of 1°C in
central temperature.

114 Interleukin-6 is recognized as the cytokine primarily responsible for


the alteration in hepatic protein synthesis recognized as the acute
phase response. Which of the following statement(s) is/are true
concerning acute phase protein response to surgical stress?
a. Glucocorticoid hormones inhibit this response
b. Proteins such as albumin and transferrin which serve in serum
transport are generally increased in this response
c. Examples of acute phase proteins include fibrinogen and C-reactive
protein
d. In general, the physiologic role of acute phase proteins are to reduce
the systemic effects of tissue damage
Answer: c, d

55 of 167 11-Aug-15 10:57 PM


MedCosmos Surgery: MCQ : General Surgery http://medcosmossurgery.blogspot.com/search/label/MCQ : General Surgery

IL-6 is now recognized at the cytokine primarily responsible for the


alteration in hepatic synthesis recognized as the acute phase response.
Glucocorticoid hormones augment this response. The primary metabolic
component of the acute phase response is a qualitative alteration in
hepatic protein synthesis with resulting alteration in plasma protein
composition. Characteristically, proteins which act as serum transport in
binding molecules, (albumin, transferrin) are reduced in quantity and
acute phase proteins (fibrinogen, C-reactive proteins) are increased.
Acute phase proteins are elaborated for the purpose of reducing the
systemic effects of tissue damage. Many act as anti-proteases, opsonins,
or coagulation and wound healing factors that generally inhibit the
tissue destruction that is associated with the local initiation of
inflammation.

115 A 59-year-old trauma patient has suffered multiple septic


complications including severe pneumonia, intraabdominal abscess, and
major wound infection. He has now developed signs of multisystem
organ failure. Which of the following statement(s) is/are true
concerning necessary changes to be made in his nutritional
management?
a. Carbohydrate load should be reduced in the face of respiratory
failure
b. In patients with renal failure, protein intake should be diminished
c. During hemodialysis protein intake should be limited to the same
extent
d. In patients with hepatic failure, carbohydrate load should be
increased
Answer: a, b

The most severe complication of sepsis is multiple system organ


dysfunction syndrome, which may result in death. The development of
organ failure requires changes in the nutritional requirements and
creates special feeding problems. A problem associated with systemic
infection is oxygenation and elimination of carbon dioxide. Most of the
enteral and parenteral formulas used to provide nutritional support for
critically ill patients contain large amounts of carbohydrate, which
generate large amounts of carbon dioxide following oxygenation. Such a
large CO2 load may worsen pulmonary function or may delay weaning
from the respirator. If this factor becomes a problem, the carbohydrate
load should be reduced to 50% of metabolic requirements and fat
emulsion administered to provide additional calories. When renal failure
becomes progressive, the use of hemodialysis minimizes the effect of
uremia superimposed on the metabolism of sepsis. Metabolic studies in
patients with acute and chronic renal failure have limited the intake of
nonessential amino acids, in an attempt to lower urea production.
Proteins of high biologic value, but in much smaller quantities than
usually given, are administered along with adequate calories, usually in
the form of glucose. When enteral feedings are not feasible, a central
venous infusion of an essential amino acid solution and hypertonic
dextrose provides calories and a small quantity of nitrogen to reduce
protein catabolism while simultaneously controlling the rise in BUN.

56 of 167 11-Aug-15 10:57 PM


MedCosmos Surgery: MCQ : General Surgery http://medcosmossurgery.blogspot.com/search/label/MCQ : General Surgery

During dialysis, protein intake is liberalized, but the BUN should still be
maintained below 100 mg/dl. Hepatic dysfunction is a common
manifestation of septicemia. The carbohydrate load is usually reduced
to consist of no more than 5% of metabolic requirements, and the
additional calories should be provided as fat emulsion. If
encephalopathy develops, protein load should also be reduced0.

116 Which of the following statement(s) is/are true concerning the role
of glutamine in total parenteral nutrition?
a. Glutamine is an essential amino acid
b. Glutamine appears to be of primary benefit in critical illness
c. Glutamine is included in most standard TPN solutions
d. Glutamine is the primary energy source for intestinal mucosal cells of
the small bowel and colon
Answer: b

Glutamine is the most studied gut-specific nutrient. Glutamine has been


classified as a nonessential or nutritionally dispensable amino acid since
glutamine can be synthesized in adequate quantities from other amino
acids and precursors. Glutamine is not included in most nutritional
formulas and has been eliminated from TPN solutions because of its
relative instability and short half life compared to other amino acids.
With few exceptions, glutamine is present in oral enteral diets but only
at relatively low levels characteristic of the concentration in most
animal and plant stores (about 7% of total amino acids). Several recent
studies, however, have demonstrated that glutamine may be an
essential amino acid during critical illness, particularly as it relates to
supporting the metabolic requirements of the intestinal mucosa. These
studies demonstrate that dietary glutamine is not required during states
of health but appears to be beneficial when glutamine depletion is
severe and/or when intestinal mucosa is damaged by insults such as
chemotherapy or radiation therapy. The addition of glutamine to enteral
diet reduces the incidence of gut translocation but these improvements
are dependent upon the amount of supplemental glutamine and the
type of insult studied. Glutamine-enriched TPN partially attenuates
villous atrophy that develops during parenteral nutrition. The use of
intravenous glutamine in patients appears to be safe and effective in its
ability to maintain muscle glutamine stores and improve nitrogen
balance. In contrast to glutamine, short chain fatty acids are primary
energy source for colonocytes.

117 Which of the following are determinants of the host response to


surgical stress?
a. Gender
b. Age
c. Nutritional status
d. Body composition
Answer: a, b, c, d

The pattern of physiologic changes elicited in response to surgical stress


results from the specific interaction of an individual patient with a
stressful stimulus. Several factors specific to the patient may determine

57 of 167 11-Aug-15 10:57 PM


MedCosmos Surgery: MCQ : General Surgery http://medcosmossurgery.blogspot.com/search/label/MCQ : General Surgery

the nature of the host response to stress. Body composition is a major


determinant of the metabolic responses observed during surgical illness.
Post-traumatic nitrogen excretion is directly related to the size of the
body protein mass. A strong relationship between protein depletion and
postoperative complications has been demonstrated in nonseptic,
nonimmunocompromised patients undergoing elective major
gastrointestinal surgery. Protein-depleted patients have significantly
lower preoperative respiratory muscle strength and vital capacity,
increased incidence of postoperative pneumonia, and longer
postoperative hospital stay. Impaired wound healing and respiratory,
hepatic, and muscle function in protein-depleted patients awaiting
surgery has also been reported. Many of the changes in the metabolic
responses to surgical illnesses that occur with aging can be attributed to
alterations in body composition and to long-standing patterns of physical
activity. Fat mass tends to increase with age and muscle mass tends to
decrease. Loss of strength that accompanies immobility, starvation and
acute surgical illness may have marked functional consequences.
Furthermore, the prevalence of cardiovascular and pulmonary diseases
increase with age. Thus, the delivery of oxygen to tissues may be
impaired in the elderly. Finally, observed differences in metabolic
responses of men and women generally reflect differences in body
composition. Lean body mass is lower in women than in men; and this
difference is thought to account for the net loss of nitrogen after major
elective abdominal surgery generally being lower in women than in
men.

118 In contrast to a patient undergoing an elective operation, which of


the following statement(s) is/are true concerning a patient who has
suffered a multiple trauma?
a. Basal metabolic rates are similar
b. The patient is highly sensitive to insulin
c. Utilization of the amino acids, glutamine and alanine, is similar to
their composition in skeletal muscle
d. Fat and protein stores are rapidly depleted
Answer: b, d

The degree of hypermetabolism is generally related to the severity of


injury. Patients with long-bone fractures have a 15–25% increase in
metabolic rate, whereas metabolic rates in patients with multiple
injuries increases by 50%. These metabolic rates in trauma patients are
contrasted with those in postoperative patients, who rarely increase
their BMR by more than 10–15% following operation. Studies have shown
that uninjured volunteers are able to dispose of exogenous glucose load
much more readily than injured patients. Other studies have
demonstrated a failure to suppress hepatic glucose production in trauma
patients during glucose loading or insulin infusion. Thus, profound
insulin resistance occurs in injured patients. Skeletal muscle is the
major source of nitrogen that is lost in the urine following extensive
injury. Although it is recognized that amino acids are released by muscle
in increased quantities following injuries, it has only been recently
appreciated that the composition of amino acid reflux does not reflect
the composition of muscle protein. The release is skewed towards

58 of 167 11-Aug-15 10:57 PM


MedCosmos Surgery: MCQ : General Surgery http://medcosmossurgery.blogspot.com/search/label/MCQ : General Surgery

glutamine and alanine, each of which comprise about one-third of the


total amino acids released by skeletal muscle. To support
hypermetabolism, stored triglyceride is mobilized at an accelerated
rate. Although mobilization and use of free fatty acids are accelerated
in injured subjects, if unfed, severely injured patients rapidly deplete
their fat and protein stores.

119 A 47-year-old patient undergoing a complicated laparotomy for


bowel obstruction develops a postoperative enterocutaneous fistula.
Which of the following statement(s) is/are true concerning parenteral
nutritional support in the postoperative period?
a. Oral intake can result in severe dehydration, electrolyte
abnormalities, and perifistula skin injury
b. Total parenteral nutrition increases the spontaneous closure rate of
intestinal fistula
c. Total parenteral nutrition decreases mortality rate in patients with
intestinal fistulas
d. The use of TPN better prepares the patient for surgery if surgical
intervention proves necessary
Answer: a, b, d

Patients with gastrointestinal-cutaneous fistulas represent the classical


indication for TPN. In such patients, oral intake of food almost
invariably results in increased fistula output with associated metabolic
disturbances, dehydration, skin breakdown, and death. Several
comprehensive reviews have concluded that TPN clearly impacts on the
treatment course of the disease in patients with GI fistulas. The
following conclusions can be drawn from studies evaluating the use of
TPN in patients with enterocutaneous fistula. First, TPN increases
spontaneous closure rate of enterocutaneous fistulas but does not
markedly decrease the mortality rate in patients with fistulas. Second,
if spontaneous closure of the fistula does not occur, patients are better
prepared for operative intervention because of the nutritional support
they have received. Finally, certain fistulas are associated with a lower
rate of spontaneous closure than others and should be treated more
aggressively surgically after a defined period of nutritional support
(unless closure occurs).

120 Appropriate guidelines for the use of TPN in cancer patients


include:
a. Long-term TPN in patients with rapid progressive tumor growth
unresponsive to other therapy
b. Mildly malnourished patients undergoing surgery for a curable cancer
c. Preoperatively administered TPN prior to surgery or other therapy in
patients with severe malnutrition
d. Patients in whom treatment toxicity precludes the use of enteral
nutrition
Answer: c, d

As a general rule, the most important factor to consider when making


decisions about the use of TPN in patients with cancer is the response of
the tumor to antineoplastic therapy. Appropriate guidelines would

59 of 167 11-Aug-15 10:57 PM


MedCosmos Surgery: MCQ : General Surgery http://medcosmossurgery.blogspot.com/search/label/MCQ : General Surgery

include the following: Short-term TPN is indicated in severely


malnourished patients or in those in whom gastrointestinal or other
toxicities preclude adequate enteral intake for seven days or a longer
period. TPN is not indicated in well nourished or mildly malnourished
patients undergoing therapy or surgery who would be expected to be
able to resume adequate nutrition in approximately seven days.
Long-term TPN is indicated in patients in whom treatment associated
toxicities preclude the use of enteral nutrition and represent the
primary impediment to the restoration of performance status. These
patients should be expected to be responding to anti-tumor therapy.
Long-term TPN is not indicated with rapidly progressive tumor growth
which is unresponsive to such therapy.

121 Which of the following statements(s) is/are true concerning human


energy requirement?
a. In normal subjects, less than 5% of basal energy requirement is spent
on cardiac output and the work of breathing
b. Mechanical ventilation can decrease the energy expenditure for
normal respiration
c. For a 70 kg male, average resting energy consumption is almost 1500
kcal/day
d. Similar increases in energy expenditures are associated with elective
surgery and trauma or thermal injury
Answer: a, c

Basal energy requirements are measured with the subject at rest when
no external work is being done; the energy is used mainly for transport
and synthetic work within cells. A surprisingly small percentage (< 5%)
of this energy is spent on cardiac output and the work of breathing in
normal subjects. In contrast, the work of breathing in individuals with
chronic obstructive lung disease or in patients on a ventilator may
account for 15–20% of caloric expenditure. The average resting
post-absorptive 70 kg male consumes about 1500 kcal/day. Energy needs
increase as severity of illness increases. The expenditure of kcal is only
minimally increased after elective surgery. The largest increase in
energy expenditure occurs in patients with severe multiple trauma or
major thermal injury. The average-sized adult who sustains a major burn
rarely may require more than 3500 kcal/day for maintenance.

122 Which of the following complications of TPN are appropriately


managed with the listed treatment?
a. Air embolism—place patient in reverse Trendelenburg and the left
lateral decubitus position and aspirate venous air
b. Hyperchloremic metabolic acidosis—give sodium and potassium as
acetate salts
c. Carbon dioxide retention—decrease glucose calories and replace with
fat
d. Line sepsis—intravenous antibiotics
Answer: b, c

A number of complications of TPN can occur which can be divided into


three types: mechanical, metabolic, and infectious.

60 of 167 11-Aug-15 10:57 PM


MedCosmos Surgery: MCQ : General Surgery http://medcosmossurgery.blogspot.com/search/label/MCQ : General Surgery

123 A 55-year-old male undergoes a total abdominal colectomy. Which


of the following statement(s) is/are true concerning the hormonal
response to the surgical procedure?
a. Adrenocorticotropic hormone (ACTH) is secreted from the anterior
pituitary gland
b. ACTH stimulation results in elevation of serum cortisol levels for up
to a week after the operation
c. An increased secretion of aldosterone and ADH may contribute to
postoperative fluid retention
d. An increase in serum insulin and a fall in glucagon accelerate hepatic
glucose production and maintain gluconeogenesis
Answer: a, c

One of the earliest consequence of a surgical procedure is the rise in


levels of circulating cortisol that occur in response to a sudden
outpouring of ACTH from the anterior pituitary. The rise in ACTH
stimulates the adrenal cortex to elaborate cortisol which remains
elevated for 24–48 hours after operation. The neuroendocrine responses
to operation also modify the various mechanisms that regulate salt and
water excretion. Alterations in serum osmolarity and tonicity of body
fluids secondary to anesthesia and operative stress, stimulate the
secretion of aldosterone and ADH. Thus, the ability to excrete a water
load after elective surgical procedures is restricted, and weight gain
secondary to salt and water retention is usual following an operation.
Alterations occur in response to the endocrine pancreas following
elective operation. Insulin elaboration is diminished and glucagon
concentrations rise. The rise in glucagon and the corresponding fall in
insulin are important signals to accelerate hepatic glucose production,
and, with other hormones (epinephrine and glucocorticoids),
gluconeogenesis is maintained.

124 A number of prospective clinical trials have addressed the role of


total parenteral nutrition in the cancer patient. The results have been
somewhat conflicting. Which of the following statement(s) have been
proven correct by prospective trials?
a. Preoperative TPN is beneficial in surgical patients with severe
preoperative nutrition
b. Postoperative TPN is of value following pancreatic resection
c. Routine use of perioperative (including prior to the procedure) TPN is
of benefit in patients undergoing hepatectomy for hepatoma
d. TPN is of no benefit in patients undergoing bone marrow transplant
Answer: a, c

Numerous clinical trials have failed to yield a consensus with regard to


the efficacy of TPN in cancer patients. In 1991, a multicenter VA
cooperative trial demonstrated that preoperative TPN is of benefit in
surgical patients (many of whom had cancer) with severe preoperative
malnutrition. Another study examined the use of routine postoperative
TPN following major pancreatic resection. Patients randomized to
receive TPN starting on postoperative day 1 were noted to have an
increased incidence of intra-abdominal abscesses as well as a tendency

61 of 167 11-Aug-15 10:57 PM


MedCosmos Surgery: MCQ : General Surgery http://medcosmossurgery.blogspot.com/search/label/MCQ : General Surgery

towards increased incidence in peritonitis and bowel obstruction. These


investigators concluded that routine use of postoperative TPN was not
indicated and may, in fact, be harmful following pancreatic resection. In
another study, however, perioperative (starting 7 days prior to the
planned procedure) TPN for patients undergoing hepatectomy for
hepatocellular carcinoma demonstrated that this regimen statistically
reduced infectious complications compared to patients who did not
receive TPN. This was one of the few studies that demonstrated that
routine TPN (without the requirement of severe preoperative
malnutrition) was of benefit. The use of TPN in patients receiving bone
marrow transplantation has also been shown to be a valuable
component of overall care.

125 Which of the following statements concerning perioperative


nutrition is true concerning the above-described patient?
a. Since the patient’s weight had been stable with no preoperative
nutritional deficit, 5% dextrose intravenous solutions are adequate for
the initial postoperative source of nutrition
b. Preoperative immunologic status should be determined including
total peripheral lymphocyte count and delayed hypersensitivity reaction
to determine skin-test response to common antigens
c. Routine postoperative fluid administration with intravenous 5%
glucose solutions can provide the calories to meet basal energy
requirements
d. A jejunal feeding catheter should be placed at the time of surgery for
postoperative enteral feeding
Answer: a

Most patients undergoing elective operations are adequately nourished.


Unless the patient has suffered significant preoperative malnutrition,
characterized by weight loss greater than 10–15%, or has major
intraoperative or postoperative complications, solutions containing 5%
dextrose may be administered for five to seven days before initiation of
enteral nutrition, with no detrimental effect on outcome. The usual
postoperative surgical patient is given intravenous glucose at 125
cc/hour receives about 500 kcal/day, far less than the actual number of
kcal needed to meet energy requirements. The increased cost of
feedings and potential complications associated with intravenous
nutrition cannot be justified. Although the use of jejunal feedings in the
postoperative period may be useful in some patients, especially those
undergoing extensive gastrointestinal surgery, this technique would not
appear indicated in the patient described above.

126 The neurohormonal arm of the stress response is well defined. Less
is known about the inflammatory arm mediated primarily by cytokines.
Which of the following statement(s) is/are true concerning this arm of
the surgical stress response?
a. Cytokines primarily work locally via direct cell-to-cell communication
b. Cytokines are never detectable in the systemic bloodstream
c. Cytokines are produced only by immune cells attracted to the site of
injury
d. Cytokine release may stimulate the release of other cytokines leading

62 of 167 11-Aug-15 10:57 PM


MedCosmos Surgery: MCQ : General Surgery http://medcosmossurgery.blogspot.com/search/label/MCQ : General Surgery

to an important cascade of events


Answer: a, d

Cytokines, which are produced at the site of injury by endothelial cells


and by diverse immune cells throughout the body, also occupy a pivotal
position in the stress response. Cytokines differ from classic endocrine
hormones in that they are produced by a variety of cell types and in
that they have the capacity to exert their tissue effects locally via
direct cell-to-cell communications in a paracrine and/or autocrine
fashion. Cytokines can stimulate the production of other cytokines,
leading to important cascades which both amplify and diversify the
effects of the proximal cytokine. Occasionally, when in excess,
cytokines act as hormones and “spill over” into the systemic circulation
and become detectable in the bloodstream.

127 Which of the following tissues contain significant collagen useful for
placing sutures to allow the prolonged tension necessary to maintain
tissue approximation?
a. Dermis
b. Intestinal submucosa
c. Muscular fascia
d. Blood vessel wall
Answer: a, b, c, d

It takes at least three weeks for collagen to undergo sufficient


remodeling and cross linking to attain moderate strength. Since most
skin sutures are removed at one to two weeks, the wound has only a
small fraction of its eventual strength and may therefore disrupt with
even modest stress. Therefore, deep sutures are placed in collagen
containing structures to maintain the prolonged tension necessary.
Dermis, intestinal submucosa, muscular fascia, tendon, ligament,
Scarpa’s fascia, and blood vessel wall represent a partial list of tissues
with high collagen content.

128 Products of platelet degranulation include:


a. Tumor necrosis factor
b. Interleukin-1
c. Transforming growth factor b
d. Platelet-derived growth factor
Answer: c, d

The initial response to injury and disruption of a blood vessel is


bleeding. The hemostatic response to this is clot formation to stop
hemorrhage. Platelet plug formation initiates the hemostatic process
along with clotting factors activated by collagen and the basement
membrane proteins exposed by the injury. Platelets then degranulate,
releasing the contents of their alpha granules and dense granules, most
notably platelet derived growth factor and transforming growth factor
b. These substances initiate chemotaxis and proliferation of
inflammatory cells, beginning the inflammatory response that will
ultimately heal the wound. Tumor necrosis factor and interleukin-1 also
stimulate fibroblast proliferation, however are produced by

63 of 167 11-Aug-15 10:57 PM


MedCosmos Surgery: MCQ : General Surgery http://medcosmossurgery.blogspot.com/search/label/MCQ : General Surgery

macrophages.

129 A patient with gross fecal contamination and peritonitis from a


ruptured sigmoid diverticulum has his midline wound left open to heal
by secondary intention. Which of the following statement(s) describes
this healing process?
a. Wounds healing in this fashion have an altered sequence of healing
compared to a primarily closed wound
b. A bed of granulation tissue forms over exposed subcutaneous tissue
c. Epithelialization is enhanced in the face of bacterial colonization
d. The ability of a wound to form granulation tissue is dependent on the
blood supply of the tissue
Answer: b, d

Open wounds, whether they be ulcers or open surgical incisions closing


by secondary intention, heal with the same sequence of inflammation,
matrix deposition, epithelialization, and scar maturation as in all
wounds. The major difference is in the healing incisional wound, the
healing process progresses in an orderly temporal sequence. In an open
wound, the healing events are spatially separated. In the healing
wound, a bed of granulation tissue forms over the exposed subcutaneous
tissue. Granulation tissue is composed of new capillaries, proliferating
fibroblasts, an immature matrix of collagen, proteoglycans, substrate
adhesion molecules, and acute and chronic inflammatory cells.
Granulation tissue is the cobblestone pink surface of the healthy new
tissue in an open wound. The ability of an open wound to form
granulation tissue is governed by the blood supply to the tissue and the
relative absence of devitalized tissue and bacteria. Epithelialization is
therefore enhanced by limiting bacterial growth which presumably
interferes via bacterial and phagocytic cell products such as proteases,
collagenases, elastases, and other enzymes.

130 Which of the following factors can be associated with impaired


wound healing?
a. Chemotherapy
b. Chronic steroid use
c. Peripheral vascular disease
d. Radiation therapy
e. Diabetes mellitus
Answer: a, b, c, d, e

Bone marrow suppression, a common consequence of chemotherapy, is


detrimental to wound healing. Quantitative and qualitative lymphocyte
and monocyte deficiency impairs cellular proliferation in the
inflammatory phase of wound healing. Any chemotherapeutic agent that
suppresses the bone marrow will impair healing. Glucocorticoids inhibit
wound healing based on their anti-inflammatory and immunosuppressive
effects. The anti-inflammatory effect of steroids is, in part, the result
of inhibiting arachidonic acid metabolism by impairing macrophage
migration, and by altering neutrophil function. Glucocorticoids also
inhibit the synthesis of procollagen by fibroblasts, thus delaying wound
contraction. Radiation injury leads to arteriolar fibrosis and impaired

64 of 167 11-Aug-15 10:57 PM


MedCosmos Surgery: MCQ : General Surgery http://medcosmossurgery.blogspot.com/search/label/MCQ : General Surgery

oxygen delivery. In addition, there is progressive obliteration of blood


vessels in the radiated area over time. Radiation also causes
intranuclear and cytoplasmic damage to fibroblasts, and this appears to
limit their proliferative potential. Diabetes mellitus is often associated
with decreased healing of open wounds and increased susceptibility of
infection. Many factors contribute to poor healing in diabetic patients
and most of them reflect local wound ischemia. However, healing is not
impaired in a normally perfused area in a well-controlled diabetic.
Peripheral arterial occlusive disease secondary to atherosclerosis can be
a primary cause of impaired healing, and may be also a cofactor with
other conditions.

131 Which of the following cells or blood elements play a role in the
initial phases of wound healing?
a. Polymorphonuclear leukocytes (PMNs)
b. Platelets
c. Monocytes
d. Lymphocytes
Answer: a, b, c, d

Shortly after the initial injury, the wound is full of debris which is
cleared over the next several days by recruited and activated
phagocytic cells. PMNs begin to arrive immediately, reaching large
numbers within 24 hours. The PMNs are followed by macrophages which
appear in wounds in significant numbers within two to three days.
Macrophages are mononuclear phagocytic cells derived from circulating
monocytes or resident tissue macrophages. They complete the process
of removing all material not necessary for the ensuing steps of wound
healing. Lymphocytes also appear in wounds in small numbers during the
inflammatory response. The role of lymphocytes in the wound healing
process remains to be clarified, but they are thought to be more related
to the chronic inflammatory processes than the initial response to
wounding. Platelets are anuclear discoid blood elements derived from
bone marrow megakarocytes which play a role in the initial hemostatic
process as well as releasing chemotactic factors and factors leading to
fibroblast proliferation.

132 Which of the following surgical techniques lead to improved wound


healing?
a. Atraumatic handling of tissue
b. Approximation of underlying fatty tissue to obliterate dead space
c. Protecting the wound from water for at least one week
d. Meticulous hemostasis
Answer: a, d

There are numerous practical implications for the care of wounds and
surgical incisions. Meticulous hemostasis reduces the inflammation of
phagocytosis necessary to clear the wound of blood. Atraumatic
handling of tissue decreases the load of necrotic or nonviable cells at
the wound margin. Deep sutures are best placed only into collagen
laden structures that will hold tension, i.e., fascia and dermis. These
tissues have a tensile strength to hold sutures under tension. Fat does

65 of 167 11-Aug-15 10:57 PM


MedCosmos Surgery: MCQ : General Surgery http://medcosmossurgery.blogspot.com/search/label/MCQ : General Surgery

not contain collagen and will not hold tension. Therefore, fatty tissue
should not be sutured as a separate layer. Given that epithelialization of
an incision is normally complete within 24–48 hours, there is no reason
to protect the incision from water beyond this time period. Allowing the
patient to wash or shower one or two days after surgery actually serves
useful purpose in debriding the wound.

133 Which of the following statement(s) is/are true concerning the


clinical management of an open wound?
a. A wet-to-dry dressing is the most optimal form of wound management
b. A moist occlusive dressing promotes epithelialization and reduces
pain
c. The protein rich plasma exudate covering the open wound facilitates
healing
d. Irrigation of the wound disrupts epithelialization therefore inhibiting
the healing process
Answer: b

Epithelialization is more rapid under moist conditions than dry


conditions. Without dressings, a superficial wound, or one with minimal
devitalized tissue forms a scab or crust, meaning that the blood and
serum will coagulate, dry, and form a protective moisture barrier over
the open wound. If a wound is kept moist with an occlusive dressing,
then epithelial migration is optimized. In addition, the pain of an open
wound is dramatically reduced under an occlusive dressing. The
traditional wet-to-dry dressing if truly left to dry, simply produces
desiccation and necrosis of the surface layer of the wound which delays
epithelialization. Although wet-to-dry dressings can be effective for
debridement of wound exudate, they are generally less desirable than a
moist healing environment combined with effective cleaning of the
wound (i.e. water irrigation). Any open wound will leak plasma. With
more inflammation, the plasma capillary permeability is further
increased. This exudate of serum proteins and inflammatory cells serves
as a rich culture medium. This, in turn, will continue to cycle bacterial
proliferation and lead to further exudate formation. The net result of
this cycle is delayed or absent wound healing. In addition, the edema
that results from capillary dysfunction, increases the distance for
diffusion from oxygen and nutrient sources to their metabolic targets.

134 Which of the following statement(s) is/are correct concerning the


management of an open wound?
a. Frequent surgical debridement is usually necessary
b. Water irrigation can effectively debride most wounds
c. Hydrogen peroxide is particularly useful in the management of open
wounds
d. A number of the newer dressing products have clearly been shown to
promote wound healing compared to simple moist occlusive dressing
Answer: b

Although there are numerous dressing products commercially available


at present, no treatment has been demonstrated to improve healing
beyond that of standard treatment which adheres to basic principles. In

66 of 167 11-Aug-15 10:57 PM


MedCosmos Surgery: MCQ : General Surgery http://medcosmossurgery.blogspot.com/search/label/MCQ : General Surgery

the absence of large amounts of necrotic tissue, wound debridement


does not need to be accomplished surgically. Simple water irrigation
either with whirlpool or by water from a hand held shower spray can
generate enough power to effectively debride most wounds. Frequent
moist dressing changes can accomplish this as well, and in some cases,
occlusive absorptive dressings can generate enough tissue proteases to
effectively degrade proteins which the absorptive dressings remove.
Deeper portions of a wound may accumulate exudate and bacteria. In
such cases, water irrigation may be particularly useful. Commonly used
agents such as hydrogen peroxide actually may be harmful to normal
tissue and are weak oxidants and do a poor job of debriding. Enzymatic
debriding agents can be effective when used properly. Most of the
newer dressing products have been designed to be more absorptive and
achieve moist healing without infection from excess exudate. However,
it must be emphasized that as long as moist healing is achieved, there
has been no evidence that one product is better than another.

135 Which of the following statement(s) is/are true concerning the


proliferative phase of wound healing?
a. The macrophage is the predominant cell type
b. The pink or purple-red appearance of a wound is due to ingrowth and
proliferation of endothelial cells
c. Collagen, the dominant structural molecule of the wound matrix,
contains two unique amino acids, hydroxyproline and hydroxylysine
d. The predominant collagen type in a scar is type 3
Answer: b, c

The proliferative phase of wound healing begins with the formation of a


provisional matrix of fibrin and fibronectin as part of the initial clot
formation. Initially, the provisional matrix is populated by macrophages;
however, by day three fibroblasts appear in the fibronectin-fibrin
framework and initiate collagen synthesis. Fibroblasts proliferate in
response to growth factors become the dominant cell type during this
phase. Growth factors produced by macrophages simultaneously induce
angiogenesis which results in the ingrowth and proliferation of
endothelial cells, forming new capillaries. This neovascularity is visible
through the epithelium and gives the wound a pink or purple-red
appearance.
Collagen is the dominant structural molecule in the wound matrix and in
the final scar. Collagen is synthesized into an organized cable-like
network in a multi-step process with both intra- and intercellular
components. The collagen molecule has quantities of two unique amino
acids, hydroxyproline and hydroxylysine. The hydroxylization processes
which form these amino acids require ascorbic acid (vitamin C) and is
necessary for the subsequent stabilization and cross linkage of collagen.
The principal collagen type scar is type 1, with lesser amounts of type 3
collagen also present.

136 Which of the following statement(s) is/are true about the role of
macrophages in the wound healing process?
a. Macrophages are the dominant cell type during the inflammatory
phase of wound healing

67 of 167 11-Aug-15 10:57 PM


MedCosmos Surgery: MCQ : General Surgery http://medcosmossurgery.blogspot.com/search/label/MCQ : General Surgery

b. Macrophages are not essential for wound healing


c. The macrophage role in wound healing is limited to phagocytosis
d. Macrophages are a source of a number of humoral factors essential
for wound healing
Answer: a, d

Within three or four days after injury, macrophages become the


dominant cell type in the inflammatory phase of wound healing. The
role of macrophages is not limited only to phagocytosis. In addition,
macrophages are the source of more than 30 different growth factors
and cytokines. These growth factors induce fibroblast proliferation,
endothelial cell proliferation (angiogenesis), extracellular matrix
production, and recruit and activate additional macrophages. The result
is the induction of a wound healing amplification cycle as growth factors
recruit macrophages and elicit additional growth factor release.
Experimental studies in which antibodies, which either destroy PMNs or
block certain aspects of their function, have shown that wounds heal
normally, but that healing is significantly impaired without functional
macrophages. These studies confirm the dominant role of the
macrophage and the inflammatory phase of wound healing.

137 Which of the following statement(s) is/are true concerning the role
of antibiotics in wound care?
a. Systemic antibiotics are indicated for all open wounds
b. Bacterial resistance can occur with systemic but not topical
antibiotics
c. An indication for systemic antibiotic administration is a granulation
tissue bacterial count in excess of greater than 105 organisms/gram of
tissue on quantitative analysis
d. Silver sulfadiazine is useful only for the management of burns
Answer: c

The role of antibiotics in wound care is controversial. All open wounds


are colonized with bacteria. Only when surrounding tissue is invaded
(cellulitis) are systemic antibiotics clearly indicated. Antibiotics may
also be useful in other situations such as when granulation tissue has a
high bacterial count (> 105 organisms/gram tissue), or in the case of
reduced resistance to bacteria such as in a diabetic foot ulcer. The
routine use of systemic antibiotics for chronic wounds should be avoided
to reduce the development of resistant bacterial strains within the
wound. Topical ointments are frequently used and can be useful. The
topical vehicle may help keep the wound moist and the bacterial count
in the wound may be lowered as the result. However, as with most
antibiotics, resistant organisms quickly emerge. Silver sulfadiazine,
frequently used for burn care, is also useful for chronic wounds. Its
broad spectrum of activity, lack of relevant drug-resistant plasmids in
bacteria, and its low cost make it a good choice.

138 Which of the following statement(s) is/are true concerning wound


contraction?
a. Wound contraction accounts for similar rates of reduction of wound
size regardless of their location

68 of 167 11-Aug-15 10:57 PM


MedCosmos Surgery: MCQ : General Surgery http://medcosmossurgery.blogspot.com/search/label/MCQ : General Surgery

b. The fibroblast, at the cellular level, is the primary force driving


wound contraction
c. Excessive wound contraction, when occurring over a joint, may lead
to disability
d. Actin microfillaments are found in fibroblasts and may play a role in
wound contracture
Answer: b, c, d

Wound contraction is an important event which contrasts healing open


wounds and closed incisions. When open wounds contract, the
surrounding skin is pulled over the open wound to reduce its size. This
can occur much faster than epithelialization. As opposed to other
animals, human skin does not have a significant degree of mobility in
most sites and specifically on the lower leg, the skin is tightly adherent
and less elastic. Therefore, although contraction may account for 90% of
reduction of wound size on the perineum, it accounts for, at most,
30–40% of healing of a lower leg ulcer. All healing wounds generate a
strong contractile force. When this force is exerted across a joint, it
may result in scar contracture which may limit the functional range of
motion. At the cellular level, the force which drives wound contraction
comes from fibroblasts. Fibroblasts, like muscle cells, contain actin
microfilaments. When these filaments increase in number, the cells take
a morphologic appearance of myofibroblasts. Myofibroblasts are seen in
an increased number in contracting wounds and are felt to play an
active role in the process of wound contraction.

139 There are a multitude of various dressings available. Which of the


following statement(s) is/are true concerning options for surgical
dressings?
a. Hydrocolloids, such as karaya compounds, offer the primary
advantage of increased absorptive ability
b. Films, such as Op-site, provide a water impermeable environment to
achieve a dry wound
c. Impregnates are fine gauze impregnated with a variety of substances
such as antibiotics or moisturizing agents that adhere tightly to the
wound and do not require a secondary dressing
d. Absorptive powders and paste are highly useful in debriding necrotic
and fibrous material from wounds and absorbing wound serum
Answer: a, d

Although the simplest dressing of gauze and tape combined with the use
of antibacterial ointment can achieve moist wound healing in most
patients. A multitude of other products are available. These can be
classified into films, foams, hydrocolloids, hydrogels, and absorptive
powders. Films are semipermeable to water, generally made of
polyurethane, and are nonabsorptive. They are useful to achieve a moist
wound healing environment over a minimally exudative wound such as
split thickness skin graft donor sites. The hydrocolloids deserve special
mention because they have achieved widespread use. These agents
contain hydrophilic materials such as karaya or carboxymethyl cellulose
with an adhesive material and are covered by a semipermeable
polyurethane film. The material adheres to the skin surrounding the

69 of 167 11-Aug-15 10:57 PM


MedCosmos Surgery: MCQ : General Surgery http://medcosmossurgery.blogspot.com/search/label/MCQ : General Surgery

wound, is highly absorptive, and achieves a moist healing environment.


Impregnants are generally fine mesh gauze impregnated with either
moisturizing, antibacterial, or bactericidal compounds. They are
generally not adherent and require a secondary dressing. They do
promote reepithelialization and have a antiinfective effect when
combined with antibacterial or bactericidal agents. A variety of
absorptive powders and pastes are available which consist of starch
copolymers or colloidal hydrophilic particles. These agents have high
absorbency for tissue wound fluid and debride necrotic and fibrous
material from the wound.

140 Which of the following statement(s) is/are true concerning the


remodeling phase of wound healing?
a. Total collagen content increases steadily through this phase
b. The normal adult ratio of collagen is approximately 4:1 of type I to
type III collagen.
c. Eventually a scar will achieve the strength of unwounded skin
d. The proteoglycans are responsible for the ground substance of the
extracellular matrix
Answer: b, d

The transition from the proliferative phase to the remodeling phase of


wound healing is defined by reaching collagen equilibrium. Collagen
accumulation within the wound becomes maximal by two to three
weeks after wounding. Although supramaximal rates of synthesis and
degradation continue throughout remodeling, there is no further change
in total collagen content. During the initial phase of wound healing,
there is a relative abundance of type III collagen in the wound. With
remodeling, the normal adult ratio of 4:1 (type I to type III) collagen is
restored. The other important component of the extracellular matrix is
the ground substance or proteoglycans. These substances are composed
of a protein background with long hydrophilic carbohydrate side chains.
The hydrophilic nature of these molecules accounts for much of the
water content of scar.
Scars never achieve the degree of order advanced by collagen in normal
skin or tendons, but they do increase in strength for six months or more,
eventually reaching 70% of the strength of unwounded skin.

141 Which of the following statement(s) is/are true concerning


pharmacologic agents used to accelerate wound healing?
a. A number of these agents are now currently approved for use in this
country
b. PDGF (platelet-derived growth factor) promotes fibroblast
proliferation, chemotaxis, and collagenase synthesis
c. PDGF has been demonstrated in a number of clinical trials to promote
healing in chronic wounds
d. Growth hormone functions by promoting fibroblast proliferation and
collagen synthesis
Answer: b, c

Currently there are no approved clinical agents that accelerate normal


healing. Although a number of clinical trials are in progress, no agents

70 of 167 11-Aug-15 10:57 PM


MedCosmos Surgery: MCQ : General Surgery http://medcosmossurgery.blogspot.com/search/label/MCQ : General Surgery

are currently approved. PDGF (platelet-derived growth factor)


accelerates wound healing by promoting fibroblast proliferation and
chemotaxis and collagenase synthesis. Clinical trials have demonstrated
that PDGF has accelerated healing in patients with chronic wounds such
as pressure sores and diabetic ulcers. Growth hormone has been
successfully used in some situations to reverse the catabolic effect of
severe injuries. Wound healing is fundamentally an anabolic event, and
in the setting of a severe burn, growth hormone administration
significantly accelerates donor site healing, presumably due to its
effects in minimizing catabolism.

142 Which of the following statement(s) describe the effects of aging on


wound healing?
a. A finer, more cosmetic scar might be expected
b. In vitro studies demonstrate decreased proliferative potential of
fibroblasts and epithelial cells
c. Skin sutures should be left in for a longer period of time
d. Wound infection occurs more frequently in elderly patients due to
diminished ability to fight infection
Answer: a, b, c

There are important age-dependent aspects of wound healing. The


elderly heal more slowly and with less scarring. There is a gradual
attenuation of the inflammatory response with age, and decreased
wound healing is one of the consequences. In vitro studies have
documented an age-dependent decrease in proliferative potential of
fibroblasts and epithelial cells. Clinically this will account for the
formation of finer scars and improved cosmetic appearance in the
elderly. Sutures should be left in place longer to allow for the slow
regain of tensile strength in the aged. This can also be done without
concern for formation of suture marks as slower epithelialization occurs
along the sutures. There is no evidence to suggest that wound infections
occur more commonly in elderly patients.

143 Reconstitution of the epithelial barrier (epithelialization) begins


within hours of the initial injury. Which of the following statement(s)
is/are true concerning the process of epithelialization?
a. Bacteria, protein exudate, and necrotic tissue all will compromise
this process
b. Epithelial cells exhibit contact proliferation
c. Epithelialization occurs only from the margins of the wound
d. Visible scarring can occur only when the injury extends deeper than
the superficial dermis
Answer: a, d

The initial step of epithelialization involves epithelial cells from the


basal layer of the wound edge flattening and migrating across the
wound, completing wound coverage within 24–48 hours in a co-opted
surgical wound. Epithelial cells exhibit contact inhibition. That is, they
will continue to migrate across an appropriate bed until a single
continuous layer is formed. Epithelial cell migration occurs by a process
in which the epithelial cells send out pseudopods, attaching to the

71 of 167 11-Aug-15 10:57 PM


MedCosmos Surgery: MCQ : General Surgery http://medcosmossurgery.blogspot.com/search/label/MCQ : General Surgery

underlying extracellular matrix by integrin receptors. Bacteria, large


amounts of protein exudate from leaky capillaries, and necrotic tissue
all compromise this process delaying epithelialization. In the case of
open wounds, epithelialization results from migration of epithelial cells
from remaining dermal appendages, sweat glands, and hair follicles, if
the dermis is not completely destroyed. In a full thickness injury, the
entire dermis is destroyed or removed. Epithelialization therefore
occurs only at the margins of a wound, at a dermal rate of 1–2 mm/day.
Visible scarring occurs only when the injury extends deeper than the
superficial dermis. Superficial abrasions and burns usually heal without
scar, while deeper abrasions and burns may scar significantly. Whenever
the dermis is incised, a scar will form.

144 Scar formation is part of the normal healing process following


injury. Which of the following tissues has the ability to heal without scar
formation?
a. Liver
b. Skin
c. Bone
d. Muscle
Answer: c

Every tissue in the body undergoes reparative processes after injury.


Bone has the unique ability to heal without scar and liver has the
potential to regenerate parenchyma, the only organ that has maintained
that ability in the adult human. Although liver does regenerate, it often
heals with scar (cirrhosis) as well. With these exceptions, all other
mature human tissues heal with scar.

145 Which of the following factors have been demonstrated to promote


wound healing in normal individuals?
a. Vitamin A supplementation
b. Vitamin C supplementation
c. Vitamin E application to the wound
d. Zinc supplementation
e. None of the above
Answer: e

Several important systemic factors or conditions influence wound


healing. Interestingly, there are no known systemic conditions that lead
to enhanced or more rapid wound healing. Overall nutrition as well as
adequate vitamins play an important role in wound healing. Vitamin A is
involved in the stimulation of fibroplasia, collagen cross-linking, and
epithelialization. Although there is no conclusive evidence in humans,
vitamin A may be useful clinically for steroid-dependent patients who
have problematic wounds or who are undergoing extensive surgical
procedures. Vitamin C is a necessary cofactor in hydroxylization of
lysine and proline in collagen synthesis and cross-linkage. The utility of
vitamin C supplementation in patients who otherwise take in a normal
diet has not been established. Vitamin E is applied to wounds and
incisions empirically by many patients. The evidence to support this
practice is entirely anecdotal. In fact, large doses of vitamin E have

72 of 167 11-Aug-15 10:57 PM


MedCosmos Surgery: MCQ : General Surgery http://medcosmossurgery.blogspot.com/search/label/MCQ : General Surgery

been found to inhibit wound healing. Zinc and copper are also important
cofactors for many enzyme systems that are important to wound
healing. Deficiency states are seen with parenteral nutrition but are
rare and readily recognized and treated with supplements. Overall,
vitamin and mineral deficiency states are extremely rare in the absence
of parenteral nutrition or other extreme dietary restrictions. There is no
evidence to support the concept that supranormal provision of these
factors enhance wound healing in normal patients.

146 Which of the following statement(s) is/are true concerning


excessive scarring processes?
a. Keloids occur randomly regardless of gender or race
b. Hypertrophic scars and keloid are histologically different
c. Keloids tend to develop early and hypertrophic scars late after the
surgical injury
d. Simple reexcision and closure of a hypertrophic scar can be useful in
certain situations such as a wound closed by secondary intention
Answer: d

True keloids are uncommon and occur predominantly in dark skinned


people with a genetic predisposition for keloid formation. In most cases,
the gene appears to be transmitted as an autosomal dominant pattern.
The primary difference between a keloid and a hypertrophic scar is that
a keloid extends beyond the boundary of the original tissue injury. It
behaves as a tumor and extends into or invades the normal surrounding
tissue creating a scar that is larger than the original wound.
Histologically, keloids and hypertrophic scars are similar. Both contain
an overabundance of collagen. Although the absolute number of
fibroblasts is not increased, the production of collagen continually out
paces the activity of collagenase, resulting in a scar of ever increasing
dimensions. Hypertrophic scars respect the boundaries of the original
injury and do not extend into normal unwounded tissue. There is less of
a genetic predisposition, but hypertrophic scars also occur more
frequently in Orientals and the Black population. They are often seen on
the upper torso and across flexor surfaces. Some improvement in a
keloid can be obtained with excision followed by intra-lesional steroid
injection. However, the resulting scar is unpredictable and potentially
worse. Reexcision and closure should, however, be considered for
hypertrophic scars, if the condition of closure can be improved. This is
especially pertinent for wounds that originally healed by secondary
intention or that are complicated by infection. Keloids typically develop
several months after the injury and rarely, if ever, subside. Hypertrophic
scars usually develop within the first month after wounding and often
subside gradually.

147 Which of the following statement(s) is/are true concerning the


vascular response to injury?
a. Vasoconstriction is an early event in the response to injury
b. Vasodilatation is a detrimental response to injury with normal body
processes working to avoid this process
c. Vascular permeability is maintained to prevent further cellular injury
d. Histamine, prostaglandin E2 (PGE2) and prostacyclin (PGI2) are

73 of 167 11-Aug-15 10:57 PM


MedCosmos Surgery: MCQ : General Surgery http://medcosmossurgery.blogspot.com/search/label/MCQ : General Surgery

important mediators of local vasoconstriction


Answer: a

After wounding, there is transient vasoconstriction mediated by


catecholamines, thromboxane, and prostaglandin F2 (PGF2a). This
period of vasoconstriction lasts for only five to ten minutes. Once a clot
has been formed and active bleeding has stopped, vasodilatation occurs
in an around the wound. Vasodilatation increases local blood flow to the
wounded area, supplying the cells and substrate necessary for further
wound repair. The vascular endothelial cells also deform, increasing
vascular permeability. The vasodilatation and increased endothelial
permeability is mediated by histamine, PGE2, and prostacyclin as well
as growth factor VEGF (vascular endothelial cell growth factor). These
vasodilatory substances are released by injured endothelial cells and
mast cells and enhance the egress of cells and substrate into the wound
and tissue.

148 Which of the following statement(s) concerning laboratory studies


used in monitoring a patient with intravenous heparinization is/are
correct?
a. The platelet count should be followed because of the risk of heparin-
associated thrombocytopenia
b. The prothrombin time should be observed if prolonged treatment is
necessary
c. The activated partial thromboplastin time (aPTT) should be
maintained at approximately 1.5 times normal
d. The serum creatinine should be measured daily to allow adjustments
in dose based on renal function
Answer: a, c

In monitoring the effect of heparin, an activated partial thromboplastin


time (aPTT) of 1.5 control or a thrombin clotting time (TCT) of 2 times
control reflects adequate anticoagulation. The prothrombin time
remains normal. Heparin-associated thrombocytopenia from an immune
mechanism is a potential complication of the use of this anticoagulant.
Therefore any patient undergoing heparin therapy should have a
platelet count determined every other day after the fourth day of
therapy or earlier if he or she is known to have been exposed to heparin
in the past. Heparin is not excreted through the kidneys or the liver but
is cleared through the reticuloendothelial system. Therefore the dose of
heparin need not be adjusted in cases of liver or renal dysfunction.

149 Which of the following statement(s) is/are true concerning heparin-


associated thrombocytopenia?
a. Heparin-associated thrombocytopenia occurs only in the face of over
anticoagulation with heparin
b. Severe thrombocytopenia (platelet count less than 100,000) is seen in
less than 10% of patients treated with heparin
c. Heparin-associated thrombocytopenia is due to the aggregation of
platelets and may result in thrombosis or embolic episodes
d. Heparin-associated thrombocytopenia may be seen within hours of
initiation of heparin therapy

74 of 167 11-Aug-15 10:57 PM


MedCosmos Surgery: MCQ : General Surgery http://medcosmossurgery.blogspot.com/search/label/MCQ : General Surgery

Answer: b, c

Heparin-associated thrombocytopenia occurs in 0.6% to 30% of patients


who receive heparin, although severe thrombocytopenia (platelet
counts less than 100,000) is seen in fewer than 10% of patients treated
with heparin. It is caused by a plasma factor, most likely a heparin-
dependent platelet antibody, that causes aggregation of platelets when
exposed to heparin. Activation of platelets in this setting results in
thrombocytopenia, thrombosis and embolic episodes, which can lead to
death. Both bovine and porcine heparin have been associated with this
syndrome, which usually begins 5 to 15 days after initiating heparin
therapy. Even trivial exposure with heparin such as coating on
pulmonary artery catheters or low rate infusion into arterial catheters
may cause this syndrome.

150 Antithrombin III deficiency is a commonly observed hypercoaguable


state. Which of the following statement(s) is/are true concerning this
condition?
a. A patient with this deficiency usually presents with thrombosis while
on heparin or exhibits an inability to become adequately anticoagulated
with heparin
b. This deficiency may be either congenital or acquired
c. Thrombotic episodes are related to predisposing events such as
operations, childbirth, and infections
d. Treatment involves acutely the administration of fresh frozen plasma
followed by long-term treatment with Coumadin
Answer: a, b, c, d

Antithrombin III deficiency accounts for about 2% of venous thrombotic


event. This deficiency has been described in patients with pulmonary
embolism, mesenteric venous thrombosis, lower extremity venous
thrombosis, arterial thrombosis, and dialysis fistula failure.
Antithrombin III is a serine protease inhibitor of thrombin and factors
Xa, IXa and XIa. Because one of the main actions of heparin is to
potentiate the anticoagulant effects of antithrombin III, a patient with
this deficiency usually presents with thrombosis while on heparin or
exhibits the inability to become adequately anticoagulated with
heparin. This deficiency may be either congenital (1n2000–5000 births)
or acquired. Acquired defects occur with inadequate production, as in
liver disease, malignancy, nephrotic syndrome, disseminated
intervascular coagulation, malnutrition, or increased protein
catabolism. Thrombotic episodes are related to predisposing events such
as operations, childbirth, and infections. Once the diagnosis of
antithrombin III deficiency is established, fresh frozen plasma should be
administered followed by long-term treatment with Coumadin.

151 Mini-dose heparin has been shown to be useful in the prophylaxis of


postoperative venous thrombosis. Mechanism(s) by which low-dose
heparin is/are thought to protect against venous thrombosis include:
a. Enhancement of antithrombin III activity
b. A decrease in thrombin availability
c. Inhibition of platelet aggregation and subsequent platelet release

75 of 167 11-Aug-15 10:57 PM


MedCosmos Surgery: MCQ : General Surgery http://medcosmossurgery.blogspot.com/search/label/MCQ : General Surgery

action
d. A mild prolongation of activated partial thromboplastin time
Answer: a, b, c

Low-dose heparin is thought to protect against venous thrombosis


through three different mechanisms. First, antithrombin III activity with
its inhibition of activated Factor X is enhanced by only trace amounts of
heparin; second, there is a decrease in thrombin availability that
prevents its activation and thus its fibrin-stabilizing effect; and third,
small doses of heparin may inhibit the second wave of platelet
aggregation and subsequent platelet release reaction. The standard
doses of heparin administered (5000 units bid) does not affect aPTT.

152 Tests of coagulation are used to monitor anticoagulation treatment


and detect intrinsic abnormalities in coagulation. Which of the following
statement(s) is/are true concerning coagulation tests?
a. Prothrombin time (PT) measures both the intrinsic and extrinsic
clotting pathways and fibrinogen
b. Activated partial thromboplastin time (aPTT) can be used to monitor
both oral anticoagulation with Warfarin and intravenous anticoagulation
with heparin
c. Thrombin clotting time (TCT) is a measurement of the time it takes
for exogenously administered thrombin to turn plasma fibrinogen into
fibrin clot
d. Whole blood activated clotting time (ACT) is a measurement of the
ability of whole blood to clot and is used to monitor heparin levels
intraoperatively during cardiovascular and peripheral vascular
operations
Answer: a, c, d

Coagulation tests include prothrombin time (PT), which measures the


intrinsic and extrinsic pathways of fibrinogen production and is the most
common method for measuring a level of oral anticoagulant therapy.
The activated partial thromboplastin time (aPTT) identifies the
abnormalities of the contact and intrinsic phases of coagulation. Values
of aPTT have variably been shown to correlate with heparin dosages and
serum heparin levels and are therefore most commonly used in
monitoring heparin therapy. It is of no value in long-term management
of patients on oral Warfarin therapy. Thrombin clotting time (TCT) is the
measure of the time it takes for exogenously administered thrombin to
turn plasma fibrinogen into fibrin clot. It is extremely sensitive to levels
of heparin and is an excellent measure of measuring the level of
heparin-induced anticoagulation. The beauty of the TCT is that it is not
specific for any disease condition; thus it may be used to differentiate
factor deficiencies from the presence of heparin, or to separate lupus
anticoagulant from abnormalities in fibrinogen levels. The whole blood
activated clotting time (ACT) is a measurement of the ability of whole
blood to clot, and as such, is an available technique for monitoring
heparin levels intraoperatively. The ACT responds in a linear fashion to
increasing heparin dosage and correlates well with the observed clinical
anticoagulation. Adequate anticoagulation for extracorporeal circulation
is defined as an ACT of 480 seconds or more while for peripheral

76 of 167 11-Aug-15 10:57 PM


MedCosmos Surgery: MCQ : General Surgery http://medcosmossurgery.blogspot.com/search/label/MCQ : General Surgery

vascular applications, values of 250 seconds or greater are considered


appropriate.

153 Thrombolytic therapy has become a useful adjunct in the


management of peripheral arterial occlusion. In this setting, direct
intraarterial administration rather than intravenous has been advocated
to decrease the risk of systemic bleeding. Which of the following true
statement(s) concerning the use of thrombolytic agents for arterial
occlusion is/are true?
a. A standard technique involves infusing high-dose urokinase, 4000
units per minute for 1–2 hours, directly into the clot by a catheter
embedded in the thrombus
b. If progress is made, further fibrinolytic therapy is given at 1000 to
2000 units per minute until clot lysis has occurred
c. The usual infusion time by the above-stated technique is usually in
excess of 24 hours
d. Successful clot lysis occurs more frequently in arterial graft
occlusions than native arterial occlusions
e. The use of intraoperative thrombolytic therapy may be indicated for
situations where complete clot evacuation cannot be accomplished
surgically
Answer: a, b, e

The most popular method for intraarterial thrombolytic therapy for


arterial occlusion involves passing a guidewire through the thrombus
with arteriographic guidance and then infusing high-dose urokinase,
4000 units per minute for 1–2 hours, directly into the clot. If progress is
made, further fibrinolytic therapy is given at 1000 to 2000 units per
minute for a 6–12 hour period or until clot lysis has occurred. Using this
technique, mean infusion time in a recent study was found to be 18
hours and the incidence of bleeding complications was significantly
lessened. Selective intraarterial infusion of urokinase was associated
with complete clot resolution in 77% of native arterial occlusions versus
only 41% with arterial graft occlusion. After thrombolytic therapy has
reopened an occluded vessel or graft, radiologic or surgical correction
of the lesion responsible for the thrombosis in the first place must be
addressed for any hope of long-term success. The use of intraoperative
thrombolytic therapy is advocated in those situations where complete
clot resolution cannot be accomplished (such as following balloon
embolectomy for acute arterial occlusion) or when distal vasculative is
occluded and precludes appropriate inflow patency.

154 Which of the following statement(s) is/are true concerning


hemophilia A?
a. Hemophilia A is inherited as a sex-linked recessive deficiency of
factor VIII
b. A positive family history for bleeding disorders present in all patients
c. Laboratory tests reveal a prolongation of aPTT, prothrombin time
(PT), thrombin clotting time and platelet aggregation
d. Spontaneous bleeding is unusual with factor VIII levels greater than
10% of normal
Answer: a, d

77 of 167 11-Aug-15 10:57 PM


MedCosmos Surgery: MCQ : General Surgery http://medcosmossurgery.blogspot.com/search/label/MCQ : General Surgery

Hemophilia A is inherited as a sex-linked recessive deficiency of factor


VIII although 0% of cases are secondary to spontaneous mutation. The
incidence of this abnormality is approximately 1/10,000 births.
Laboratory screening tests usually reveal a prolongation of an aPTT but
normal prothrombin time (PT), thrombin clotting time (TCT) and
platelet aggregation testing. The minimum level of VIII required for
hemostasis is 30% for minor bleeding, whereas spontaneous bleeding is
unusual with factor levels greater than 5 to 10% of normal. In severe
genetic deficiency states however, factor levels as low as 1% have been
noted and patients are at risk for spontaneous bleeding.

155 Fibrinolytic therapy is based on activation of plasminogen, the


inactive proteolytic enzyme of plasma that binds to fibrin during the
formation of thrombosis. Activation of plasminogen to plasmin results in
selective thrombolysis at the fibrin clot surface. Which of the following
statement(s) is/are true concerning agents used in thrombolytic
therapy?
a. Streptokinase is a bacterial protein which is antigenic in humans,
resulting in allergic reactions in up to l5% of cases
b. Tissue plasminogen activator acts directly on plasmin without an
intermediate drug–plasmin complex
c. The half-life of urokinase, streptokinase, and TPA all exceed 30
minutes
d. Streptokinase is significantly cheaper than urokinase or TPA
Answer: a, b, d

Streptokinase is a bacterial protein produced by group C b-hemolytic


streptococci. It is therefore antigenic in humans and can be associated
with allergic reaction in between 2 and 18% of cases. In addition an
unusual serum sickness has been reported with streptokinase. Neither
urokinase or TPA which is now manufactured with recombinant DNA
technology are either associated with allergic side effects or
antigenicity. Streptokinase acts through a streptokinase-plasmin
complex, whereas urokinase and TPA act directly on plasmin without
intermediate drug plasmin complex. The level of the lytic state is
greatest with streptokinase, intermediate with urokinase, and least with
TPA with the half-lives ranging all less than 1/2 hour in duration.
Although the relative efficacy of the three agents has been compared in
a number of studies, there appears to be no significant benefit of one
agent over the other. Streptokinase however, is markedly less expensive
than either urokinase or TPA.

156 Von Willebrand’s disease is a common, congenital bleeding disorder.


Which of the following statement(s) is/are true concerning Von
Willebrand’s disease?
a. As in hemophilia, it is much more common in men
b. A history of spontaneous bleeding is common
c. Screening laboratory tests will include a prolonged aPTT with a
normal prothrombin time
d. Pre-treatment for elective surgery require administration of
cryoprecipitate to achieve levels of 23–50% of normal

78 of 167 11-Aug-15 10:57 PM


MedCosmos Surgery: MCQ : General Surgery http://medcosmossurgery.blogspot.com/search/label/MCQ : General Surgery

Answer: c, d

Von Willebrand’s factor is an adhesive protein that mediates platelet


adhesion to collagen. In addition, it protects and prevents the rapid
removal of factor VIII from blood. The classical deficiency state, Von
Willebrand’s disease, is caused by reduction of factor VIII activity
(although not as great as Hemophilia A) and the Von Willebrand factor.
Clinical manifestations include epistaxis, gingival bleeding,
menorrhagia, rare joint or muscle bleeding, and subcutaneous bleeding.
Spontaneous bleeding is not as common as in classic Hemophilia A. The
syndrome is transmitted as both autosomal dominant (heterozygous) and
autosomal recessive disease (homozygous) traits. Therefore there is no
sex predilection. Screening laboratory tests include a prolonged aPTT
with a normal prothrombin time. In addition, because of the importance
of this factor in platelet adhesion, patients display a prolonged bleeding
time and have decreased level of factor VIII activity, decreased
immunoreactive levels of Von Willebrand’s antigen, and abnormal
platelet aggregation responses to ristocetin. The most reliable source of
Von Willebrand’s factor is cryoprecipitate.

157 External pneumatic compression has been advocated for the


prevention of deep venous thrombosis during operative procedures.
Which of the following statement(s) concerning the use of external
pneumatic compression devices is/are true?
a. Intermittent pneumatic compression is as effective as low-dose
heparin in prevention of venous thrombosis
b. These devices function by compressing the lower extremities
therefore augmenting venous return
c. Pneumatic compression devices may also exhibit their antithrombotic
effect through stimulating local and systemic fibrinolysis
d. The length of time that intermittent pneumatic compression should
be used includes through the operation and for at least several days in
the postoperative period
Answer: b, c, d

In many well-controlled studies of venous prophylaxis, intermittent


pneumatic compression has been found to be as effective as low-dose
heparin therapy. In addition to augmentation of venous return with
these devices, local and systemic fibrinolysis appears to be stimulated.
Fibrinolytic activities are usually reduced for a 7–10 day period after an
operation. Studies have demonstrated that the pneumatic-compression
devices may exhibit their antithrombotic effect through prevention of
this fibrinolytic shutdown even when applied to the upper extremity.
The length of time that intermittent pneumatic compression should be
used has not been adequately determined but most data suggest that
devices should be used through the operation and for at least five days
in the face or prolonged immobilization.

158 The standard management oral anticoagulant therapy for chronic


treatment of venous thromboembolism is with the drug warfarin. Which
of the following statement(s) is/are true concerning the administration
of warfarin?

79 of 167 11-Aug-15 10:57 PM


MedCosmos Surgery: MCQ : General Surgery http://medcosmossurgery.blogspot.com/search/label/MCQ : General Surgery

a. An important complication of warfarin therapy is skin necrosis in


patients with protein C deficiency
b. Warfarin interferes with vitamin K dependent clotting factors II, VII,
IX, X
c. For effective anticoagulation the prothrombin time (PT) should be
kept at 2 control
d. It is recommended that warfarin be continued for at least one year
after initial episode of deep venous thrombosis
Answer: a, b

Warfarin interferes with the vitamin K dependent clotting factors II, VII,
IX and X, protein C, and protein S. An important complication of
warfarin is skin necrosis with patients both with and without protein C
deficiency. This syndrome usually involves full thickness skin slough over
fatty areas such as the breasts and buttocks. Warfarin therapy should be
monitored using the one stage prothrombin time (PT). The PT should be
kept at 1.3 to 1.4 control for effective anticoagulation. At higher levels,
there is a five-fold increase in the frequency of bleeding complications.
Two major complications of Warfarin therapy include recurrent
thrombosis and bleeding. It is recommended that Warfarin be continued
four months after an initial episode of deep venous thrombosis. Between
ten weeks and four to six months after deep vein thrombosis, there is a
recurrent thrombosis rate of 8.3 episodes per 1000 patient months.
Between four months and three years, recurrences fall to four episodes
per 1000 patient-months. At four months, the risks of bleeding
complications matches and exceeds the benefit from anticoagulant
therapy and thus is the basis for discontinuing warfarin administration at
this time.

159 Which of the following statement(s) is/are true concerning the


management of a patient with hemophilia A undergoing an elective
surgical operation?
a. Concentrates of factor VIII should be given several days prior to
elective surgery
b. The half-life of factor VIII concentrates is less than 24 hours
c. A dose of 40–50 IU/kg of factor VIII concentrate should be given prior
to the planned surgical procedure
d. Factor VIII concentration administration should be given for the first
24 hours after surgery but may then be stopped if no abnormal bleeding
has been observed
e. A new recombinant preparation of factor VIII offers the advantage of
being virus-free
Answer: b, c, e

Although the half-life of factor VIII is 2.9 days in normal individuals, the
half-life of factor VIII concentrates is 9 to l8 hours. Levels of 80% to
100% of normal should be obtained for surgical bleeding or
life-threatening hemorrhage. A dose of 40 to 50 IU/kg of factor VIII
should be given with half of this dose then administered every twelve
hours. After surgery, transfusion of factor VIII concentrates should be
continued for at least ten days. Unfortunately, past use of concentrates
of factor VIII obtained from donors has led to a high incidence of HIV

80 of 167 11-Aug-15 10:57 PM


MedCosmos Surgery: MCQ : General Surgery http://medcosmossurgery.blogspot.com/search/label/MCQ : General Surgery

infection in the hemophilia population. A new recombinant preparation


of factor VIII offers the advantage of being virus-free.

160 Transfusions of blood products can be associated with a number of


complications including immediate and delayed hemolytic reactions;
nonhemolytic reactions; infectious disease transmission; and
complications of massive transfusions. Which of the following
statements are true concerning complications of blood transfusions?
a. Immediate hemolytic transfusion reactions are caused by major ABO
blood group incompatibility
b. Nonhemolytic transfusion reactions are usually due to RH
incompatibility and are therefore more common in women of
childbearing age
c. The most common complication of massive blood transfusion is
dilutional thrombocytopenia
d. Routine impaired calcium supplementation is necessary during most
massive transfusion episodes
Answer: a, c

Immediate hemolytic reactions are usually caused by blood group ABO


incompatibility although they may be caused by antigens of other blood
group systems on the transfused red blood cells. The clinical
manifestations revolve around the antigen on the red blood cell stroma
and the antibody in the patient’s serum, and include production of
bradykinin, compliment activation, release of vasoactive agents from
platelets, and initiation of systemic clotting. Chills and fevers, chest
pain and lumbar pain, tachycardia and hypotension in the conscious
patient, and often diffuse bleeding in the anesthetized, unconscious
patient constitute this syndrome. Although reaction occurs immediately,
death related to the syndrome is uncommon, unless associated with a
transfusion of more than 100 ml of blood. Death usually occurs from
acute renal failure or hemorrhage due to DIC. Nonhemolytic reactions
occur with the frequency of 1 to 2% of all transfusions and consist
primarily of chills and fevers during the transfusion or in the first 2 to 3
hours after the transfusion is complete. Mechanism of these reactions
includes the presence of antibodies to white blood cell antigens in the
transfused blood, especially in the multitransfused or multiparous
patient. Massive transfusion complications relate to the rate and volume
of blood transfused. The most common complication is dilutional
thrombocytopenia. Factor deficiency of the labile factors V and VIII
rarely is of sufficient magnitude to result in problems with hemostasis.
For hypocalcemia to occur with massive transfusion, citrated blood must
be administered, one unit every five minutes. Routine empiric calcium
supplementation is unnecessary during most massive transfusion
episodes. Conversely, hypothermia is clearly a problem, especially when
associated with massive transfusion during complex intraoperative
procedures such as thoracoabdominal aneurysm resection.

161 A 67-year-old male with advanced cholangiocarcinoma develops


gram-negative sepsis. Excessive bleeding is noted around vascular
catheters and from needle puncture sites. The diagnosis of disseminated
intervascular coagulation (DIC) is considered. Which of the following

81 of 167 11-Aug-15 10:57 PM


MedCosmos Surgery: MCQ : General Surgery http://medcosmossurgery.blogspot.com/search/label/MCQ : General Surgery

laboratory test(s) is/are indicative of DIC?


a. Decreased platelet count
b. Decreased fibrinogen level
c. Normal prothrombin time
d. Elevated fibrin split products
Answer: a, b, d

Disseminated intravascular coagulation (DIC) is the primary form of


acute thrombosis. Causes of this syndrome include abruptio placenta,
gram-positive and gram-negative sepsis, endotoxemia, malignant
tumors, pelvic operations, certain snake bites, hematologic
malignancies, and hepatic failure. Blood coagulation is activated by the
release of tissue factor into the circulation, which activates factor VII of
the extrinsic pathway to VIIa, leading to massive thrombin production
and fibrin generation. This in turn activates the fibrinolytic system,
leading to bleeding in the later stages of the syndrome due to
consumption of coagulation factors, depletion of fibrinogen, and
unchecked plasma activities. Laboratory values in DIC usually include a
decline in the platelet count and fibrinogen level, along with an
elevation of fibrin split products.

162 Which of the following substances, not normally present in the


circulation, trigger the initiating events in the hemostatic process?
a. Thrombin
b. Platelet factor 3
c. Tissue factor
d. Collagen
Answer: c, d

The initiating agents for hemostasis involve two substances that are not
normally present in the circulation—collagen and tissue factor. Tissue
factor is released from injured cells, beginning the activation of the
extrinsic pathway of coagulation, while disruption of the protective
endothelial barrier of blood vessels exposes the underlying collagen to
the activation of platelets. In the bloodstream, tissue factor complexes
with factor VII which then activates factor X to factor Xa. At the same
time, activated platelets change from their discoid shape with their
procoagulant phospholipid (termed platelet factor 3) buried on the inner
side of the surface membrane to a spreading shape to allow for the
externalization of platelet factor 3 activity. Activated factor X,
activated factor V, ionized calcium and factor II (prothrombin) then
assemble on the platelet phospholipid surface to form the so-called
prothrombinase complex which catalyzes the formation of thrombin.

163 Bleeding complications are frequently associated with fibrinolytic


therapy. Which of the following statement(s) concerning complications
of fibrinolytic therapy is/are true?
a. Careful monitoring of prothrombin time and aPTT time are necessary
to avoid bleeding complications
b. A level of serum fibrinogen less than 100 mg/dl is associated with an
increased risk of bleeding
c. Recent (less than 10 days) major surgery is a contraindication to

82 of 167 11-Aug-15 10:57 PM


MedCosmos Surgery: MCQ : General Surgery http://medcosmossurgery.blogspot.com/search/label/MCQ : General Surgery

systemic but not regional fibrinolytic therapy


d. A patient with a cerebrovascular event occurring less than two
months ago can be treated with fibrinolytic therapy if head CT scan is
normal
Answer: b

Fibrinolytic therapy induces a hemostatic defect through a combination


of factors. Hypofibrinogenemia and fibrin degradation products inhibit
fibrin polymerization and, in combination with a decrease in the
clotting factors V and VIII, prolong the ability of blood to clot. However,
coagulation tests in general do not correlate well with bleeding
complications. A level of fibrinogen less than 100 mg/dl is associated
with an increased risk of bleeding. Absolute contraindications to
thrombolytic therapy include active internal bleeding, recent (less than
2 months) cerebral vascular accident, and documented left heart
thrombosis. Recent (less than 10 days) major surgery, obstetric delivery,
organ biopsy, or major trauma is considered a major relative
contraindication to either regional or systemic thrombolytic therapy.

164 Which of the following statement(s) is/are true concerning the


results of a National Institute of Health Consensus Conference on venous
thrombosis and low-dose heparin prophylaxis?
a. The odds of developing deep venous thrombosis with low-dose
heparin prophylaxis decreases by 67%
b. The risk of pulmonary embolism is decreased by almost 50%
c. There is no increase in mortality from other causes found in patients
treated with low-dose heparin
d. There was no difference in the incidence of bleeding complications
Answer: a, b, c

In a metaanalysis of 70 randomized trials in 16,000 patients comparing


low-dose heparin prophylaxis with standard therapy, the odds of
developing deep venous thrombosis with low-dose heparin prophylaxis
decreased 67%, whereas for pulmonary embolism (both fatal and
non-fatal), the odds decreased by 47%. For fatal pulmonary embolism,
the odds reduction was even greater (64%). No increase in mortality
from other causes was found in those patients treated with low-dose
heparin. Bleeding complications were more frequent in the heparin-
treated patients, with no difference between 5000 units twice daily and
5000 units three times daily. Similarly, the effectiveness of prophylaxis
was not influenced by either two or three times daily dosage.

165 Laboratory monitoring of coagulation and anticoagulation includes


testing of platelet function. Which of the following statements is/are
true concerning tests of platelet function?
a. A platelet count of 50,000/µL or more usually ensures hemostasis
b. Bleeding time assays assessibility of platelets to perform hemostatic
plugs and is determined from a sample of blood drawn in an EDTA
coated test tube
c. Aspirin therapy can be associated with a bleeding time in the range of
8–15 minutes
d. Tests of platelet aggregation should be part of the standard

83 of 167 11-Aug-15 10:57 PM


MedCosmos Surgery: MCQ : General Surgery http://medcosmossurgery.blogspot.com/search/label/MCQ : General Surgery

preoperative evaluation of patients using aspirin


Answer: a, c

Tests of platelet function include peripheral platelet counts, bleeding


times, and platelet aggregation. Usually, a platelet count of 50,000/mL
or more ensures adequate hemostasis, whereas counts less than
10,000/mL are dangerous and may lead to spontaneous bleeding.
Bleeding time performed by observing the clotting of blood induced with
a small needle stick, assesses the ability of platelets to perform
hemostatic plugs and are usually shorter than eight minutes. A bleeding
time between 8 and 15 minutes most often reflects a low plasma level
of Von Willebrand’s Factor or the use of antiplatelet drugs. A bleeding
time greater than 15 minutes is clearly prolonged and indicates severe
platelet functional impairment. Platelet aggregation studies involve the
use of a number of different agonists. Although a relatively
straightforward technique, platelet aggregation is not available in most
laboratories, probably because of the observer-dependent nature of the
test.

166 As thrombin generation proceeds, the body has natural


anticoagulant systems opposing further thrombus formation. Natural
anticoagulants include:
a. Tissue plasminogen activator (TPA)
b. Antithrombin III
c. Activated protein C
d. Heparin cofactor II
Answer: b, c, d

Just as thrombin generation is the key to coagulation, antithrombin III is


the most central anticoagulant proteins. This glycoprotein binds to
thrombin, preventing its removal of fibrinoprotein A and B from
fibrinogen, prevents the activation of factor V and VIII and the
activation and aggregation of platelets. The second line of defense is
the activated protein C, which inactivates factors Va and VIIIa. This
inactivation reduces the ability of the prothrombinase complex to
accelerate the rate of thrombin formation. A third natural anticoagulant
is heparin cofactor II. Its concentration in plasma is estimated to be
some four-fold lower than antithrombin III, and its action is primarily
implicated in the regulation of thrombin formation in extravascular
tissues. Tissue plasminogen activator (TPA) is a natural catalyst for the
activation of plasminogen to plasmin, the main fibrinolytic enzyme in
the body. Therefore, TPA is part of the fibrinolytic system rather than a
natural anticoagulant.

167 Infectious disease transmission during blood transfusions is of


clinical significance to surgeons and of major importance to patients
contemplating surgery potentially associated with the need for blood
administration. Which of the following statement(s) is/are true
concerning the transmission of infectious disease during blood
transfusions?
a. Post-transfusion hepatitis is usually due to hepatitis B
b. Hepatitis and HIV transmission is greatest with the administration of

84 of 167 11-Aug-15 10:57 PM


MedCosmos Surgery: MCQ : General Surgery http://medcosmossurgery.blogspot.com/search/label/MCQ : General Surgery

pooled plasma products such as serum albumin


c. The most important cause of post-transfusion disease in
immunosuppressed patients is CMV infection
d. The risk of HIV transmission in blood transfusions is significantly less
than the risk of hepatitis transmission
Answer: c, d

The most common infectious diseases transmitted during blood


transfusions include viral hepatitis, CMV, and HIV infection.
Post-transfusion hepatitis in 90% of cases consists of non-A, non-B
hepatitis known as hepatitis C. All blood products except for immune
serum globulin and albumin can carry and transmit this form of
hepatitis. Because heat treatment eliminates the risk of viral
transmission, products from pooled plasma that are heat treated such as
albumin are not at risk for HIV or hepatitis transmission. CMV
transmission exists in three forms—primary, reinfection, and
reactivation. Primary exposure results in an IgM response to the virus.
Reactivation is most commonly related to pregnancy, transplantation,
and immunosuppression, and is the most important cause of
post-transfusion disease accompanying immunosuppression of patients.
Although the risk of the public concern for transmission of HIV disease
associated with blood transfusions has significantly outweighed other
infectious disease transmission, the risks of HIV transmission is markedly
less than that of hepatitis.

168 There are a number of hypercoaguable states which can be


associated with arterial or venous thrombosis and embolic phenomenon.
These include:
a. Heparin-associated thrombocytopenia
b. Antithrombin III deficiency
c. Von Willebrand disease
d. Vitamin C deficiency
Answer: a, b

A number of hypercoaguable states are present. These include heparin-


associated thrombocytopenia in which a heparin-dependent platelet
antibody causes aggregation of platelets when the patient is exposed to
heparin. Activation of platelets in this setting results in
thrombocytopenia, thrombosis, and embolic episodes. Antithrombin III
deficiency accounts for about 2% of venous thrombotic events and has
been described in pulmonary embolism, mesenteric venous thrombosis,
lower extremity venous thrombosis, arterial thrombosis, and dialysis
fistula failure. Von Willebrand’s disease is a hereditary complex
coagulation factor deficiency which is manifested by a reduction of
factor VIII activity, and the Von Willebrand factor which is an adhesive
protein that mediates platelet adhesion to collagen. Severe vitamin C
deficiency results in a disorder in soft tissue increasing vascular
permeability and fragility resulting in the potential for bleeding
disorders.

169 Cytokines with clearly defined actions in acute inflammation and


early tissue injury include which of the following?

85 of 167 11-Aug-15 10:57 PM


MedCosmos Surgery: MCQ : General Surgery http://medcosmossurgery.blogspot.com/search/label/MCQ : General Surgery

a. Cysteine-X-Cysteine (C-X-C) chemokines


b. Tumor Necrosis Factor (TNFa)
c. Transforming Growth Factor-b (TGF-b)
d. Interleukin-6 (IL-6)
e. Platelet Derived Growth Factor (PDGF)
Answer: a, b, c, d, e

Polypeptide mediators, such as TNFa and IL-1, are considered “early


response” cytokines and are actively involved in the initiation of the
cascade of events which precipitate acute inflammation. In addition to
being important triggers for the induction of other cytokines important
inflammatory network, TNFa and IL-1 appear to be key mediators in
promoting the adherence of inflammatory cells to the endothelium. IL-1
is a complex, multifunctional molecule that shares many overlapping
biological properties with TNFa. In addition, both IL-1 and TNFa
potentiate the effects of one another. The most important function of
IL-6 appears to be the regulation of the hepatic acute phase response.
Following injury, a number of physiologic changes develop within several
hours. IL-6 is one of the primary stimuli for the production of acute
phase proteins from the liver. Endotoxin, IL-1, TNFa and PDGF are
capable of causing significant induction of IL-6 synthesis.
Over the last decade, at least 12 different C-X-C chemokines have been
identified. These include IL-8, one of the most potent mediators of
chemotaxis known. TNFa and IL-1 are key molecules for the induction of
IL-8, which in turn is important for the induction of neutrophil
recruitment and activation.
Similar properties are apparent for other members of this chemokine
family.
Platelet activation and degranulation occur during coagulation following
injury, leading to the deposition of a number of cytokines into the
provisional matrix. These cytokines include transforming growth
factor-a, (TGFa), transforming growth factor b (TGF-b), platelet-derived
growth factor (PDGF), and neutrophil activating peptide-2 (NAP-2).
These cytokines are either important growth factors or chemotaxis for
leukocytes, endothelial cells, fibroblasts, and keratinocytes which are
key components in the process of tissue repair. Thus, coagulation and
platelet activation provide the initial foundation for subsequent cellular
recruitment.

170 Which of the following statements regarding transforming growth


factor b (TGF-b) are true?
a. TGF-b expression is autoregulated
b. TGF-b enhances collagen synthesis
c. TGF-b inhibits extracellular matrix production
d. TGF-b may inhibit or promote cellular proliferation
Answer: a, b, d

TGF-b appears to be one of the key cytokines in control of tissue repair.


TGF-b is strongly chemotactic for neutrophils, T cells, monocytes, and
fibroblasts. TGF-b activates inflammatory cells to elaborate fibroblast
growth factor, TNFa, IL-1 and increase their synthesis of extracellular
matrix proteins. TGF-b also induces both the infiltrating cells and

86 of 167 11-Aug-15 10:57 PM


MedCosmos Surgery: MCQ : General Surgery http://medcosmossurgery.blogspot.com/search/label/MCQ : General Surgery

resident cells to produce more TGF-b. This auto-induction amplifies its


biological effects at the site of injury and may play an important role in
the development of chronic fibrosis in a variety of pathologic states.
TGF-b enhances collagen synthesis as well. Lastly, TGF-b may function
as a mitogen or growth inhibitor for a wide variety of cell types,
including selected cell types of mesenchymal origin. Whether TGF-b
stimulates or inhibits proliferation depends on the presence of other
growth factors, the concentration of TGF-b, and the cell density. Thus,
at low doses, TGF-b stimulates the proliferation of densely plated
human marrow fibroblasts, but is inhibitory at high concentrations.

171 Leukocyte activation and adhesion to vascular endothelial cells is a


critical step in the inflammatory process. This process is regulated by
which of the following molecules?
a. The selectins
b. The b5 integrins
c. The immunoglobulin supergene family
d. Nitric oxide
e. IL-8
Answer: a, c, d, e

The temporal events that initiate and propagate neutrophil recruitment


and inflammation include endothelial cell activation and expression of
endothelial-derived neutrophil adhesion molecules, neutrophil-
endothelial cell adherence, and neutrophil transendothelial migration
via established neutrophil chemotactic gradients. There are three major
families of adhesion molecules which are expressed on the surface of
leukocytes and endothelial cells and are important for leukocyte-
endothelial cell interactions. These include the immunoglobin
supergene family (ICAM-1, VCAM-1, and PECAM-1), the selectins
(E-selectin, P-selectin and L-selectin), and the integrins. The leukocyte
b2 integrin adhesion molecule family consists of three members with
heterodimeric glycoproteins displayed as a variable alpha and a constant
beta chain. Nitric oxide regulates the adhesion process both by direct
influence on leukocyte binding as well as by regulation of regional blood
flow. IL-8 is one of the most potent mediators of chemotaxis in the
C-X-C chemokine family. It serves an important role in neutrophil
recruitment and activation, and the continued propagation of the
inflammatory response.

172 A 65-year old patient has colon carcinoma metastatic to the liver
and lungs. He has had a weight loss of 10 kg. Cytokine-dependent tumor
cachexia is attributable to which of the following?
a. Increased glucose uptake and increased glycogen breakdown occur in
this circumstance.
b. Suppressed activity of lipoprotein lipase results from TNFa
c. TNFa stimulates lipolysis
d. The differentiation process of pre-adipocytes is impaired
e. Partial reversal of differentiated adipocytes to pre-adipocyte
morphology and gene expression occurs
Answer: a, b, c, d, e

87 of 167 11-Aug-15 10:57 PM


MedCosmos Surgery: MCQ : General Surgery http://medcosmossurgery.blogspot.com/search/label/MCQ : General Surgery

Tumor cachexia appears to be mediated by TNFa. Lipopolysaccharide


(LPS), as well as other cytokines, activate a variety of inflammatory
cells, most importantly macrophages, to produce TNFa. Both the chronic
administration of TNFa to rats and implantation of tumors secreting
TNFa in mice induce a syndrome of cachexia. In vitro, higher TNFa
concentrations alter glucose metabolism in cultured myotubules by
increasing glucose uptake and glycogen breakdown. It has also been
demonstrated that purified TNFa suppresses lipoprotein lipase activity
and stimulates lipolysis in cultured adipocytes. Further, TNFa not only
inhibits the differentiation process of preadipocytes, but partially
reverses differentiated adipocytes to a preadipocyte morphology and
pattern of gene expression. All of these metabolic effects at least
partially explain the chronic syndromes of anorexia, weight loss, and
cachexia that are associated with both chronic infection and
malignancy.

173 Which of the following statements regarding fibroblasts and their


function in wound healing are true?
a. IL-1 has both inhibitory and promotional effects on fibroblast growth
b. TNFa stimulates fibroblast collagen synthesis
c. IL-1 and TNFa have opposite effects on the healing of bone
d. In human clinical trials, EGF (epithelial growth factor) has been
demonstrated to accelerate epidermal regeneration in cutaneous
wounds
Answer: a, d

IL-1 appears to be important in the process of normal wound repair. IL-1


has been shown to stimulate skin fibroblast and keratinocyte growth, as
well as fibroblast collagen synthesis and keratinocyte chemotaxis. IL-1
also promotes increased transcription of the matrix degradative
enzymes collagenase and stromelysin. These are important and potent
tissue degrading proteinases. Other studies have demonstrated that IL-1
inhibits fibroblast growth and matrix synthesis, and stimulates
collagenase production. These actions are at least partly due to the
ability of IL-1 to upregulate prostaglandin E2 production which results in
the down regulation of matrix synthesis. IL-1 has both promoting and
inhibiting effects on fibroblast collagen synthesis, therefore, the overall
activity in this area is somewhat unclear in comparison to other
well-defined fibroblast growth-promoting cytokines. TNFa inhibits
fibroblast collagen synthesis, however it also has potent mitogenic
effects. The mitogenic response correlates well with an increased
stimulation of tyrosine phosphorylation. Both IL-1 and TNFa have similar
effects upon bone. Both stimulate cartilage resorption, the release of
proteoglycans from cartilage by limited proteolytic degradation, and
both inhibit proteoglycan synthesis. Recent studies have also
demonstrated that TNFa inhibits fracture healing in experimental
animals. This is due to the inhibition of cartilage formation and new
bone synthesis, and the inhibition of mesenchymal cell differentiation
into chondroblasts. The family of epithelial growth factor (EGF)-like
molecules induce mitogenesis and play a role in wound healing. In
human clinical trials, EGF has been demonstrated to accelerate
epidermal regeneration in cutaneous wounds. In vitro data show that

88 of 167 11-Aug-15 10:57 PM


MedCosmos Surgery: MCQ : General Surgery http://medcosmossurgery.blogspot.com/search/label/MCQ : General Surgery

recombinant EGF enhances keratinocyte migration. EGF is also a potent


chemoattractant for granulation tissue fibroblasts.

174 Neutrophil chemotaxis is a fundamental aspect of inflammatory


injury in conditions such as the Adult Respiratory Distress Syndrome
(ARDS). Neutrophil chemotaxis is directly attributable to which of the
following molecules?
a. C5a
b. TNFa
c. LPS
d. IL-1
e. ENA-78 (Epithelial Neutrophil Activating Protein)
Answer: a, e

There is a large collection of peptide, polypeptide and lipid mediators


which have chemotactic properties. Although TNF a, IL-1 and LPS were
initially reported to have direct neutrophil chemotactic activity, recent
studies have demonstrated that these molecules are not directly
chemotactic for neutrophils. This finding suggests that cytokine
networks may be operative in vivo and depend on the initial expression
of early response cytokines. This initial interaction is followed by the
generation of more distal inflammatory mediators that directly
influence neutrophil chemotaxis and activation. There is a particularly
important group of novel chemotactic cytokines which share significant
homology with the presence of four conserved cysteine amino acid
residues. These cytokines in their monomeric forms are all less than 10
kD, are characteristically basic heparin-binding proteins, have specific
neutrophil chemotactic activity and display four highly conserved
cysteine amino acid residues, with the first two cysteines separated by
one non-conserved amino acid residue. Because of their chemotactic
properties and the presence of C-X-C cysteine motif, these have been
designated the C-X-C chemokine family. Twelve different chemokines
have been identified in the last decade. These include IL-8, epithelial
neutrophil activating protein (ENA-78), and others. Among the other
endogenous chemoattractants are several complement-derived
peptides. Perhaps, the most potent of these is the short-lived C5a
peptide.

175 Which of the following statements regarding angiogenesis are true?


a. Angiogenesis is a seminal biologic event with clinical relevance
limited to its effect upon tumor growth
b. C-X-C chemokines regulate angiogenesis
c. PF-4 has angiogenic properties
d. Sites of atherosclerosis demonstrate chronic angiogenic activity
Answer: b, d

An important component of tissue repair and wound healing is the


process of angiogenesis. This normal, physiologic process is a local,
transient event which is regulated strictly. A biological imbalance in the
production of angiogenic and angiostatic factors contributes to the
pathogenesis of several angiogenesis-dependent disorders. These
include both malignant and nonmalignant disorders such as rheumatoid

89 of 167 11-Aug-15 10:57 PM


MedCosmos Surgery: MCQ : General Surgery http://medcosmossurgery.blogspot.com/search/label/MCQ : General Surgery

arthritis, scleroderma, psoriasis, atherosclerosis, and idiopathic


pulmonary fibrosis. Persistent neovascularization in these benign
disorders is a prerequisite for the perpetuation of fibroproliferation.
IL-8 and potentially other C-X-C chemokines are involved with the
angiogenesis process. IL-8 is a potent angiogenic factor. In contrast,
another member of the C-X-C chemokine family, PF-4 has angiostatic
properties. This suggests that the C-X-C chemokines may function as
either angiostatic or angiogenic factors, and the biologic balance that is
maintained between these factors may govern overall angiogenic
potential in a variety of physiological and pathophysiological states.

176 Which of the following statements regarding IL-1 are correct?


a. While IL-1 and TNFa share many biologic effects, IL-1 appears to be
more potent
b. IL-1 expression is in part autoregulated
c. IL-1 inhibits prostaglandin production
d. The ability of IL-1 to upregulate endothelial cell-neutrophil adhesion
molecules is relatively limited
Answer: b

IL-1 and TNFa share many biologic properties. In addition, each


potentiates the effects of the other one when given concurrently.
Overall, IL-1 alone probably has weaker effects than TNFa with respect
to the induction of shock; its role is likely to be important with respect
to its marked potentiating abilities as it relates to TNFa. IL-1 expression
is regulated by a host of factors including IL-2, granulocyte macrophage
colony stimulating factor (GM-CSF), transforming growth factor b
(TGF-b), TNFa, all of the interferons, and IL-1 itself. Other endogenous
stimuli for IL-1 production include antigen-antibody complex, the Fc
region of IgG, and C5a; other nonspecific exogenous stimuli include
silica particles and UV irradiation.
One of the key proinflammatory features of IL-1-induced inflammation is
the stimulation of arachadonic acid metabolism. IL-1 stimulates the
release of pituitary stress hormones and increases the synthesis of
collagenases, resulting in the destruction of cartilage, bone and other
collagen-rich structures. IL-1 stimulates prostaglandin production.
One of the most important properties of IL-1 involves its interaction
with the vascular endothelium. This includes the adherence of
neutrophils, basophils, eosinophils, monocytes, and lymphocytes to the
vascular endothelium via interaction between adhesion molecules on
leukocytes and adhesion-receptor complex on the endothelial cells. By
inducing the expression of ICAM-1, E-selectin, and VCAM-1 on
endothelial cells, IL-1 provides a key step in the extravasation of
leukocytes to sites of local inflammation and injury.

177 Which of the following statements regarding TNFa are true?


a. TNFa has a marked procoagulant effect
b. Passive immunization of patients with neutralizing antibodies to TNFa
improves survival from multi-organ system failure
c. TNFa upregulates E-selectin expression
d. The most potent known stimulus for TNFa production and release is
IL-1

90 of 167 11-Aug-15 10:57 PM


MedCosmos Surgery: MCQ : General Surgery http://medcosmossurgery.blogspot.com/search/label/MCQ : General Surgery

Answer: a, c

TNFa has a marked procoagulant effect on endothelial cells,


precipitating intravascular thrombosis. TNFa causes endothelial cells to
release procoagulant activity (tissue factor), platelet activating factor,
and von Willebrand factor, all of which favor thrombosis. TNFa also
down regulates the expression of thrombomodulin, which has the
potential to block the assembly of protein C and protein S complexes,
further decreasing the anticoagulant properties of the endothelial cell
surfaces.
Administration of recombinant TNFa to experimental animals produces a
clinical syndrome similar to that seen in septic shock and multi-organ
system failure in humans. Passive immunization of animals with
neutralizing antibodies against TNFa, prior to the infusion of TNFa or
endotoxin, has been shown to prevent the development of this
syndrome. No such evidence exists in human patients.
TNFa upregulates a variety of leukocytic adhesion molecules including
ICAM-1, PECAM-1, VCAM-1, E-selectin and P-selectin. A variety of
exogenous and endogenous factors (including IL-1) are capable of
inducing cells to produce TNFa, however the most potent stimulus for
TNFa production and release is endotoxin.

178 Which of the following belong to the family of C-X-C chemokines?


a. IL-8
b. IL-10
c. Growth Related Oncogene-a
d. Leukotreine B4
e. b Thromboglobulin
Answer: a, c, e

A particularly important group of novel chemotactic cytokines has been


elucidated over the last decade. Twelve are known and are listed below.
C-X-C Chemokines
Connective Tissue Activating Protein III
b-Thromboglobulin
Growth Related Oncogene-a
Growth Related Oncogene-b
Related Oncogene-g
Interleukin-8
Epithelial Neutrophil Activating Protein
Granulocyte Chemotactic Protein-2
Platelet Factor-4
g-Interferon-inducible Protein
Monokine-induced by g-Interferon
Each has unique biologic functions. There appear to be important in vivo
cytokine networks involving these molecules which regulate chemotaxis,
and other fundamental aspects of inflammation.

179 Which of the following statements regarding the complement


system are true?
a. Complement activation yields products which are directly cytotoxic
as well as products which act indirectly via activated leukocytes

91 of 167 11-Aug-15 10:57 PM


MedCosmos Surgery: MCQ : General Surgery http://medcosmossurgery.blogspot.com/search/label/MCQ : General Surgery

b. Complement products referred to as anaphylatoxins include C1, C3a,


C4a, and C5a
c. The principal role of C5a is in bacterial opsonization
d. The alternative and classical pathways converge proximal to
generating the membrane attack complex (C5b-9)
Answer: a, d

The complement system is composed of two different but linked


sequences, the classic and alternative pathways. The pathways involve
serum proteins that act to amplify the inflammatory-immune response
as well as to directly mediate tissue injury. Complement activation by
either pathway has been associated with a cascade of events, some of
which are mediated directly at a physiologic level by complement
products and some of which occur indirectly via activated leukocytes.
The direct physiologic effects mediated by C3a and C5a, and to a lesser
extent C4a, include increased vascular permeability and contraction of
smooth muscle. These are key elements of anaphylaxis. C1 is not an
anaphylatoxin as it is the initial complement component which binds to
antigen-antibody complex to initiate classical pathway activation. C5a
acts principally to alter the behavioral characteristics of leukocytes.
Effects include enhanced adherence, enhanced chemotactic activity,
release of proteinases, and production of toxic metabolites of oxygen.
C3, on the other hand, plays a key role in bacterial opsonization,
resulting in enhanced phagocytosis of invading microorganisms. The
alternative and classical complement pathways converge at the C5 level
proximal to generating the membrane attack complex (C5b-9) (Figure
6-3).

180 Which of the following statements regarding neutrophils are true?


a. The neutrophil undergoes final maturation after release into the
circulation
b. Patients with chronic granulomatous disease have a defective
neutrophil H-oxidase system
c. Neutrophil killing of bacteria is achieved by oxidants, proteinases and
cationic proteins
d. The normal human neutrophil circulates in the blood for 7–10 days
Answer: b, c

The neutrophil is a migratory phagocytic cell that defends the host


against bacteria and eliminates necrotic tissue. The neutrophil matures
in the bone marrow and is released into the circulation as a fully
differentiated cell. It is loaded with granules containing a variety of
proteinases, hydrolases, antimicrobial agents and cationic proteins. The
cell phagocytoses material and the granules fuse with the phagocytic
vacuoles to degrade the foreign material. When the cells are challenged
with a large amount of material, the granule contents may be released
into the extracellular space where damage to surrounding tissue occurs.
The neutrophil normally circulates in the human bloodstream for 7 to 10
hours. Thereafter, neutrophils are thought to exist for 1 to 2 days in the
tissues before being cleared from the system. Granule constituents are
formed during differentiation and replenishment of spent granules does
not occur once the cells are in the circulation. Hence, the neutrophil is

92 of 167 11-Aug-15 10:57 PM


MedCosmos Surgery: MCQ : General Surgery http://medcosmossurgery.blogspot.com/search/label/MCQ : General Surgery

a fully differentiated end-cell poised to respond rapidly to stimuli, but it


is rapidly spent in the process. Neutrophils have a NADPH-oxidase
enzyme system on the plasma membrane which can be activated to
produce toxic oxygen species including the superoxide anion (02–).
Patients with chronic granulomatous disease (CGD) have a defective
NADPH-oxidase system in their neutrophils, and are thus unable to
generate 02–. Although neutrophils from patients with CGD are able to
phagocytose bacteria, they are unable to kill the intracellular microbes
and chronic, unresolved infections result.

181 Which of the following statements regarding the alternative


complement pathway are true?
a. C1, C4 and C2 are involved
b. NH3 apparently activates complement via this pathway
c. Factors B and D are involved
d. Endotoxin activates complement via the alternative pathway
Answer: b, c, d

The alternative pathway differs from the classic pathway in that the
first steps involving C1, C4 and C2 are bypassed. (See Figure 6-3
previously reproduced.) This pathway can be directly activated by
agents other than antigen–antibody complex (e.g., complex
polysaccharides like endotoxin and zymosan). Other serum protein
factors (e.g., factors B and D) are involved in the activation sequence.
Ammonia can attack the thiol-ester, producing amidated C3 and activate
the alternative pathway. This leads to membrane attack complex
formation (C5b-9) and activation of a number of phagocytic cell
functions including toxic oxidant production. This phenomenon may
have relevance to several in vivo disease states. In animal models of
renal failure, elevated levels of renal vein NH3 have been correlated
with impaired renal function and the presence of complement
components at the sites of renal injury.

182 Platelet activating factor is:


a. Generated by the action of phospholipase A2 on membrane
phospholipids
b. Antiinflammatory in most of its actions
c. Synthesized by endothelial and other cells
d. Exerts a variety of biologic effects which are platelet-independent
Answer: a, c, d

Like the eicosanoids, platelet-activating factor (PAF) is not stored in


cells but is rapidly produced during inflammation. PAF exerts a variety
of biologic effects that are platelet-independent. The synthesis of PAF is
initiated by the activation of phospholipase A2. Activation of
phospholipase A2 releases arachidonic acid in addition to lyso-PAF.
Hence, PAF synthesis and eicosanoid production are coordinately
regulated. PAF is synthesized on activation of a variety of inflammatory
cells including platelets, neutrophils, basophils, mast cells, mononuclear
phagocytes, eosinophils and vascular endothelium. PAF is a stimulatory
agonist for many inflammatory cells, as well as for smooth muscle cells,
vascular endothelium and others. PAF enhances the ability of

93 of 167 11-Aug-15 10:57 PM


MedCosmos Surgery: MCQ : General Surgery http://medcosmossurgery.blogspot.com/search/label/MCQ : General Surgery

neutrophils to respond to challenge with N-formylpeptides and LTB4.


There is considerable overlap and redundancy in the effects produced
by PAF and eicosanoids.

183 Platelets have a wide array of functions in inflammation. Which of


the following are among these?
a. Synthesis and release of vasoactive eicosanoids
b. Release of chemotactic factors
c. Adherence to and coating of bacterial and tumor cells
d. Increase of vascular permeability
e. Phagocytosis of bacteria
Answer: a, b, c, d

Platelets are anucleated cells derived from megakaryocytes in the bone


marrow. Their central role in hemostasis is well known. Platelets possess
a wide array of functions in inflammation, including the following:
Synthesis and release of vasoactive eicosanoids
Release of chemotactic factors
Interaction with other inflammatory cells
Interaction with endothelial cells
Adherence to and coating of bacterial and tumor cells
Platelets are not capable of phagocytosis.
Few of the factors released or the functions carried by platelets during
inflammation are unique to this cell type. Other inflammatory cells
often have the same or similar capabilities. Indeed, some platelet
functions may reflect vestigial functions inherited from a primitive
precursor inflammatory cell. Platelets serve primarily as an amplifier or
modulator of the inflammatory response.

184 Eicosanoids mediate inflammation in a variety of ways. Of the


following statements, which are true with regard to this?
a. Eicosanoids are stored in cytoplasmic granules for release after
receptor mediated signaling
b. Eicosanoids include prostaglandins, thromboxanes, leukotrienes and
lipoxins
c. Eicosanoids generally have a plasma half-life measured in hours
d. Physiologic responses to eicosanoids include vasodilatation,
vasoconstriction, increased vascular permeability and both chemotaxis
and chemoattractant inhibition
Answer: b, d

The eicosanoids are derived from arachidonic acid (eicosatetraenoic


acid) and consist of prostaglandins, thromboxanes, leukotrienes and
lipoxins. The eicosanoids are not stored in cells but are rapidly
synthesized by cells in response to a variety of stimuli. They have
potent effects on vascular and bronchial smooth muscle including
vasodilatation, vasoconstriction, bronchodilation and
bronchoconstriction. In addition, they directly regulate vascular
permeability. LTB4 is a potent, neutrophil chemoattractant whereas
lipoxin A4 inhibits other chemoattractants. It appears that eicosanoids
are important regulators of the endogenous inflammatory response. The
rapid destruction of eicosanoids in the circulation limits their role

94 of 167 11-Aug-15 10:57 PM


MedCosmos Surgery: MCQ : General Surgery http://medcosmossurgery.blogspot.com/search/label/MCQ : General Surgery

primarily to that of mediators of local inflammatory changes. The local


effects can be substantial. In general, the eicosanoids are rapidly
metabolized or are so chemically unstable that they primarily exert
their effects near the site of synthesis. Arachidonic acid does not exist
in cells but is esterified to membrane phospholipids. Thus, the first step
in the production of eicosanoids is phospholipase action, which liberates
arachidonic acid. (Figure 6-6)

185 Which of the following statements are true?


a. Eosinophils are the major, if not sole, source of histamine in the
blood
b. Basophils are effector cells in allergic reactions by virtue of IgE
receptors
c. Mast cells are the major source of tissue histamine except in the
stomach and central nervous system
d. Mononuclear phagocytes release a variety of proinflammatory
cytokines and growth factors
Answer: b, c, d

Eosinophils constitute 1% to 3% of the leukocyte population of the


bloodstream. They also reside in tissues and they exhibit phagocytic
capabilities. They are less effective as bactericidal cells than
neutrophils, but play a major role in defense against parasites.
Eosinophils are primary effectors in allergic reactions by virtue of IgE
receptors (which are not found on neutrophils).
Basophils are fully differentiated cells released into the circulation from
bone marrow. Basophils are the major, if not sole, source of histamine in
the blood. Histamine is a vasoactive amine and the major mediator of
the IgE-mediated immediate hypersensitivity response. Histamine
release from basophils is induced by complement products as well as by
IgE receptors.
Mast cells are formed from bone marrow precursors that differentiate
and proliferate in connective tissue. Mast cell granules contain
histamine and proteoglycans. They represent the major source of
histamine in most tissues except the stomach and central nervous
system.
The monocyte–macrophage system consists of phagocytic cells scattered
throughout the body. During acute inflammation, monocytes respond to
chemoattractants released and are recruited to the site of
inflammation. Mononuclear phagocytes respond to inflammatory stimuli
by releasing M-CSF, GM-CSF, IL-1, and TNF, in addition to a variety of
growth factors. These factors increase the production of mononuclear
phagocytes and several of these factors enhance the ability of effector
cells to respond to chemotactic stimuli released at the site of injury.
Thus, the mononuclear phagocytes are important in initiating and
augmenting the cycle of events that result in recruitment and activation
of inflammatory cells at sites of inflammation.

186 Cellular injury from oxidants may be manifest by which of the


following?
a. Cell membrane lipid peroxidation
b. DNA strand breaks

95 of 167 11-Aug-15 10:57 PM


MedCosmos Surgery: MCQ : General Surgery http://medcosmossurgery.blogspot.com/search/label/MCQ : General Surgery

c. Cytoskeletal disassembly
d. ATP depletion
Answer: a, b, c, d

Free oxygen radicals are chemical species that are intermediates in the
normal process of cellular respiration. Oxidants that are free radicals
have been implicated as initiators of reactions which lead to a variety of
cellular injuries. Oxidants are derived from several sources, notably
phagocytes. Among the effects of oxygen free radicals are membrane
lipid peroxidation, DNA strand breaks, cytoskeletal disassembly and
inhibition of glucose metabolism leading to decreased cellular ATP
concentrations. (Figure 6-16)

187 Which of the following acute-phase protein levels are increased in


human plasma following acute inflammation?
a. C-reactive protein
b. Serum amyloid
c. a -Proteinase inhibitor
d. Fibrinogen
e. Albumin
Answer: a, b, c, d

The acute-phase response is a series of homeostatic responses of the


organism to tissue injury in infection and inflammation. After an
inflammatory stimulus occurs, a number of events occur within hours.
These reflect altered set-points for various physiologic parameters
including thermoregulation (fever), nitrogen balance (negative), and
levels of various plasma proteins (increased or decreased). The
erythrocyte sedimentation rate, which increases with inflammatory
states, is an example of this phenomenon. The increased sedimentation
rate is due to increased levels of fibrinogen and some of the other
acute-phase reactants in plasma. Some proteins show a large increase
(about 1000-fold), some a 4-to 5-fold increase, and others about a 50%
increase over resting nonstressed levels.
Note that albumin is an acute-phase reactant. Levels of albumin drop
after an inflammatory stimulus, usually 30% to 50% of the level before
injury. The reason for the decrease in production is poorly understood.

188 Which of the following statements regarding endothelial cells in


acute inflammation are true?
a. Endothelial cells are characterized by phenotypic homogeneity
b. Specific patterns of receptor expression regulate leukocyte
adherence
c. Endothelial cell nitric oxide generation regulates regional blood flow
and leukocyte adhesion
d. Endothelial cells may be capable of phagocytosis
Answer: b, c, d

Endothelial cells are increasingly recognized to be phenotypically


heterogeneous. Specific receptor molecules are expressed at various
sites where they help to direct lymphocytes and other leukocytes to
their appropriate target organ. In the high endothelial venues, these

96 of 167 11-Aug-15 10:57 PM


MedCosmos Surgery: MCQ : General Surgery http://medcosmossurgery.blogspot.com/search/label/MCQ : General Surgery

receptor molecules are known as vascular addressing. Endothelial cells


play a major role in regulating vascular tone. This is the result of
angiotensin-converting enzyme on the cell surface as well as the
production of both endothelia (a potent vasoconstrictor) and nitric
oxide (a potent vasodilator). Both play important physiologic roles in
determining the distribution of blood flow. In addition, recent evidence
suggests that NO may have direct effects upon the expression of a
variety of leukocyte adhesion molecules. Under unusual circumstances,
endothelial cells can exhibit macrophage-like properties in that they
can act as antigen-presenting cells and also phagocytose particles. They
may also be a significant source of oxidants in inflammatory reactions
after ischemic injury. Endothelial cells are not passive participants in
inflammatory processes; rather, they possess the ability to direct and
focus many aspects of an inflammatory event.

189 The first line of host defense is the barrier presented to the
external environment. Which of the following statement(s) is/are true
concerning host barriers?
a. Sebaceous glands secrete chemical compounds that maintain a
relatively high pH, providing effective bacterial stasis
b. Within the respiratory tract, ciliary function serves to extrude
microorganisms trapped within the mucus secretion layer
c. The low pH within the stomach markedly decreases bacterial content
of the upper gastrointestinal tract
d. Gut peristalsis serves to prevent microbial adherence and invasion
Answer: b, c, d

The skin, mucus membranes, and epithelial layers of various organs of


the body constitute effective physical barriers against microbial
invasion. In certain portions of the body, these barriers have developed
ancillary adaptations to increase the effectiveness of the barrier
functions. Skin structures such as sebaceous glands secrete chemical
compounds that serve to maintain a relatively low pH, providing
effective bacterial stasis. Mucus secretion by specialized glands within
the bronchi and gut provide a mucus layer that represents a physical
and chemical barrier to microbial invasion. Within the respiratory tract,
ciliary function serves to extrude microorganisms trapped within this
mucus layer. In the alimentary track, the very low pH within the
stomach and gut peristalsis both serve to prevent microbial adherence
and invasion.

190 Which of the following statement(s) is/are true concerning the


antibody response to an invading antigen?
a. All antibodies are composed of one type of heavy and one type of
light protein chain
b. The carboxyl terminus of the heavy chain is the antigen binding site
c. Antibody of the immunoglobulin G class is the initial antibody
produced in response to an antigenic stimulus
d. Immunoglobulins A, D, and E play an active role in the circulating
humoral response
Answer: a

97 of 167 11-Aug-15 10:57 PM


MedCosmos Surgery: MCQ : General Surgery http://medcosmossurgery.blogspot.com/search/label/MCQ : General Surgery

Humoral defenses consist of antibody (immunoglobulin; Ig) and


complement. All Ig classes (IgM, IgG, IgA, IgE, IgD) and IgG subclasses
are composed of one type (M, G, A, E, D) of heavy and one type (K and g
) of light protein chains that consist of several domains both structurally
and functionally. Each Ig molecule contains one or more units that
consist of two heavy and two light chains linked by disulfide bonds. The
amino terminus of both heavy and light chains contain several
hypervariable regions that fold in three dimensions to produce the
antigen-binding site. The carboxyl terminus of the heavy chains contain
regions that activate complement and bind Fc receptors, by which
direct adherence to polymorphonuclear leukocytes and macrophages
take place after antigen binding occurs.
Initially, antibody of the IgM class is produced in response to an
antigenic stimulus. A second exposure to the same antigen, or a cross-
reactive antigen, leads to the so-called second set response, in which
antibody of the IgG class with two binding sites is produced more rapidly
and in larger quantity compared to the initial IgM primary response.
Immunoglobulin of the IgA class is secreted by gut-associated lymphoid
tissue and is combined with secretory components of protein to form a
dimer termed secretory IgA. This antibody acts at a variety of epithelial
sites to prevent microbial adherence and invasion. IgD and IgE exist in
smaller amounts in the circulation and do not appear to play a major
role as host defense components.

191 Increasing evidence has implicated gram-negative bacterial


lipopolysaccharide (LPS endotoxin) as the portion of the gram-negative
bacterial cell membrane responsible for many, if not all the toxic
effects that occur during gram-negative bacterial sepsis. The following
statement(s) is/are true concerning LPS and the host response.
a. The LPS molecule can in itself cause physiologic responses similar to
that seen during gram-negative bacterial sepsis
b. LPS triggers host macrophages to release a variety of cytokines
including TNF-a, IL-1a, and IL-1b, IL-6, and IFN-a
c. Excessive cytokine production is not associated with detrimental
consequences
d. TNFa and IL-1b appear to be the primary mediators within the host,
exerting deleterious effects on the host when excessive amounts reach
the systemic circulation
Answer: a, b, d

The LPS molecule exerts diverse effects on the mammalian host.


Immunologic responses to LPS include nonspecific polyclonal B-cell
proliferation, macrophage activation and cytokine secretion, tolerance
to subsequent LPS or bacterial challenge, and production of antibody
directed against various portions of the LPS molecule after repeated
challenge. Physiologic responses similar to those seen during
gram-negative bacterial sepsis occur during LPS administration alone
and include hypotension, hypoxemia, acidosis, bacterial translocation
across the gut, complement and coagulation cascade activation, white
blood cell and platelet margination, and death. Indirect effects result

98 of 167 11-Aug-15 10:57 PM


MedCosmos Surgery: MCQ : General Surgery http://medcosmossurgery.blogspot.com/search/label/MCQ : General Surgery

from LPS-triggering of host macrophages. Activated macrophages


secrete a wide array of cytokines that include TNF-a, IL-1a, and IL-1b,
IL-6 and interferon-a (IFNa). Excessive secretion of cytokines produce
substantial systemic effects in the mammalian host. TNFa and IL-1b
appear to be the primary mediators within the local host milieu,
exerting deleterious effects on the host only after large amounts are
secreted and reach the systemic circulation.

192 Which of the following statement(s) concerning the gut microflora


is/are correct?
a. Gut microflora evolves constantly throughout development
b. The gut microflora can contribute to the physical and chemical
barriers at the mucus membrane level
c. Most of the microorganisms found in the oropharynx eventually pass
into the intestine
d. In the colon, anaerobic organisms outnumber aerobic organisms in a
ratio in excess of 100:1
Answer: b, d

The composition of the gut microflora is established in neonates after


ingestion of microbes that are acquired during contamination from the
birth canal and during initial feeding, and remain relatively constant
thereafter. Although this flora acts to promote development of the
immune system, the specific interactions that produce this effect have
not been fully elucidated. The microflora also contributes to physical
and chemical barriers at the mucus membrane level, in that many
autochthonous microbes possess adhesion proteins by which they can
bind to certain areas of the mucosal cell or to specific types of bacteria,
occupying potential binding sites for pathogenic organisms and
producing a substantial physical mucobacterial layer. The oropharynx
contains a number of aerobic and anaerobic microorganisms, however,
these microbial inhabitants do not usually pass into the intestine,
because the stomach itself represents a significant barrier to invading
microorganisms by virtue of its low pH, which kills most microbes. The
upper small intestine contains few organisms, mainly gram-positive
aerobes and lactobacilli. Conversely, the lower small intestine contains
a large number of aerobes and anaerobic forms, especially in patients in
whom the ileocecal valve allows free backwash of cecal contents into
the terminal ileum. Within the colon, a wide diversity and a large
number of facultative and strict anaerobic isolates are present. Only a
small number of aerobes are present, these microbes being
outnumbered 100–300 to 1 by anaerobes.

193 The use of antibiotics can be based on either the clinical course of a
patient without the benefit of well-defined microbiologic data (empiric
therapy), or targeted at specific identified pathogens once sensitivity
reports are available (directed therapy). The following statement(s)
is/are true concerning these therapies.
a. The issue of toxic side effects of antibiotics is only important in
dealing with emperic therapy

99 of 167 11-Aug-15 10:57 PM


MedCosmos Surgery: MCQ : General Surgery http://medcosmossurgery.blogspot.com/search/label/MCQ : General Surgery

b. Single agent therapy is generally inferior to specific multi-drug


therapy (aminoglycoside plus an antianaerobic agent) for the treatment
of secondary bacterial peritonitis due to appendicitis, diverticulitis,
penetrating gastrointestinal injury, or anastomotic leak
c. With the empiric use of antibiotics, a diligent search for the septic
source should be undertaken and continued until identified
d. In clinical situations in which polymicrobial infection is identified,
specifically-directed treatment for the predominant organism is
satisfactory
Answer: c

The use of empiric therapy without the benefit of well-defined


microbiologic data is appropriate when there is sufficient clinical
evidence to support the diagnosis such that it would be imprudent to
withhold antimicrobial therapy. In this setting, however, a diligent
search for the septic focus source should be undertaken and continued
(cultures, radiographic procedures, etc.), and initial limits should be
placed in the course of empiric therapy with continued reevaluation
based on the clinical course of the patient. The choice of antibiotic
agents should be based on the clinical situation and known activity
patterns within the given institution. Single broad-spectrum agents,
although suffering slightly from a lack of individual pathogen specificity,
are useful in this setting in that they provide a broad coverage against
several groups of pathogens and may avoid some of the toxic effects
with specific combined modality regimens. Similarly, for directed
therapy, single-agent therapy has been demonstrated to be equivalent
to combined therapy and should be chosen in an attempt to select
agents with appropriate sensitivities which retain suitable clinical
efficacy but exhibit minimal toxicity. After review of cultural reports,
many patients have demonstrated polymicrobial infection. Because
experimental clinical evidence supports the concept of aerobic-
anaerobic synergy, therapy should be directed against all potential
components of the infection if the body site is such that these
microorganisms may be present.

194 The following statement(s) is/are true concerning newer detection


methods of systemic infection.
a. Enzyme-linked immunosorbent assay (ELISA) is a rapid immunologic
assay used for both antigen and antibody detection
b. Southern, northern, and western immunoblot techniques are used to
detect DNA, RNA, or proteins, respectively
c. Polymerase chain reaction (PCR) is a sensitive assay used to detect
small amounts of microbial DNA, thus detecting infection at its early
stages
d. Infectious agents currently detected using advanced molecular
techniques include cytomegalovirus (CMV) and human immunodeficiency
virus (HIV)
Answer: a, b, c, d

Although the classic detection of infection based on clinical signs of


infection and bacterial culture remain the most common clinical tools,

100 of 167 11-Aug-15 10:57 PM


MedCosmos Surgery: MCQ : General Surgery http://medcosmossurgery.blogspot.com/search/label/MCQ : General Surgery

increasing reliance has been placed on assays that do not employ


cultural data. Specifically, the antibody and cytokine host responses are
being intensely examined and extremely sensitive amplified assays that
rely on antigen, antibody or microbial DNA detection are employed in
the clinical setting. Enzyme-linked immunosorbent assay (ELISA) is a
rapid, antigen-based, immunologic assay that can be used for both
antigen and antibody detection, for determination of antibody titer, as
well as for screening for monoclonal antibody production. Transblot
techniques are being used increasingly in the clinical setting. These
include southern, northern, and western immunotransblot techniques
used to detect DNA, RNA, or proteins, respectively. The polymerase
chain reaction (PCR) is being used in some centers as a sensitive assay
to detect small amounts of microbial DNA. This technique involves
extraction of the DNA from the test sample with in vitro amplification
through repeated nucleic acid denaturing and polymerization so that the
gene copy number increases exponentially. This marked amplification of
the gene copy number results in extremely sensitive tests which can
detect infection at its early stages.
Clinically, these detection methods are being used to detect a wide
variety of infectious agents including CMV and HIV. Furthermore,
preliminary investigations into possible detection of fungal pathogens
are underway.

195 Cytokines are low-molecular-weight polypeptides exerting a wide


variety of biologic effects at both local and systemic levels. Which of
the following statement(s) is/are true concerning the production and
actions of cytokines?
a. Cytokines are produced solely by macrophages
b. Cytokines act only on other cells within the same local environment
c. Cytokines may have both protective and deleterious effects on the
host
d. Each specific cytokine is produced by a single cell type
Answer: c

Macrophages, endothelial cells, lymphocytes, and other cells secrete a


large number of different compounds, termed cytokines, that are most
probably evolved for the purpose of local intercellular and intracellular
signaling. Cytokines frequently are secreted after initial lymphocyte or
macrophage activation, and may act on the secreting cell itself
(autocrine activation) or on other cells within the same local
environment (paracrine activation) to cause increased secretion of the
same cytokine or other cytokines, respectively. Some cytokines are
produced by several cell types, and most produce a wide array of
effects. The duality of the effects of the cytokine component of host
defenses, exerting both salutatory and deleterious effects on the host,
has become increasingly evident.

196 The following statement(s) is/are true concerning cellular defense


mechanisms.
a. Macrophages function solely as antigen processing cells in the initial

101 of 167 11-Aug-15 10:57 PM


MedCosmos Surgery: MCQ : General Surgery http://medcosmossurgery.blogspot.com/search/label/MCQ : General Surgery

reaction to exposure to an antigen


b. Macrophages may become activated and secrete cytokines
c. Macrophages serve as phagocytic cells in the tissues but not within
the bloodstream
d. Polymorphonucleocytes (PMNS) are normally present in only small
numbers within the tissue and enter an area of infection through
diapedesis
Answer: b, d

A wide variety of cell types serve to provide host defense at several


levels. Macrophages act as the initial antigen processing cell that serves
to present antigen to help T cells, thus initiating the immune response.
Macrophages, however, are pluripotent cells that, in the process of
engulfing and processing antigen, may become activated. Activated
macrophages secrete a variety of cytokines. Macrophages also act as
phagocytic cells in the tissues and within the bloodstream, and because
of their resident nature in many tissues, also represent the first line of
host defenses in many areas of the body, even before activation. PMNS
are present within the bloodstream, but only in small numbers within
the tissue, and enter an area of infection through diapedesis after
chemotactic stimuli are excluded by macrophages, bacterial breakdown
products, and complement activation.

197 A diabetic develops a severe perineal infection with skin necrosis,


subcutaneous crepitance, and drainage of a thin, watery, grayish and
foul-smelling fluid. Management should consist of:
a. Gram stain of the fluid, which will likely demonstrate multiple
bacteria including predominantly gram-positive rods
b. A CT scan is indicated in a stable patient to define the extent of the
disease
c. Broad spectrum antibiotics followed with prompt extensive
debridement is indicated
d. A safe guideline is to resect infected necrotic tissue so that a several
centimeter margin of grossly normal, healthy tissue can be achieved
e. A colostomy is of little benefit in this situation
Answer: a, b, c, d

The presence of severe perineal infection (referred to as Fournier


gangrene when this process involves the perineum and scrotum in
males) is associated with a continued high mortality despite aggressive
and appropriate therapy. The clinical description provided would
suggest an underlying soft tissue necrosis. In a stable patient radiologic
studies including a CT scan to define the extent of the disease and the
presence of pelvic infection is indicated. Gram stain will likely show
evidence of polymicrobial organisms but the presence of Clostridia
marked by gram-positive rods would suggest involvement with this
organism. Prompt, aggressive and extensive debridement to remove all
devitalized and affected tissue and the addition of broad spectrum
antibiotics, fluid resuscitation, hemodynamic monitoring, and
nutritional support would appear to afford the patient the best chance
of survival. The clearest guidelines to determine the limits of resection

102 of 167 11-Aug-15 10:57 PM


MedCosmos Surgery: MCQ : General Surgery http://medcosmossurgery.blogspot.com/search/label/MCQ : General Surgery

involve removal of clearly infected, necrotic tissue so that margins


several centimeters into grossly normal, healthy tissue are achieved.
Because the entire perineal region and buttocks are frequently involved
in these patients, performance of a fecal stream diversion by means of a
colostomy often provides improved wound care and patient
management, although it is not invariably a positive outcome.

198 The use of prophylactic antibiotics has become commonplace.


Which of the following statement(s) is/are true concerning the
prophylactic use of antibiotics?
a. The appropriate use of prophylactic antibiotics must include the
initiation of the agent prior to the surgical procedure
b. Continuing the antibiotic into the postoperative period has led to
improved results in antibiotic prophylaxis
c. The prophylactic administration of broad-spectrum agents (third-
generation cephalosporins) has been shown to be particularly
advantageous
d. The topical use of antimicrobial agents is of no advantage in the
prophylactic setting
Answer: a

Intravenous administration of an antibiotic is clearly indicated for


patients undergoing clean contaminated operations. These antibiotics
should be administered prior to surgery to obtain adequate tissue levels
at the time of potential contamination. However, there has been no
added benefit demonstrated for the postoperative use of antibiotics
with regard to prophylaxis. The choice of antibiotic is a complex issue
which remains unresolved largely because both superficial and deep
wound infections can occur as a result of either or both skin (superficial
wound) flora (e.g., Staphylococcus aureus) and body site (deep wound)
infection. For this reason, the administration of agents which possess
activity directed against these expected agents is reasonable. Although
administration of a first-generation cephalosporin is acceptable, second-
generation cephalosporins or extended-spectrum penicillins with
gram-positive and gram-negative activity and biliary tract excretion may
be more suitable for patients undergoing gastrointestinal or biliary tract
procedures. Similarly, the use of agents with additional anaerobic
activity for patients undergoing gastrointestinal procedures involving
the small bowel or colon should be considered. The administration of
broad-spectrum agents such as third-generation cephalosporins for
prophylaxis does not seem to provide additional benefit in comparison
to the above-mentioned type antibiotics and may foster the
development of resistant organisms within a given institution or
superinfection within a given patient. There is evidence that in some
cases the topical use of antimicrobial agents is equivalent to the
administration of intravenous antimicrobial agent antibiotics.

199 If a necrotizing soft tissue infection is considered, therapy


mandates:
a. Empiric administration of antibiotics active against gram-positive,

103 of 167 11-Aug-15 10:57 PM


MedCosmos Surgery: MCQ : General Surgery http://medcosmossurgery.blogspot.com/search/label/MCQ : General Surgery

gram-negative, and anaerobic bacteria


b. Due to usually resistant species, penicillin is not indicated
c. Immediate operative intervention and aggressive resection of all
involved tissues is mandatory
d. The use of hyperbaric oxygen has been demonstrated to be clearly
advantageous
Answer: a, c

Identification of a necrotizing, soft tissue infection mandates immediate


operative intervention with aggressive resection of all involved tissues
and empiric administration of antibiotics active against gram-positive,
gram-negative, and anaerobic bacteria. In most cases, this involves the
use of several antimicrobial antibiotics in combination. Because of
concern in all cases for the presence of Clostridia infection, high doses
of aqueous penicillin G are administered. Gram-positive organisms are
treated with vancomycin or a semisynthetic penicillin and
gram-negative organisms are treated with an aminoglycoside or a
monobactam. Anaerobic coverage is typically achieved by use of
metronidazole of clindamycin. The use of hyperbaric oxygen therapy is
controversial and unfortunately due to the rarity of the disease,
prospective randomized data is not available so that the literature
remains without controlled trials demonstrating any additional benefits
derived from hyperbaric oxygen therapy.

200 Wounds are classified according to the likelihood of bacterial


contamination. Which of the following statement(s) is/are true
concerning wound classifications?
a. A clean-contaminated wound would be that associated with an
elective colon resection with adequate mechanical and antibiotic bowel
preparation
b. A contaminated wound would include a resection of obstructed bowel
with gross spillage of intestinal contents
c. In a clean wound, no viscus is entered
d. Antibiotic prophylaxis should be administered for all clean-
contaminated and contaminated wounds and selectively in patients
involving a clean wound
Answer: a, b, c, d

Wounds are classified under three classes according to the likelihood of


bacterial contamination: 1) clean (no viscus is entered; e.g.,
herniorrhaphy); 2) clean-contaminated (minimal contamination; e.g.,
elective colon resection with adequate mechanical and antibiotic bowel
preparation, and 3) contaminated (heavily contaminated surgery; e.g.,
resection of unprepared, obstructed bowel with gross spillage of
intestinal contents or stool, drainage of abscesses, debridement of
traumatic neglected wounds). Antibiotic prophylaxis generally should be
administered for class 2 and 3 types of wounds, but patients undergoing
clean surgery do not always require antimicrobial antibiotic prophylaxis.
An exception to this tenet involves cases in which a prosthetic material
may be used (artificial joint, heart valve, tissue patch).

104 of 167 11-Aug-15 10:57 PM


MedCosmos Surgery: MCQ : General Surgery http://medcosmossurgery.blogspot.com/search/label/MCQ : General Surgery

201 The following statement(s) is/are true concerning HIV infection.


a. Initial screening with ELISA is highly sensitive but can be associated
with a false positive rate of 25%
b. Treatment with azidothymidine (AZD) appears to prolong survival
when administered early in the disease
c. Predisposition to infection in HIV infection is primarily due to
reduction in the number of helper T cells
d. Common infections in patients with AIDS are Pneumocystis, carinii
pneumonia, CMV pneumonitis, Cryptococcus meningitis, and
disseminated infection due to atypical mycobacteria
Answer: b, c, d

Acquired immunodeficiency syndrome (AIDS) is a syndrome caused by


the human retrovirus (HIV-1) that infects T lymphocytes and causes
severe immunosuppression. Individuals who become infected with HIV
are prone to a variety of infections and different types of malignancy. A
spectrum exists in which patients regress from asymptomatic infection,
to development of AIDS-related complex (ARC) of diseases to AIDS itself.
Common infections occurring in patients with AIDS are Pneumocystis
carinii pneumonia; CMV pneumonitis; gastritis, hepatitis and meningitis
due to Cryptococcus neoformans; and pneumonia and disseminated
infection due to atypical mycobacteria. Predisposition to these
infections is due, in part, to the lymphotrophic nature of HIV, which
markedly reduces the number of helper T cells as well as the absolute
number of T cells.
HIV detection typically consists of initial ELISA screening, but this test
has about a 1–3% false-positive rate, thus mandating all positive tests be
confirmed by the western immunoblot analysis.
Treatment of ARC and AIDS consists of aggressive antiinfective therapy
once a specific infection occurs and the use of AZT. AZT has been shown
to prolong survival when administered early in the course of disease and
is considered routine therapy.

202 The following statement(s) is/are true concerning initial


microbiologic diagnostic techniques.
a. Appropriate expeditious transport of specimens to the microbiology
laboratory is essential for obtaining accurate clinical information
b. The use of potassium hydroxide in preparing a specimen slide for light
microscopy will be useful in identification of anaerobic bacteria
c. Antibiotic sensitivity is determined by exposing the specific
microorganism to varying amounts of antibiotic with the concentration
of the antibiotic inhibiting growth referred to as the MIC (minimal
inhibitory concentration)
d. Serum levels of antimicrobial agents should achieve in excess of a
4-to 8-fold increase over the MIC to be considered clinically efficacious.
Answer: a, c, d

Because most surgical infections are polymicrobial, specimens should be


cultured for aerobic and anaerobic bacteria, as well as fungi. Although
aerobic and aerotolerant microorganisms often do not require special

105 of 167 11-Aug-15 10:57 PM


MedCosmos Surgery: MCQ : General Surgery http://medcosmossurgery.blogspot.com/search/label/MCQ : General Surgery

transport media, a delay in specimen processing may markedly reduce


the yield, and anaerobic transport media have been demonstrated to
markedly increase the cultural yield of this type of organism. The initial
piece of information gained concerning potential infection may come
from simple staining of a specimen. Gram stain, which will identify the
staining characteristics of the organisms, as well as their number should
be performed on all specimens. Potassium hydroxide is useful in that it
will lyse bacteria and other cellular elements within a preparation and
allow observation of yeast or mycelial elements.
Initial culture results may solely indicate that microorganisms are
growing and full characterization may take two to three days. Once a
specific microorganism is identified, a sample is inoculated during the
log phase into broth containing varying amounts of an antibiotic. After
an 18-to 24-hour period, the tube or well that exhibits no visible growth
is then noted, and the reciprocal of this dilution is termed the minimal
inhibitory concentration (MIC). This value may be compared to either
measured or known achievable serum levels for a particular antibiotic.
In general, antimicrobial agents that achieve in excess of a 4-to 8-fold
increase over MIC during the peak serum level have been demonstrated
to be clinically efficacious.

203 The complement system consists of a series of serum proteins that


exist in a quiescent or very low-level state of activation in the
uninfected host. Which of the following statement(s) is/are true
concerning complement activation?
a. The alternate (properdin) pathway of complement activation can
occur directly through contact with fungal or bacterial cell wall
compounds
b. Complement component fragments may serve to decrease vascular
permeability
c. Excessive complement activation can produce deleterious effects
d. Fragments of certain complement components serve as
chemoattractants to additional cellular components of the host defense
mechanism
Answer: a, c, d

Complement activation can occur through either classic or alternate


(properdin) pathways, both of which eventuate in deposition of terminal
complement pathway components on the antigenic cell surface. The
classic pathway of complement activation usually begins with
immunoglobulin G-binding which has also bound the antigen. The
alternate pathway activation occurs in response to activation of direct
binding of the antigen or directly through contact with fungal and
bacterial cell wall compounds such as zymosan and gram-negative
bacterial lipopolysaccharide (LPS endotoxin). Several complement
components represent important host defenses acting to recruit or
augment cellular host defenses or to directly inactivate invading
microbes through lytic activity. The production of complement
component fractions C3a and C5a during activation of this cascade serve
primarily to markedly increase vascular permeability, and C5a functions
as a PMN and macrophage chemoattractant. This process leads to the

106 of 167 11-Aug-15 10:57 PM


MedCosmos Surgery: MCQ : General Surgery http://medcosmossurgery.blogspot.com/search/label/MCQ : General Surgery

recruitment of additional humoral and cellular defenses to the specific


area of infection. Excessive complement activation can produce
deleterious effects in some instances. Complement activation causes
enhanced PMN adhesion, margination, and release of lysosomal enzymes
that can directly damage certain target tissues, such as the lung.

204 A 55-year-old renal transplant patient has been hospitalized in a


Surgical Intensive Care Unit, receiving a prolonged course of antibiotics
following an attack of acute cholecystitis. The following statement(s)
is/are true concerning his management.
a. Due to the potential risk of Candida infection, prophylaxis with oral
nystatin should be instituted early in the patient’s course
b. A Candida urinary tract infection should be treated with systemic
amphotericin B
c. Changes of Candida retinitis are of little significance
d. The presence of a virulent Candida bacteremia should suggest a
dosage reduction in immunosuppressive agents until the infection can be
adequately controlled
Answer: a, d

Infections due to fungal pathogens have become increasingly common


during the past decade, frequently occurring in patients undergoing
prolonged hospitalization in the Surgical Intensive Care Unit and in
immunocompromised individuals. Prophylaxis with oral antifungal agents
(nystatin) is warranted, especially during periods of maximal
immunosuppression in transplant patients, in patients with uncontrolled
diabetes, or during some cases of prolonged antibacterial microbial
therapy. In general, local, apparently noninvasive Candida infections
involving the integument and mucus membranes are treated with oral
decontamination and topical antifungal therapy using topical agents
such as nystatin. Candida urinary tract infections can be treated with
either an oral antifungal agent or with topical amphotericin B as a
continuous bladder irrigation. Several studies have demonstrated that
those patients with three positive sites of Candida infection, or with
peritoneal or blood cultures positive for Candida exhibit higher survival
rates when amphotericin B therapy is instituted earlier in the course of
infection. The presence of retinal changes compatible with Candida
retinitis or Candida present within the peritoneal cavity are considered
indications for a limited course of amphotericin B therapy (300% to 500
mg). Patients receiving exogenous immunosuppressive agents should
undergo a marked dose reduction, and some agents should be
discontinued until evidence of infection is absolutely controlled or is
eradicated.

205 The initiation of a humoral immune response involves a complex


interaction of the antigen, cells and intercellular messengers. Which of
the following statement(s) concerning the initiation of the humoral
immune response is/are correct?
a. Helper T lymphocytes stimulate B lymphocytes through secretion of
cytokines such as interleukin 4 and 6

107 of 167 11-Aug-15 10:57 PM


MedCosmos Surgery: MCQ : General Surgery http://medcosmossurgery.blogspot.com/search/label/MCQ : General Surgery

b. A number of cells can aid in presenting the antigen to the helper T


cell including B lymphocytes and macrophages
c. All antigens require coordinated efforts of the various cellular
components of the immune system
d. An antigen must be a living microorganism
Answer: a, b

Stimulation of the immune system occurs after a variety of antigen-


presenting cells (B lymphocytes, macrophages, dendritic cells, and
Langerhans cells) act to engulf, process, and present antigen to T
lymphocytes of helper lineage. These T lymphocytes, in turn, act to
stimulate B lymphocytes to become mature plasmacytes (through
secretion of cytokines such as interleukin 4 and 6) dedicated to the
production of antibody directed against the specific antigen. An antigen
may be defined as any substance that stimulates the host immune
response; that is, that the host immune system recognizes is foreign.
Thus, an antigen may be an invading microorganism, an inert particle,
or any type of chemical compound that triggers the host immune
system. Although some antigens are able to directly stimulate B
lymphocytes in and of themselves to produce antibody (many
polysaccharides), most antigens require coordinated efforts of the
various cellular components of the immune system.

206 The following statement(s) is/are true concerning viral infections.


a. The most common post-transplantation viral infections are caused by
herpes viruses and include CMV and herpes simplex virus
b. Viral infections occur at equal frequency anytime during the
post-transplantation period
c. CMV infection in the post-transplant patient is most likely a
pulmonary process
d. Herpes simplex virus (HSV) infection primarily presents with a
mononucleosis-type syndrome with fever, lethargy, and cough
Answer: a, c,

Solid organ transplant patients are prone to develop viral infection by


virtue of exogenous immunosuppression. The most common
post-transplantation viral infections are those caused by herpes viruses
(CMV, herpes simplex virus [HSV], Epstein-Barr virus [EBV], and
Varicella-Zoster virus [VZV]). All are most common during periods of
maximal host immunosuppression that occur immediately
post-transplantation and during periods of allograft rejection. CMV is a
common cause of fever after solid organ transplantation, and evidence
of CMV infection occurs in approximately 30% of patients. The most
common presentation for CMV infection is that of a febrile, leukopenic
patient with a cough, diffuse interstitial infiltrates on chest x-ray, and
hypoxia.
HSV infection causes primarily oral pharyngeal ulcerations in most
cases, although sporadic cases of disseminated disease have been
reported. EBV causes an occasional case of mononucleosis-type
syndrome but has also been clearly indicated in the pathogenesis of
post-transplantation lymphomas. VZV infection can present as

108 of 167 11-Aug-15 10:57 PM


MedCosmos Surgery: MCQ : General Surgery http://medcosmossurgery.blogspot.com/search/label/MCQ : General Surgery

disseminated and occasionally life-threatening infections in the


nonimmune transplant patient or as painful herpes zoster in patients
who have previously developed chicken pox.

207 The following statement(s) is/are true concerning necrotizing


fascitis.
a. Mortality rates as high as 40% can be expected
b. The infection involves only the superficial fascia, sparing the deep
muscular fascia
c. An impaired immune system is a common factor predisposing to this
condition
d. The infection is usually polymicrobial
e. Necrotizing fascitis is most likely to develop in the face of impaired
fascial blood supply
Answer: a, c, d, e

Necrotizing fascitis is an uncommon infection of the deep and


superficial fascia that is associated with mortality as high as 40% in
many series. Although many underlying disease processes predispose
patients to necrotizing fascitis, three common factors are almost
invariably present: 1) impairment of the immune system; 2) compromise
of fascial blood supply, and 3) the presence of microorganisms that are
able to proliferate within this environment. Infections of this type are
usually polymicrobial in nature, with gram-positive organisms such as
staphylococci and streptococci, gram-negative enteric bacteria, and
gram-negative anaerobic being frequently identified. These
polymicrobial cultural results are assuredly indicative of the occurrence
of a synergistic process, perhaps in large part accounting for the
severity of these infections. Some microorganisms possess virulence
factors that, in conjunction with an underlying host predisposition,
allow this disease process to occur without dependence on other
bacteria. Examples of such bacteria include Clostridium, Pseudomonas,
and Aeromonas. In these patients, the process is often fulminant and is
frequently associated with cellulitis, myositis, fascitis, and bacteremia
with attendant high mortality.

208 New treatment modalities designed to modulate host defense


mechanisms that have been demonstrated conclusively to be of benefit
include:
a. Gut decontamination
b. Anti-LPS antibody
c. Anti-TNF antibody
d. Thymopentin
e. None of the above
Answer: e

Selective gut decontamination involves the use of orally administered


antibiotics that achieve a high intraluminal level directed against
gram-negative aerobes and yeast, leaving the host anaerobic intestinal
microflora relatively undisrupted. Although a reduction and alteration of

109 of 167 11-Aug-15 10:57 PM


MedCosmos Surgery: MCQ : General Surgery http://medcosmossurgery.blogspot.com/search/label/MCQ : General Surgery

the microorganisms responsible for infectious episodes have been


demonstrated in certain groups of patients, a clear-cut impact on host
mortality has not been shown. Because LPS may be responsible for
toxicity both directly and through host mediator systems, the
availability of agents to bind against this portion of the gram-negative
bacteria to reduce mortality has been intensively examined.
Unfortunately, large multicenter randomized trials provide no evidence
of benefit for this treatment. Similarly, since many of the systemic
manifestations of gram-negative bacteremia are mediated by cytokines,
the effect of an anti-TNF antibody preparation is currently in clinical
trial. No proven benefits have yet been identified. Finally, the use of
immunostimulants to enhance the state of activation of host defenses
has been proposed. Thymopentin is a peptide that contains active
thymopoetin, a thymic molecule that acts to stimulate T-lymphocyte
activity. Preliminary trials indicate that this agent ameliorates host
septic response after major operations and trauma but conclusive
evidence that concurrent reduction of infection-related mortality occurs
is not available.

209 Antibacterial agents can be classified with regard to their structure,


mechanism of action, and activity pattern against various types of
bacterial pathogens. Which of the following statement(s) is/are true
concerning antimicrobial classes?
a. Penicillins and cephalosporins share the compound structure of a
b-lactam ring which binds to bacterial division plate proteins
b. Tetracyclines and macrolides such as erythromycin inhibit bacterial
ribosomal activity and therefore protein synthesis
c. Aminoglycosides act in a similar fashion to tetracyclines and
therefore are both bacteriostatic
d. Sulfonamides and trimethoprim act synergistically to inhibit purine
synthesis
Answer: a, b, d

Penicillins, cephalosporins, and monobactams possess a b-lactam ring of


some type and act to bind bacterial division plate proteins, thus
inhibiting cell wall peptidoglycan synthesis and either causing or
inducing autolytic bacteriolysis. Because gram-positive and
gram-negative bacteria possess different types of division plate
proteins, many of these agents exhibit differential activity between
these two types of microorganisms. Tetracyclines, chloramphenicol, and
macrolides inhibit bacterial ribosomal activity, and thus overall protein
synthesis by a variety of different mechanisms. Aminoglycosides act to
inhibit protein synthesis and also presumably act on a different target
site, a supposition based on the fact that aminoglycosides are
bacteriolytic and the other agents are bacteriostatic. Vancomycin
inhibits assembly of peptidoglycan polymers, whereas quinolones bind to
DNA helicase proteins and inhibit bacterial DNA synthesis. Sulfonamides
and trimethoprim act in different mechanisms to inhibit protein
synthesis, therefore two agents in combination act synergistically.

110 of 167 11-Aug-15 10:57 PM


MedCosmos Surgery: MCQ : General Surgery http://medcosmossurgery.blogspot.com/search/label/MCQ : General Surgery

210 The treatment of the following patient should include:


a. Initial empiric therapy directed against both aerobes and anaerobes
b. The addition of anti-fungal therapy in an elderly patient
c. A minimum of two weeks of antibiotic therapy is indicated
d. The addition of appropriate antibiotic therapy has made surgical
therapy unnecessary in such cases
e. Either a single agent or combination therapy is appropriate if the
agents selected possess activity against both aerobic and anaerobic
bacteria
Answer: a, e

The primary treatment for a perforated viscus is surgical, however


antimicrobial therapy is an extremely important adjunct. Empiric
antibiotic therapy for secondary bacterial peritonitis and intraabdominal
abscess should be directed against both aerobes and anaerobes.
Administration of an agent directed against only one component of the
infection or the other is inferior to combined therapy. Several studies
indicate that the results of using several agents in combination is
equivalent to the use of a single agent therapy as long as the agents
selected possess activity against both components of the infection. The
addition of antientercoccal or antifungal agents as initial therapy has
not been substantiated. The most beneficial duration of antibiotic
therapy must be based on the setting for the specific patient. Minimal
peritoneal contamination with adequate surgical treatment may be
treated with a three-to five-day course of antibiotics, whereas longer
periods are indicated for immunosuppressed patients and with patients
with extensive contamination.

211 The following statement(s) is/are true concerning host defense


mechanisms to intraabdominal infection.
a. Bacterial clearance can occur via translymphatic absorption
b. Phagocytic activity and bacterial killing can occur via resident
phagocytic cells and an influx of PMNs
c. A fibrinogen-rich inflammatory exudate is released into the
peritoneal cavity, trapping large numbers of bacteria and other
particulate matter
d. Perforations of a bowel may be walled off but are seldom sealed by
the omentum and other mobile viscera
Answer: a, b, c

The introduction of microorganisms into the normally sterile peritoneal


environment invoke several potent specialized host antimicrobial
defense mechanisms. Bacterial clearance, also termed translymphatic
absorption, occurs through specialized structures found only on the
peritoneal mesothelium on the underside of the diaphragm that act as
conduits for both fluid and particulate matter. Lymphatic channels
eventually form which drain into the venous circulation via the thoracic
duct. Bacteria not cleared via translymphatic absorption are rapidly
engulfed by resident and recruited phagocytic cells including resident
macrophages on the peritoneal surface and omentum and attracted
PMNs. The final primitive host defense mechanism is sequestration by

111 of 167 11-Aug-15 10:57 PM


MedCosmos Surgery: MCQ : General Surgery http://medcosmossurgery.blogspot.com/search/label/MCQ : General Surgery

which a fibrinogen-rich exudate containing plasma oposonins appears


during peritoneal infection and fibrin polymerization occurs. Fibrin has
the capacity to trap large numbers of bacteria and other particulate
matter. Acting in conjunction with omentum and other mobile viscera,
perforations are sealed and the contaminated enteric contents walled
off, preventing continued soilage of the peritoneal cavity.

212 A 67-year-old male presents with an intraabdominal abscess


secondary to perforated sigmoid diverticulitis. The following
statement(s) is/are true concerning his intraabdominal abscess.
a. Culture will likely reveal a solitary organism
b. Both aerobic and anaerobic islets are encountered in 50% of
specimens
c. The most common aerobic islet will be likely E. coli and other
gram-negative enteric bacilli
d. The most common anaerobic islet will be a Bacteroides species
Answer: b, c, d

Typically an intraabdominal infection results in perforation of a hollow


viscus and the ensuing contamination of a normally sterile peritoneal
cavity. The normal bacterial flora found in that particular location of
the alimentary tract thus determines the initial inoculum. In parallel
with the overall quantity of microorganisms, (both aerobes but
predominantly anaerobes) perforations of the lower small bowel and
colon produce a high frequency of infections that contain anaerobic
microorganisms. Certain predictable patterns of bacterial islets are
found, but on average four to five islets occur in patients with
established intraabdominal infection, more than half of which are
anaerobes. Both aerobes and anaerobes are encountered in 80% to 90%
of specimens. Commonly encountered aerobes isolated are E. coli and
other gram-negative enteric bacilli such as Enterobacter, Klebsiella.
Among the anaerobes, Bacteroides species (especially B. fragilis,
Clostridium), and anaerobic cocci are most consistently isolated.

213 The following statement(s) is/are true concerning gram-negative


bacterial sepsis.
a. Mortality due to this condition has almost been eliminated due to
therapeutic intervention with antibiotics, aggressive hemodynamic
monitoring and fluid resuscitation
b. Recent series have noted a decrease in the incidence of this condition
c. Predisposing factors include old age, malnutrition, and
immunosuppression
d. Pseudomonas bacteremia is the most common cause of gram-negative
sepsis
e. Polymicrobial sepsis is generally considered a more serious problem
than sepsis due to a single organism
Answer: c, e

Gram-negative bacterial sepsis is a serious disease process that produces


substantial morbidity and mortality in both normal and

112 of 167 11-Aug-15 10:57 PM


MedCosmos Surgery: MCQ : General Surgery http://medcosmossurgery.blogspot.com/search/label/MCQ : General Surgery

immunocompromised patients (10% to 20% and 30% lethality,


respectively), despite therapeutic intervention with antimicrobial
agents, aggressive hemodynamic monitoring, fluid resuscitation, and
metabolic support. During the past several decades, nosocomial
infections due to gram-negative pathogens have increased in frequency
with resultant increase in the incidence of gram-negative bacteremia to
between 3 and 13 cases per 1000 hospital admissions. Factors that
predispose to these infections include: 1) underlying host disease
processes such as malignancy, diabetes; 2) old age and disability; 3)
malnutrition; 4) previous or concurrent antimicrobial antibiotic therapy;
5) major operations; 6) respiratory or urinary manipulation or
intubation; and 7) immunosuppression.
Although many different organisms cause this form of sepsis, E. coli
predominates in overall frequency. Also common are isolates of
Klebsiella, Enterobacter and Serratia; Pseudomonas bacteremia is less
common. Some studies, however, have suggested that Pseudomonas
sepsis is associated with the highest lethality. In several series, 10% to
20% of patients have had polymicrobial series, and most investigators
agree that polymicrobial sepsis is more lethal than infection with a
single organism.

214 Which of the following statement(s) is/are true concerning the


various types of shock?
a. Traumatic shock is more commonly associated with subsequent organ
injury and multiorgan failure syndrome than hemorrhagic shock
b. Cardiogenic shock can be of either an intrinsic or compressive nature
c. Hypodynamic septic shock is associated with a decreased mortality
risk when compared with hyperdynamic septic shock
d. Hypoadrenal shock usually responds quickly to resuscitation
e. Neurogenic shock occurs with the absence of sympathetic activity
Answer: a, b, d, e

Classification schemes of shock based on cause have been developed for


the seemingly dissimilar processes leading to circulatory collapse and
the shock state. Hypovolemic shock, the most common, is the result of
intravascular volume depletion through loss of red blood cell mass or
plasma volume. Microvascular hypotension results from a combination of
low intravascular blood volume, diminished cardiac output, and
compensatory sympathetic peripheral vasoconstriction. Shock associated
with trauma (traumatic shock) arises from the consequences of
hypovolemia due to hemorrhage in conjunction with direct soft tissue
injury and bone fracture. Hypovolemia caused by blood loss and fluid
extravasation into injured tissues is compounded by activation of
maladaptive inflammatory cascades initiated by the tissue injury. In
contrast to pure hemorragic shock, subsequent organ injury and
multiorgan failure syndrome (MOFS) occurs much more frequently
following traumatic shock due to the over-expression of these immuno-
inflammatory cascades. Cardiogenic shock is the result of failure of the
heart as an effective pump, resulting in inadequate cardiac output,
tissue perfusion and oxygen delivery. Intrinsic causes include myocardial
infarction, cardiomyopathy, valvular heart disease, or rhythm
disturbances. Compressive cardiogenic shock is a discrete entity that

113 of 167 11-Aug-15 10:57 PM


MedCosmos Surgery: MCQ : General Surgery http://medcosmossurgery.blogspot.com/search/label/MCQ : General Surgery

results when extrinsic compression of the heart limits diastolic filling


and thus systolic ejection and cardiac output. Septic shock refers to
hypotension and circulatory insufficiency developing as a consequence
of infection and the systemic response to that infection. In its
hyperdynamic form, septic shock is marked by diminished peripheral
vascular resistance and generalized vasodilatation causing relative
hypovolemia. In contrast, hypodynamic septic shock occurs in situations
of inadequate resuscitation or preterminal cardiovascular
decompensation, and is associated with vasoconstriction and a greatly
increased mortality risk. Sympathetic denervation through spinal cord
injury, spinal anesthesia, or severe head injury produces generalized
arterial vasodilatation and venodilation. Shock occurs when the normal
blood volume fails to fill the available intravascular space and severe
relative hypovolemia exists. Despite hypotension, there is a noteworthy
absence of sympathetic activity, as occurs in hypovolemia or cardiogenic
shock. Profound shock can occur in surgical patients following stress due
to the loss of the homeostatic corticosteroid response. Hemodynamic
instability may develop after an operative procedure or coincident with
an unrelated illness. The profound circulatory collapse is often
refractory to vigorous resuscitation with fluids and pressor agents. The
response to exogenous corticosteroids is usually dramatic and
potentially life-saving.

215 Which of the following statement(s) is/are true concerning


metabolic derangements in sepsis and the systemic inflammatory
response syndrome which may follow progressive shock?
a. Alterations in glucose metabolism lead to the development of
efficient substrate utilization
b. A progressive rise in serum triglyceride levels result from less
efficient clearance and increased hepatic lipogenesis
c. A net negative nitrogen balance occurs due to the oxidative
metabolism of proteins to meet energy needs
d. The serum aromatic amino acids fall rapidly as they are actively used
in oxidative metabolism
Answer: b, c

A broad spectrum of metabolic abnormalities become apparent in sepsis


and the systemic inflammatory response syndrome following shock.
Disruption of the normal cycles of carbohydrate, lipid, protein, and
oxygen metabolism occur as hypermetabolism develops. Through the
Cori cycle, lactate from the periphery is shuttled back to the liver,
where it is used in the production of glucose. Because pyruvate is
converted to alanine in the periphery, flux of alanine also contributes to
hepatic gluconeogenesis. The glycolytic oxidation of glucose to pyruvate
and its subsequent glugoneogenic regeneration from lactate is an
inefficient cycling of substrate. There is no net energy production, but
heat is released in significant quantities. Alterations in lipid metabolism
cause a progressive rise in the serum triglyceride level as a result of less
efficient clearance of exogenous triglycerides coupled with increased
hepatic lipogenesis. Profound alterations in protein and amino acid
metabolism develop with characteristic changes in amino acid levels,

114 of 167 11-Aug-15 10:57 PM


MedCosmos Surgery: MCQ : General Surgery http://medcosmossurgery.blogspot.com/search/label/MCQ : General Surgery

nitrogen balance, and skeletal muscle mass. Initially levels of the


branch chain amino acids are reduced, whereas those of the aromatic
amino acids are elevated. There is an increase in the oxidative
metabolism of protein to meet energy needs and a tremendous
mobilization of nitrogen with net negative nitrogen balance. The branch
chain amino acids are preferentially utilized in the TCA cycle to
maintain an activity that otherwise would be lost from the diminished
entry of carbohydrate-and fatty acid-generated acetyl coenzyme A. This
results in reduced serum level of leucine, isoleucine and valine.

216 Which of the following statement(s) is/are true concerning the


microvascular and cellular response to shock?
a. Osmodically induced mobilization of intracellular fluid is the initial
response to restore intravascular volume
b. With larger volume hemorrhagic shock deterioration of normal
cellular transmembrane potential occurs resulting in an increase in
extracellular sodium and water
c. The accumulation of anaerobic metabolites override normal
homeostatic vasomotor tone and contribute to the maladaptive
vasodilatation
d. Abnormal intracellular calcium homeostasis may contribute to the
cellular dysfunction of shock
Answer: c, d

Moderate hypovolemia results in a relatively rapid spontaneous


restitution of intravascular volume through expansion of the plasma
space. This plasma expansion by erythrocyte free fluid occurs within
one hour as a result of alterations in pressure and osmolarity and
produces an associated hemodilution. Sympathetic discharge, associated
arteriolar constriction, and induced metabolic changes in osmolarity
initiate the compensatory events at the microcirculatory level. The
initial pressure-related phase of restitution of blood volume in shock is
overlapped by a second phase involving osmotically induced
mobilization of intracellular fluid. Osmotic mechanisms contributing to
the restitution of blood volume after moderate hemorrhage are not
adequate in hemorrhage of greater magnitude. In larger hemorrhages
(over 25% blood volume), there is also deterioration of the normal
cellular transmembrane potential, an increase in intracellular sodium
and water, and a concomitant decrease in extracellular fluid volume.
Tissue hypoxia results, anaerobic metabolites accumulate, and the cell
cannot maintain the normal cell membrane potential. Accumulation of
hydrogen ion, lactate, and other products of anaerobic metabolism
override homeostatic vasomotor tone and contribute to a maladaptive
vasodilatation, further augmenting hypotension and hypoperfusion. The
uptake of fluid by the “failing” cell is a major source of food
sequestration following shock. Loss of membrane function is
proportional to both the extent and duration of shock or degrees of
sepsis. The etiology of membrane failure is unclear but appears
multifactorial. Loss of intracellular ATP energy stores during
hypoperfusion or direct toxicity during sepsis may inhibit the membrane
sodium-potassium pump. Cellular dysfunction also appears to be related

115 of 167 11-Aug-15 10:57 PM


MedCosmos Surgery: MCQ : General Surgery http://medcosmossurgery.blogspot.com/search/label/MCQ : General Surgery

to abnormal intracellular calcium homeostasis.

217 Which of the following statement(s) is/are true concerning the


pulmonary response to shock?
a. The acute pulmonary vascular response to shock differs markedly
from that of systemic vasculature
b. The pulmonary edema of ARDS occurs in the face of elevated left
heart pressures
c. The initial physiologic changes of ARDS involve the capillary
endothelial cells and the type I pneumocyte
d. Mechanisms proposed in the pathogenesis of ARDS include injury from
mediators of inflammation elsewhere and from activated cellular
elements
e. A decrease in lung compliance may result from the loss of type I
pneumocytes
Answer: c, d, e

Contributing pathophysiologic processes to the pulmonary


manifestations of shock include the pulmonary component of the
cardiovascular response, disruption of the normal lung mechanics, and
acute lung injury or ARDS due to sepsis. Pulmonary function may be
further compromised by pathology intrinsic to the lung itself, including
pulmonary contusion, aspiration, airway obstruction, pneumonia,
pneumothorax, hemothorax, and atelectasis. The acute pulmonary
vascular response to shock largely parallels that of the systemic
vasculature. The increase in pulmonary vascular resistance, which may
proportionally exceed that of the systemic circulation, transiently
accompanies the systemic adrenergic response. ARDS is a syndrome of
progressive lung injury that may arise as a direct consequence of shock
or other disease processes. The characteristic findings of ARDS are the
presence of pulmonary edema, hypoxemia, and significantly decreased
lung compliance. The pulmonary edema is noncardiac in origin and
occurs in the face of normal left heart pressures. The hypoxemia results
from the development of intrapulmonary shunting and perfusion of
under and nonventilated alveoli. The decrease in lung compliance
results from the loss of surfactant and lung volume in combination with
the presence of interstitial fluid and alveolar edema. Progressive
histologic changes of ARDS become apparent in pulmonary capillaries,
interstitium, and alveoli. Initially, interstitial edema develops with
swelling of the capillary endothelial cells and the type I pneumocytes.
The type I pneumocytes subsequently slough, and alveolar edema
ensues. Functional surfactant is lost with a significant increase in alveoli
opening pressure and decrease in alveolar surface tension. Mechanisms
proposed in the pathogenesis of ARDS include injury from mediators of
inflammation elaborated elsewhere, and from activated cellular blood
elements.

218 Which of the following statement(s) is/are true concerning the


diagnosis and management of hypovolemic shock?
a. A fall in hematocrit or hemoglobin always accompanies hemorrhagic

116 of 167 11-Aug-15 10:57 PM


MedCosmos Surgery: MCQ : General Surgery http://medcosmossurgery.blogspot.com/search/label/MCQ : General Surgery

shock
b. The treatment of shock is generic regardless of the etiology
c. Pharmacologic intervention to increase myocardial contractility in
hypovolemic shock is an important part the early management
d. Complications are less frequent after treatment of hemorrhagic shock
than septic or traumatic shock
Answer: d

Hypovolemic shock is readily diagnosed when there is an obvious source


of volume loss and overt signs of hemodynamic instability and increased
adrenergic output are present. After acute hemorrhage, hemoglobin and
hematocrit values do not change until compensatory fluid shifts have
occurred or exogenous fluid is administered. These values decrease
once transcapillary refill, osmotic-induced shifts, or non-RBC volume
resuscitation expands the blood volume. It is imperative that the
distinction be made between hypovolemic and cardiogenic forms of
shock, because appropriate therapy differs dramatically. Restoration of
perfusion in hypovolemic shock requires reexpansion of circulating blood
volume in conjunction with necessary interventions to control ongoing
volume loss. Continued hemodynamic instability after fluid resuscitation
implies that shock has not been reversed or that there is ongoing blood
or volume loss. In severe, prolonged hypovolemia, ventricular
contractile function may itself become depressed and require inotropic
support to maintain ventricular performance, but in general,
pharmacologic interventions directed toward increased contractility in
situations of inadequate preload are ineffective, further complicate
metabolic derangements, and are not indicated until adequate volume
replacement has been completed. Complications are less frequent after
treatment of hemorrhagic shock than in situations of septic or traumatic
shock. In the later circumstances, the massive activation of
inflammatory mediator response systems and consequences of their
disseminated, indiscriminate cellular injury can be quite profound.

219 Which of the following statement(s) is/are true concerning the


neuroendocrine responses to shock?
a. Sympathetic nerve endings release epinephrine which is responsible
for greater than 80% of systemic vascular resistance
b. Endogenous epinephrine is the primary contributor to systemic
vascular resistance
c. Increased pancreatic secretion of glucagon contributes to glucose
intolerance associated with injury and sepsis
d. The renin-angiotensin axis further augments the sympathetic-
mediated vasoconstriction
Answer: c, d

The neuroendocrine response to shock attempts to achieve restoration


of effective blood volume, mobilization of metabolic substrates, and
maintenance of central profusion. Both peripheral and central afferent
stimuli to the central nervous system are involved in inducing this
response. Hypotension, associated with a decrease in impulses from the
aortic and carotid baroreceptors, disinhibits the vasomotor center. This

117 of 167 11-Aug-15 10:57 PM


MedCosmos Surgery: MCQ : General Surgery http://medcosmossurgery.blogspot.com/search/label/MCQ : General Surgery

disinhibition results in increased adrenergic output and decreased vagal


activity. Sympathetic nerve endings release norepinephrine, inducing
peripheral and splanchnic vasoconstriction which is responsible for
greater than 80% of systemic vascular resistance and is a major
contributor to maintenance of central organ perfusion and venous
return. Plasma levels of both epinephrine and norepinephrine are
elevated with injury, and the degree of the catecholamine elevation
corresponds to the magnitude of injury. In shock the effects of
endogenous epinephrine are largely metabolic. In addition to initiating
autonomic nervous activity, the hypothalamus secretes releasing
hormones, which induce the stress hormone release of the pituitary. As
part of this response, adrenocorticotropic hormone (ACTH) secretion by
the anterior pituitary is increased stimulating cortisol secretion by the
adrenal cortex. In conjunction with elevated plasma levels of cortisol
and epinephrine, increased pancreatic secretion of glucagon accelerates
hepatic gluconeogenesis and further aggravates the glucose intolerance
that follows injury and sepsis. The secretion of renin is increased in
responses to adrenergic discharge and decreased perfusion of the
juxtaglomerular apparatus in the kidney. Renin allows formation of
angiotensin I in the liver, which is then converted to angiotensin II in the
lungs. Angiotensin II is an extremely effective vasoconstrictor that
further augments sympathetic-mediated vasoconstriction.

220 A 22-year-old man sustains a single stab wound to the left chest and
presents to the emergency room with hypotension. Which of the
following statement(s) is/are true concerning his diagnosis and
management?
a. The patient likely is suffering from hypovolemic shock and should
respond quickly to fluid resuscitation
b. Beck’s triad will likely be an obvious indication of compressive
cardiogenic shock due to pericardial tamponade
c. Echocardiography is the most sensitive noninvasive approach for
diagnosis of pericardial tamponade
d. The placement of bilateral chest tubes will likely resolve the problem
Answer: c

Shock from cardiac compression occurs when external pressure on the


heart impairs ventricular filling. Because ventricular filling is a function
of venous return and myocardial compliance, any process that places
pressure on the heart can cause compressive cardiogenic shock.
Included among these are pericardial tamponade, tension
pneumothorax, mediastinal hematoma, and positive pressure from
mechanical ventilation. Any patient with hypotension after a wound in
proximity of the heart should be considered to have compressive
cardiogenic shock until proven otherwise. The classical clinical findings
of pericardial tamponade include Beck’s triad of hypotension, neck vein
distention and muffled heart sounds. Pulses paradoxus may be noted
(this involves a decrease rather than the normal increase of systolic
blood pressure with inspiration; values 10mmHg are significant). These
findings, however, may be obscured in a noisy emergency room
environment by positive pressure ventilation or by associated injuries.
Placement of a CVP catheter confirms the elevation of right-sided filling

118 of 167 11-Aug-15 10:57 PM


MedCosmos Surgery: MCQ : General Surgery http://medcosmossurgery.blogspot.com/search/label/MCQ : General Surgery

pressure. If a pulmonary artery catheter has been placed, findings


consistent with tamponade or other forms of cardiac compression are a
trend toward equalization of chamber pressures as hypotension
progresses. In the patient at risk, echocardiography is an extremely
sensitive and noninvasive approach to demonstrate pericardial fluid and
the need for operation. Pericardial tamponade must be relieved
urgently and cardiac injuries require emergent sternotomy. Chest tube
placement would not be appropriate as the sole treatment in this
patient.

221 A 32-year-old man suffers a spinal cord injury with a resultant


paraplegia in a motorcycle accident. He presents to the emergency
room with hypotension. Which of the following statement(s) is/are true
concerning his diagnosis and management?
a. The low blood pressure can be assumed to be due to neurogenic
shock
b. The sole cause of hypotension is the loss of sympathetic input to the
venous system
c. Despite significant hypotension, secondary organ injury will be
uncommon
d. There is no role for pharmacologic intervention to maintain blood
pressure
Answer: c

Neurogenic shock results from interruption of sympathetic vasomotor


input and develops after spinal cord injury, spinal anesthesia, and
severe head injury. Under normal conditions, baseline sympathetic
activity establishes a degree of arteriolar and venous constriction.
Ablation of this tone results in decreased systemic vascular resistance
and a dramatic increase in venous capacity, causing hypotension due to
relative hypovolemia. Arteriolar dilatation not only lowers the systemic
vascular resistance but also allows previously unopened vascular beds to
be perfused, greatly expanding venous capacity. Removal of sympathetic
inputs to innervated portions of the venous system allows further
venodilatation. Restoration of an effective, albeit expanded,
intravascular volume may require extremely large volumes of
resuscitation fluid to restore normal cardiac filling pressures. This will
restore cardiac output and reverse hypotension. However,
pharmacologic intervention with vasoactive drugs may be necessary and
is preferable to excessive volume resuscitation. Post-shock sequelae are
infrequent. Although there is significant hypotension with neurogenic
shock, there is usually little if any hypoperfusion. Thus, activation of
inflammatory cascade and subsequent organ injury rarely occur.
A major pitfall in the management of neurogenic shock arises when
there is coexistent hemorrhage or ongoing volume loss that is not
appreciated. This is not an unusual situation because cervical spine
trauma causing paraplegia or severe head injury is frequently associated
with multiple injuries. Thus, in trauma the initial response to
neurogenic shock is large volume resuscitation regardless of the
presumed etiology. If hemodynamic instability persists after initial
trauma resuscitation, one must assume that the cause is not neurogenic

119 of 167 11-Aug-15 10:57 PM


MedCosmos Surgery: MCQ : General Surgery http://medcosmossurgery.blogspot.com/search/label/MCQ : General Surgery

and search for occult blood loss or cardiogenic causes of shock.

222 Which of the following statement(s) is/are true concerning septic


shock?
a. The clinical picture of gram negative septic shock is specifically
different than shock associated with other infectious agents
b. The circulatory derangements of septic shock precede the
development of metabolic abnormalities
c. Splanchnic vascular resistance falls in similar fashion to overall
systemic vascular resistance
d. Despite normal mechanisms of intrinsic expansion of the circulating
blood volume, exogenous volume resuscitation is necessary
Answer: d

The clinical findings in sepsis and septic shock represent the host
response to infection. Gram-positive and gram-negative bacteria,
viruses, fungi, rickettsiae, and protozoa have all been reported to
produce a clinical picture of septic shock, but the overall response is
independent of the specific type of invading organism. Septic shock
develops as a consequence of the combination metabolic and circulatory
derangements accompanying the systemic infection. It appears that the
circulatory deficits are preceded by the metabolic abnormalities
induced by infection. In fact, the circulatory changes in hyperdynamic
sepsis appear to be an adaptive response to the underlying metabolic
dysfunction. Cardiac output is high and systemic vascular resistance low
in hyperdynamic septic shock. However, splanchnic vasoconstriction is
pronounced even in the absence of systemic hypotension and even
though systemic vascular resistance is reduced. Expansion of circulating
blood volume can occur through either transcapillary refill or fluid
resuscitation. Due to the ongoing inflammatory mediator-induced
increases in capillary permeability and continued loss of intravascular
volume, exogenous volume resuscitation must be provided to restore
venous return and ventricular filling.

223 Which of the following statement(s) is/are true concerning tumor


necrosis factor (TNF)?
a. TNF is a product of activated macrophages secreted in response to
contact with endotoxin or lipopolysaccharide, antibody complexes, or
inflammatory stimuli
b. The liver and gut appear to be a major source of TNF following
hypoperfusion
c. Circulating levels of TNF correlate well with severity of tissue injury
in shock
d. Recently completed clinical trials of anti-TNF antibody in septic
patients shows a marked improvement in survival
Answer: a, b

Tumor necrosis factor (TNF), a protein product of activated


macrophages, is secreted in response to contact with endotoxin or
lipopolysaccharide, antibody complexes, or other inflammatory stimuli.

120 of 167 11-Aug-15 10:57 PM


MedCosmos Surgery: MCQ : General Surgery http://medcosmossurgery.blogspot.com/search/label/MCQ : General Surgery

Elevation of serum levels of TNF have been reported shortly after


experimental trauma and shock, however, documentation of elevated
circulating levels of TNF in human shock is less clear. Furthermore,
circulating levels of TNF cannot be correlated with severity of tissue
injury or shock. This variability is thought to be due to rapid clearance
and uptake by membrane receptors and by soluble membrane receptors
that are released from multiple cells following stress and injury.
Following hypoperfusion the liver and gut appear to be the major source
of TNF that is rapidly cleared but responsible for inducing hepatocyte
changes following shock. The release of breakdown products and escape
of bacterial and endotoxin through the damaged mucosal barrier of the
gut following shock allows or induces activation of tissue-fixed
macrophage (Kupffer cell) of the liver which then produces secondary
inflammatory mediators contributing to the post-resuscitation clinical
response and inflammatory mediator activation seen in the systemic
inflammatory response syndrome.
TNF is central to inflammatory response, particularly in sepsis and
following endotoxemia or bacteremia. TNF also induces secondary
inflammatory responses through direct interaction with specific
membrane receptors, TNF-r. Treatment with anti-TNF antibody in the
experimental setting protects animals from the deleterious effects of
lethal bacteremia and endotoxemia. However, recently completed
clinical trials in septic patients utilizing infusion of monoclonal
antibodies to the TNF molecule have shown no overall survival benefit.

224 Which of the following statement(s) is/are correct concerning the


immunoinflammatory response to shock?
a. The anaphylactoxins, C3a and C5a, are products of activation of only
the classical pathway of the compliment cascade
b. Eicosanoids, such as prostaglandins are stored in platelets and
endothelial cells and released in response to inflammatory stimuli
c. Thromboxane and PGI2 have similar effects
d. Platelet-activating factor can be released by both circulating and
fixed tissue cells
Answer: d

Inflammatory mediators have recently been recognized as playing a


significant role in the clinical manifestations and progression of shock
and the development of subsequent complications. These mediator
systems function primarily as parcrine and autocrine agents in the local
environment and are not usually detectable systemically. The
over-expression and systemic dissemination of these mediators produces
the toxic autodestructive processes underlying multiorgan failure
syndrome with attendant high mortality. The compliment cascade is
activated in shock and tissue injury through both the classical and
alternative pathways. Activation of either pathway results in generation
of the anaphylatoxin, C3a and C5a, soluble products with potent
systemic hemodynamic effects. The eicosanoids, which include the
prostaglandins and leukotrienes are formed acutely from arachidonic
acid released from the membrane phospholipid by phospholipase A2.
Eicosanoids are not stored in any measurable level and are generated as

121 of 167 11-Aug-15 10:57 PM


MedCosmos Surgery: MCQ : General Surgery http://medcosmossurgery.blogspot.com/search/label/MCQ : General Surgery

needed from readily available arachidonic acid in response to various


inflammatory phenomena. Platelets, white cells, and endothelial cells
are a rich source of these compounds. Thromboxane (TXA2) is the major
arachidonic acid metabolite elaborated by platelets. TXA2 induces
intense vasoconstriction, platelet aggregation and degranulation,
neutrophil margination in the microcirculation and bronchial
constriction. PGI2, the major arachidonic acid metabolite formed by
endothelial cells, serves a check against actions of TXA2. PGI2 is a
vasodilator and a potent inhibitor of platelet aggregation. Platelet
aggravating factor is a potent phospholipid mediator released by
neutrophils, platelets, macrophages and endothelial cells in response to
ischemia, tissue injury and sepsis. Its effects include decreased cardiac
function, increased pulmonary vascular resistance, bronchoconstriction,
peripheral vasodilatation, and increased vascular permeability.

225 Which of the following physical findings are associated with the
various classes of hemorrhagic shock?
a. Mild shock (< 20% blood volume): Pallor, cool extremities, diminished
capillary refill and diaphoresis
b. Moderate shock (20%–40% blood volume): All of the above plus
tachycardia and hypotension
c. Severe shock (> 40% blood volume): Systemic hypotension, changes in
mental status, tachycardia, oliguria
d. All of the above
Answer: a, c
PHYSICAL FINDINGS IN HEMORRHAGIC SHOCK*

Moderate
Mild (<20% (20%-40% Severe(>40%
Blood Volume) Blood Volume) Blood Volume)
Pallor Pallor Pallor
Cool extremities Cool extremities Cool extremities
Diminished capillary Diminished capillary Diminished capillary
refill refill refill
Diaphoresis Diaphoresis Diaphoresis
Collapsed Collapsed Collapsed
subcutaneous subcutaneous subcutaneous
veins veins veins
Tachycardia Tachycardia
Oliguria Oliguria
Postural Hypotension
hypotension Mental status
changes

* Alcohol or drug intoxication may alter physical findings.

226 A 68-year-old male who underwent a repair of an abdominal aortic


aneurysm 5 days ago, develops tachycardia, tachypnea, hypotension
with cool, pale, mottled cyanotic extremities. He is agitated and
complains of shortness of breath. Which of the following statement(s)

122 of 167 11-Aug-15 10:57 PM


MedCosmos Surgery: MCQ : General Surgery http://medcosmossurgery.blogspot.com/search/label/MCQ : General Surgery

is/ are correct concerning his diagnosis and management?


a. Myocardial ischemia secondary to preexisting coronary artery disease
is most likely the underlying cause of this problem
b. Invasive hemodynamic monitoring with a Swan-Gantz catheter will
demonstrate a low cardiac output, a high systemic vascular resistance,
and elevated cardiac filling pressures
c. The use of morphine sulphate and nitrates should be part of the
initial management
d. The primary pharmacologic treatment involves the use of moderate
doses of inotropic agents
e. Afterload reduction with nitroprusside is absolutely contraindicated
Answer: a, b, d

Intrinsic cardiogenic shock results from failure of the heart as an


effective pump. Coronary artery disease is the most common cause of
myocardial insufficiency, but contractile dysfunction may also rise as a
consequence of cardiomyopathy, myocarditis, or metabolic
abnormalities. Invasive hemodynamic monitoring often establishes a
specific nature of shock and allows appropriate treatment to be
delivered in an effective and expedient manner. Hemodynamic findings
consistent with cardiogenic shock include a low cardiac output and high
systemic vascular resistance, with elevated cardiac filling pressures.
The initial measures in the management of cardiogenic shock include
the administration of supplemental oxygen, mechanical ventilation (as
needed), and appropriate treatment of dysrhythmias. Hypotension
usually precludes the use of morphine sulfate and nitrates, drugs
typically used in simple congestive heart failure to alleviate cardiac pain
and ameliorate pulmonary vascular congestion. The use of
beta-adrenergic agonists such as dopamine and dobutamine, in
moderate doses, offers positive inotropic support without excessive
alpha-adrenergic activity. Increasing the inotropic state of the heart
shifts the entire Starling curve upward, resulting in increased cardiac
output for each level of cardiac filling. Afterload reduction may prompt
increases in cardiac output through decreases in resistance to flow. The
use of nitroprusside or other dilators requires relative blood pressure
stability and close hemodynamic monitoring. Infusion of afterload-
reducing agents can be administered in conjunction with inotropic
support.

227 Which of the following statement(s) is/are true concerning ischemia


reperfusion injury?
a. During ischemia, ATP degradation results in increased plasma and
intracellular levels of hypoxanthine and xanthene
b. Oxygen free radicals such as the superoxide radical are involved in
the expression of the proinflammatory phenotype of endothelial cells,
macrophages and neutrophils
c. The intracellular adhesion molecule-1 (ICAN-1) contributes to injury
and disruption of the endothelial lining, with extensive capillary leak
and resultant interstitial edema
d. Animal models have demonstrated that passive immunization with
antibodies to neutrophil adhesive complex lessen the

123 of 167 11-Aug-15 10:57 PM


MedCosmos Surgery: MCQ : General Surgery http://medcosmossurgery.blogspot.com/search/label/MCQ : General Surgery

ischemic/reperfusion microvascular injury


Answer: a, b, c, d

During the ischemia and hypoperfusion phase, degradation of ATP stores


essential to maintain cell integrity and significant loss of diffusible
intracellular adenine neuclotides occurs. As ATP further degrades there
is an elevation in plasma and intracellular levels of hypoxanthine and
xanthene which upon restoration of perfusion and reoxygenation are
catalyzed by xanthine oxidase resulting in the formation of superoxide
radicals. These radicals plus others such as hydrogen peroxide and
hydroxyl radical are generated and lead to endothelial and parenchymal
cell injury through membrane lipid peroxidation and activation of
critical enzymes. These radicals have also been shown to be involved in
the expression of proinflammatory phenotype endothelial cells and on
macraphages and neutrophils. The proinflammatory phenotype of the
endothelium includes procoagulant activity and the expression of
adhesion molecules on the membrane surface, including the
intercellular adhesion molecule-1 (ICAN-1) and the selectins. The
subsequent adhesion of activated neutrophils to the endothelial leads to
an explosive oxidative burst producing additional radicals and extensive
release of proteolytic enzymes leading to injury and disruption of the
endothelial lining, extensive capillary leak, and massive interstitial
edema. Passive immunization of animals with monoclonal antibodies to
either the neutrophil adhesive complex or the endothelial selectins
dramatically lessens ischemia/reperfusion microvascular injury.

228 Which of the following statement(s) is/are true concerning the


physiology of the microvascular system?
a. Filtration of capillary fluid into the interstitial and the subsequent
reabsorption is influenced by Starling’s law of ultrafiltration
b. The most important variable controlling blood to a capillary bed is
the length of the vessel
c. Most of the resistance to systemic blood flow occurs at the arteriolar
level
d. Adrenergic vasoconstriction can arrest blood flow to an entire
capillary bed
Answer: a, c, d

Exchange of material between the vascular space and the cell of various
tissues via the interstitial space is essential for organ viability and
occurs at the capillary level. The filtration of capillary fluid into the
interstitium and its subsequent reabsorption into the post capillary
venule is governed by microvascular permeability in conjunction with
the balance between hydrostatic and oncotic pressures. The relation of
these forces to one another (and their net effects) are illustrated by
what is termed Starling’s law of ultrafiltration. In normal circumstances,
a net filtration from capillary to interstitium is effected by a relatively
higher capillary hydrostatic pressure, whereas net reabsorption from the
interstitium back into the post capillary venule occurs as hydrostatic
pressure falls and oncotic forces predominate. Although the mechanisms
controlling blood flow to the capillary bed are complicated and vary

124 of 167 11-Aug-15 10:57 PM


MedCosmos Surgery: MCQ : General Surgery http://medcosmossurgery.blogspot.com/search/label/MCQ : General Surgery

among the different tissues, certain concepts are useful. Poiseuille’s law
describes the relation between flow of fluid through a tube and the
tube length and radius, the fluid viscosity, and the pressure gradient
between ends of the tube. The radius of the tube (or vessel) is the
single most important variable, because flow is proportional to the
radius to the fourth power. Vasoconstrictive and vasodilatory influences
directly impact local blood flow, as well as flow to other tissues through
secondary effects on the systemic pressure. This secondary effect of
peripheral vasoconstriction maintains the pressure gradient for central
perfusion of the heart and brain. Systemic blood flow meets most of its
resistance at the arteriolar level. While the individual capillary radius is
significantly smaller, the vast number of capillaries offers less total
resistance. The vascular smooth muscle in arterioles has both a-and b-
adrenergic receptors. Alpha stimulation affects vasoconstriction where
beta stimulation affects vasodilatation. The efferent sympathetic fibers
innervating the precapillary resistance vessels and the venous
capacitance vessels release norepinephrine on stimulation, which
induces smooth muscle contraction and narrowing of the caliber of the
vessels. These contractions are potent enough that blood flow to entire
capillary beds can be arrested by adrenergic vasoconstriction.

229 Which of the following statement(s) is/are true concerning the


effects of MOFS?
a. Pulmonary dysfunction tends to arise early and may resolve within 7
to 10 days
b. Unless the precipitating insult has prompted oliguric acute tubular
necrosis, renal function tends to be maintained early in the course of
MOFS
c. Although hepatic dysfunction is common with MOFS, the GI tract plays
little role in this process
d. Intercurrent nosocomial infection, most commonly pulmonary, is a
common complication providing a “second hit” to the patient
Answer: a, b, d

Pulmonary dysfunction typically arises early in the development of


systemic inflammation and may represent mild relatively localized acute
lung injury or it may be a prelude to fulminant ARDS. The lung injury,
and associated dysfunction, may resolve over the initial 7 to 10 days or
persist, depending on the ongoing pathologic process. Many times a
“second hit” such as a nosocomial infection, which is most commonly
pulmonary, is a complication which can frequently worsen the
pulmonary condition. Renal function tends to be maintained early in the
course unless the precipitating insult has been prompted by a sudden
oliguric acute tubular necrosis. With persistent activation and
inflammatory mediators, glomerular filtration falls and the development
of oliguric or polyuric renal failure marks the gradual transition into
MOFS. Gastrointestinal abnormalities include ileus, stress ulceration,
diarrhea, and mucosal atrophy. Breakdown of the mucosal barrier allows
translocation of bacteria and endotoxin. Hepatic dysfunction is marked
by progressive rise in serum bilirubin levels after a latent period of
several days.

125 of 167 11-Aug-15 10:57 PM


MedCosmos Surgery: MCQ : General Surgery http://medcosmossurgery.blogspot.com/search/label/MCQ : General Surgery

230 Which of the following statement(s) is/are true concerning


hypoadrenal shock?
a. In the United States, idiopathic adrenal atrophy (Addison’s disease) is
the most common cause
b. Laboratory abnormalities include hyponatremia, hypochloremia, and
hyperkalemia
c. Fever may be seen with hypoadrenal shock
d. ACTH stimulation test is the diagnostic test of choice to confirm
hypoadrenal shock
Answer: b, c, d

Shock of a dramatic nature, poorly responsive to resuscitation, may


develop as a consequence of adrenal insufficiency. In this country,
adrenal insufficiency most commonly arises as a consequence of the
chronic therapeutic administration of high doses exogenous
corticosteroids causing adrenal suppression. Other causes include
idiopathic adrenal atrophy (Addison’s disease), tuberculosis, metastatic
disease, bilateral hemorrhage, and amyloidosis. The stress of illness,
operation, or trauma typically requires that the adrenal glands secrete
cortisol in excess of that required in the nonstressed state
(approximately 3–4 fold). Insufficiency not otherwise apparent may
manifest itself only after major physical stress. Findings associated with
adrenal insufficiency include weakness, fatigue, anorexia, abdominal
pain, nausea, vomiting, and weight loss. Surgical patients with
significant adrenal insufficiency need not present with the above
findings. More typical is the development of refractory shock,
frequently with hyperthermia, in the course of injury or illness.
Hypotension may be dramatic despite massive volume resuscitation and
pressor support. Laboratory findings suggesting hypoadrenalism include
hyponatremia, hypochloremia and hyperkalemia. The diagnosis of
adrenal insufficiency may be confirmed or excluded by means of an
ACTH stimulation test. A significant major cortisol response should be
elicited by ACTH administration.

231 Which of the following statement(s) is/are correct concerning the


cardiovascular response to shock?
a. Changes in cardiac contractile function shift the Frank Starling curve
up and down
b. Venoconstriction from skeletal muscle is a significant contributor to
the restoration of blood volume with shock
c. Arterial vasoconstriction affects all vascular beds equally
d. The total circulating blood volume is equally split between the
arterial and venous system
Answer: a

Central in the general cardiovascular response to shock is the action of


the heart itself. The principle determinants of cardiac function in the
normal heart are the volume of blood available for the heart to pump
(preload), the systolic contractile capability, and the diastolic filling of

126 of 167 11-Aug-15 10:57 PM


MedCosmos Surgery: MCQ : General Surgery http://medcosmossurgery.blogspot.com/search/label/MCQ : General Surgery

the ventricles. In hypovolemia, the two dynamic variables of cardiac


function, ventricular filling and myocardial contractility remain
paramount and determine the stroke volume. The product of stroke
volume and heart rate in turn determines the cardiac output. Increases
in ventricular end-diastolic volume, reflecting venous return, cause
ventricular distention. Ventricular distention in turn produces increased
volume output with each stroke, the Frank Starling mechanism.
Contractile function may vary independent of volume status. Changes in
the contractile function shift the Starling curve up and down, producing
increases or decreases in stroke volume for any given end-diastolic
volume. A fundamental requirement for cardiovascular function is
adequate cardiac filling, and cardiac output cannot exceed venous
return. The venous system contains nearly two-thirds of the total
circulating blood volume, including 20% to 30% within the splanchnic
venous system. Most of this volume resides in small veins, which
comprise the bulk of venous capacitance. The venous system, especially
that of the splanchnic circulation, becomes important in the physiologic
compensation to hypoperfusion because it serves as a dynamic reservoir
for the autoinfusion of blood volume involving both active and passive
mechanisms. The splanchnic circulation makes major contributions to
the maintenance of venous return, therefore, it is likely that
sympathetic venoconstriction is responsible for a portion of the blood
mobilized from the splanchnic venous circulation. Sympathetic
mediated venoconstriction in skin and skeletal muscle is probably not as
significant as a source of blood volume. Selective vasoconstriction
occurs in response to alpha adrenergic receptor stimulation with
increased sympathetic activity in shock. Sympathetic stimulation does
not cause significant vasoconstriction of either cerebral or coronary
vessels, with normal blood flow maintained in these circulations. Blood
flow to the skin is sacrificed early, followed by that to the kidneys and
splanchnic viscera.

232 Which of the following statement(s) is/are true concerning


pharmacologic agents used in the treatment of shock?
a. The primary difference between dopamine and dobutamine is the
absence of significant a adrenergic activity
b. The renal and mesenteric vasoconstrictive effects of norepinephrine
complicate and sometimes restrict its use
c. The apparent paradoxical use of vasodilators, such as nitroprusside,
in shock is indicated as a means to augment cardiac function
d. Isoproterenol with its potent b-adrenergic effect, is a particularly
useful agent in the treatment of all forms of shock
Answer: a, b, c

Therapeutic adjustments of intravascular volume (preload) and systemic


vascular resistance (afterload) form the basis of the treatment
strategies for all forms of shock. Optimal volume resuscitation should
precede measures to augment to contractile function of the heart.
Inotropic agents are used in shock when there is inadequate cardiac
output despite adequate circulating blood volume. Dopamine and
dobutamine are often times first line agents in the pharmacologic

127 of 167 11-Aug-15 10:57 PM


MedCosmos Surgery: MCQ : General Surgery http://medcosmossurgery.blogspot.com/search/label/MCQ : General Surgery

treatment of shock. Dopamine, at low doses, stimulates dopaminergic


receptors producing renal arteriolar vasodilatation with associated
increases in renal blood flow, urine output, and sodium excretion. At
moderate doses, stimulation of cardiac b-receptors produces increases
in contractility and cardiac output with little effect on heart rate or
blood pressure. At higher doses, peripheral vasoconstriction from
increasing a activity becomes more pronounced, prompting significant
increases in vascular resistance and blood pressure. Dobutamine’s
predominant effect is an increasing cardiac contractility with lesser
increases in heart rate. Some reduction of peripheral vascular resistance
may also occur. When compared to dopamine, dobutamine produces less
peripheral vasoconstriction and less chronotropic response.
Norepinephrine exerts both a and b-adrenergic effects, with a effects
being evident at lower infusion rates and a effects more prominent at
high doses. The major use of norepinephrine in current practice is in the
patient with hypotension that persists despite appropriate volume
resuscitation and the use of inotropic agents. Renal and mesenteric
vasoconstrictive effects of norephinepherine complicate its use,
especially when support is needed for significant periods of time.
Isoproterenol is a potent b-adrenergic agent. With isoproterenol,
myocardial oxygen demand is increased and diastolic coronary feeling is
limited by tachycardia or diminished diastolic pressure. Indications for
the use of isoproterenol are fairly limited, because agents with fewer
adverse effects have become available.
Vasodilators are used to augment cardiac function through optimization
of ventricular filling pressures (preload) and systemic vascular
resistance (afterload) both of which reduce demands on the
myocardium. Decreases in afterload prompt increases in cardiac output
and venodilatation contributes to decreases in pulmonary venous
pressure and central venous pressure. Hypotension, however, may
develop therefore patients must have careful constant monitoring of
arterial pressure and repeated hemodynamic measurements with a
pulmonary artery catheter.

233 Which of the following statement(s) is/are true concerning the


treatment of MOFS?
a. Prevention and therapy of MOFS requires control of the infectious or
inflammatory source
b. Restoration of normal clinical parameters such as blood pressure,
pulse rate, and urine output ensures optimal resuscitation in most
patients
c. Branch chain amino acids play and important role in the nutritional
support of the patient
d. Because of the nature of gut injury, total parenteral nutrition is
preferred for most patients with MOFS
Answer: a, c

The therapy of MOFS is directed towards interrupting the involving


pathophysiologic process and providing an optimal physiologic
environment for healing and recovery. Fundamental concerns are
control of the source of infection, inflammation or instability;

128 of 167 11-Aug-15 10:57 PM


MedCosmos Surgery: MCQ : General Surgery http://medcosmossurgery.blogspot.com/search/label/MCQ : General Surgery

restoration of microcirculatory blood flow and oxygen transfer, and the


institution of optimal supportive care. Both the prevention and therapy
of MOFS, therefore, requires source control and restoration of adequate
profusion. Resuscitation efforts are directed toward restoration of
adequate microcirculatory blood flow in all organ systems. Restoration
of normal clinical parameters such as blood pressure, pulse rate, urine
output, and acid-base balance does not ensure optimal resuscitation.
The physiologic endpoint that most closely corresponds with adequate
microcirculatory flow is the level of cardiac output and the oxygen
delivery at which oxygen consumption and lactate production remain
independent of flow.
The importance of metabolic support in the patient with MOFS cannot
be overemphasized. The malnutrition of MOFS is markedly different
than that of starvation and the nutritional requirements also differ. If
optimal quantities of appropriately formulated amino acid solutions are
given, protein synthetic rates can approach catabolic rates and the goal
of nitrogen balance can be achieved. Formulas rich in branch chain
amino acids appear to be more efficient in promoting nitrogen retention
and minimizing urea production. Whenever feasible, enteral feeding is
preferred over TPN because evidence suggests that bacterial
translocation from the gut can be limited through the use of enteral
feeds. Enteral absorption and processing of nutrients appears superior
to TPN and lessens overall complications.

234 Which of the following statement(s) is/are true concerning the


multiorgan failure syndrome (MOFS)?
a. Changes in the splanchnic and pulmonary microcirculation are critical
to the development of MOFS
b. Tissue fixed microphages, including the liver Kupffer cell, have little
role in the development of MOFS
c. MOFS represents systemic consequences of loss of homeostatic
control of local inflammation and microcirculatory hypoperfusion
d. The nature of MOFS is highly dependent upon the etiology of the
underlying problem
Answer: a, c

The nature of multiorgan failure syndrome (MOFS) is that of a diffuse


cellular injury, developing systemically as a consequence of losing
homeostatic control of local inflammation and microcirculatory
hypoperfusion. Endothelial injury, platelet aggregation and activation of
macrophages and neutrophils occur, and the clotting, fibrinolytic, kinin,
and complement cascades are activated, along with the release of
potent inflammatory cytokines. The effects of shock, resuscitation, and
reperfusion, and the subsequent development of MOFS appear to be
critically dependent on changes in the splanchnic and pulmonary
microcirculations. These vascular beds appear to be major sites of
activation of subsequent inflammatory mediator production that
underlies the diffuse systemic inflammatory response. Extensive
activation of the liver Kupffer cell and release of inflammatory
mediators coupled with the ongoing release of activated neutrophils and
by-products of activated gut macrophages is responsible for the injury

129 of 167 11-Aug-15 10:57 PM


MedCosmos Surgery: MCQ : General Surgery http://medcosmossurgery.blogspot.com/search/label/MCQ : General Surgery

to the pulmonary microcirculation and secondary induction of alveolar


macrophage and additional inflammatory mediator systems. Excessive
and persistent macrophage activation plays an essential role in MOFS
and is hypothesized to represent the penultimate step in a series of
continuous immuno-inflammatory stimulatory events, including local
hypoxia, exposure to bacteria and toxins, and mediator release from
localized areas of inflammation. When infection is the underlying or
major contributing process, the diffuse inflammatory response develops
independently of the specific type of microorganism. In noninfectious
cases, the response also appears independent of the specific underlying
cause.

235 Invasive hemodynamic monitoring using a Swan-Gantz catheter is


essential in the optimal management of patients in shock or those
suffering post-shock sequelae. Which of the following physiologic
characteristics are associated with the various forms of shock?
a. Hypovolemic shock: Decreased pulmonary capillary wedge pressure
(PCWP), decreased cardiac output, increased systemic vascular
resistance (SVR)
b. Cardiogenic shock: Increased PCWP, decreased cardiac output,
decreased SVR
c. Septic shock (hypodynamic): Decreased cardiac output, increased SVR
d. Neurogenic shock: Decreased PCWP, increased cardiac output,
decreased SVR
Answer: a, c

236 Which of the following statement(s) is/are true concerning the


relationship between cardiac function and effective blood volume?

a. A pulmonary capillary wedge pressure of 5–10 rules out fluid overload


as a cause of pulmonary edema
b. A shift to the right in the Frank-Starling curve is associated with
compromised cardiac function
c. Dilutional anemia may contribute to tachycardia even though blood
volume and filling pressures are normal
d. The sole purpose of a pulmonary artery catheter is to measure
pulmonary artery pressure and cardiac output
Answer: b, c

Although physical findings are often adequate to establish a diagnosis


and institute management of cardiac failure, direct measurement of
filling pressures of the right heart (central venous pressure) or the left
heart (pulmonary artery pressure) may be required. Placement of a
pulmonary artery catheter allows us to measure cardiac output by
thermodilution and, more importantly, to sample mixed venous blood
for saturation measurements which tell us the ratio between systemic
oxygen delivery and oxygen consumption. From all of these
measurements we can determine if cardiac output is normal for the
level of filling pressure of the left ventricle, or if contractility is
decreased. In the latter case, cardiac output will be lower than
predicted for a given level of filling pressure. In the Frank-Starling

130 of 167 11-Aug-15 10:57 PM


MedCosmos Surgery: MCQ : General Surgery http://medcosmossurgery.blogspot.com/search/label/MCQ : General Surgery

curve, if the patient is to the right of the normal range, then cardiac
function is compromised either because of valvular disease, extrinsic
pressure such as pericardial tamponade, or more commonly, a decrease
in contractility. If cardiac function and anatomy are normal, then blood
volume, filling pressure and cardiac function are related to the Starling
curve. The intake and output of fluid and salt is autoregulated to
maintain the filling pressure of the left ventricle around 10 mm Hg.
Extracellular fluid expansion is usually associated with normal blood
volume. Gross expansion of extracelluar space results in deleterious
effects if tissue edema can and often do exist with perfectly normal
blood volume. In other words, a pulmonary capillary wedge pressure of
5–10 does not rule out fluid overload as a cause of pulmonary or GI
dysfunction. In critically ill patients, the fear of hypotension and effect
of perfusion usually results in infusion of intravenous salt and water in
quantities which exceed losses. Consequently, most patients in the
Intensive Care Unit have anemia, dilutional hypoproteinemia, and a
compensatory increase in cardiac output. In response to anemia, these
patients are tachycardic, even though blood volume is normal, filling
pressures are normal, and total body extracellular fluid is excessive.

237 Which of the following statement(s) is/are true concerning methods


of nutritional support?

a. Optimal results for enteral feedings are achieved with approximately


half of calories supplied as carbohydrate and half as fat
b. Diarrhea is the most common complication of enteral feeds and is
due to the high osmolarity of the carbohydrate components
c. The hyperosmolar nature of parenteral fat solutions requires central
venous administration
d. Approximately 25–50% of calories should be provided as fat emulsion
in patients receiving total parenteral nutrition
Answer: a, b, d

Most formulas for enteral feeding range from 1.0 to 2 cal/ml and
include 3 to 7% protein. Most of the calories are supplied as glucose or
sucrose, so that the solutions have a very high osmolarity. Cramps or
diarrhea can result when these high osmolar solutions are placed into
the stomach or intestine. Diarrhea is the major complication with most
tube feeding formulas. Diarrhea can be minimized by the use of starch
or fat as an energy source in tube feedings. This can be supplied as part
of the commercial preparation or added in the form of medium chain
triglycerides or other oils. The best results are usually achieved by
supplying approximately half the calories as carbohydrate and half as
fat. In patients receiving total parenteral nutrition, energy source is
provided as carbohydrate, fat, and amino acid solutions. Parenteral
feeding with carbohydrate is limited by the sclerotic effect of
hyperosmolar solutions on veins. Fat is a more efficient energy source
and can be given through peripheral veins in concentrations of either 10
or 20%. Most intensivists favor supplementing standard total parenteral
nutrition solution with intravenous fat to provide at least 100 grams of
fat emulsion each week to preclude fatty acid deficiency. Giving up to

131 of 167 11-Aug-15 10:57 PM


MedCosmos Surgery: MCQ : General Surgery http://medcosmossurgery.blogspot.com/search/label/MCQ : General Surgery

25 to 50% of calories each day as fat emulsion may optimize the delivery
of this caloric delivery.

238 Which of the following statement(s) is/are true concerning the


autoregulation necessary to maintain oxygen consumption and oxygen
delivery?

a. A change in oxygen consumption is followed by a proportionate


change in oxygen delivery
b. A change in oxygen delivery is followed by a change in oxygen
consumption
c. Increases in oxygen delivery are due solely to an increase in cardiac
output
d. The normal ratio of oxygen delivery to consumption is 2:1
Answer: a

The relationships between oxygen consumption and oxygen delivery


represent one of the most interesting regulation systems in homeostasis.
First of all, if one of the three components of oxygen delivery is
abnormal, endogenous mechanisms regulate the other two until normal
oxygen delivery has been restored. The various combinations of
compensatory mechanisms supply adequate oxygen for systemic
metabolism through a wide range of variations in oxygen delivery. When
there is a change in oxygen consumption, there is a proportionate
change in oxygen delivery, which occurs almost immediately, mediated
completely by a change in cardiac output. Conversely, a primary change
in oxygen delivery is not followed by any change in oxygen consumption.
The normal ratio of oxygen delivery to consumption is approximately
5:1.

239 Which of the following statement(s) is/are true concerning O2


venous saturation monitoring?

a. The normal saturation of mixed venous blood is 50%


b. Mixed venous blood obtained for saturation monitoring can be
obtained from any peripheral vein
c. If arterial blood is fully saturated, the saturation of mixed venous
blood is 80%
d. In less than fully saturated blood, the difference between arterial
and venous saturation corresponds to oxygen extraction
Answer: c, d

The relationship between oxygen delivery and oxygen consumption is


reflected in the amount of oxygen in venous blood. Under normal
circumstances, oxygen delivery is 1000 cc/min and oxygen consumption
is 200 cc/min. The amount of oxygen extracted is 20% of that delivered,
and 80% of oxygen is still present in venous blood returning to the heart.
Usually arterial blood is fully saturated, and under normal
circumstances, the saturation of mixed venous blood (SVO2) will be 80%.
This measurement must be made in mixed venous blood since the

132 of 167 11-Aug-15 10:57 PM


MedCosmos Surgery: MCQ : General Surgery http://medcosmossurgery.blogspot.com/search/label/MCQ : General Surgery

relative extraction of organs served by the superior and inferior vena


cava and coronary sinus are quite different. As long as arterial blood is
fully saturated, this observation holds true regardless of the absolute
level of oxygen consumption or oxygen delivery. If the arterial blood is
less than fully saturated, the difference between arterial and venous
saturation corresponds to the oxygen extraction, hence the oxygen
delivery/oxygen consumption ratio.

240 Which of the following statement(s) is/are true concerning oxygen


kinetics in a critically ill, febrile patient?

a. Oxygen consumption will likely exceed three times normal


b. The high cardiac output and pulse rate are designed to increase
oxygen delivery
c. The hyperdynamic response may actually increase oxygen delivery to
exceed the increase in oxygen consumption
d. The patient can maintain adequate compensation as long as the
oxygen delivery/oxygen consumption rate is greater than 2:1
Answer: b, d

In critically ill patients oxygen consumption may be elevated or


depressed, but slight to moderate elevations in oxygen consumption is
the most common abnormality in critically ill patients. Oxygen
consumption will be elevated in proportion to the amount of
inflammation. A febrile patient with significant signs of septic toxicity
will typically have an oxygen delivery at 1.5 to 2 times normal. It is very
unusual for a critically ill patient to experience oxygen consumption
greater than twice normal. This occurs only in situations of severe
muscular exercise such as seizures or tetanus. During hypermetabolism,
a change in oxygen consumption is followed promptly by a proportionate
change in oxygen delivery. Hence, it is “normal” for a hypermetabolic
patient to have a high cardiac output and pulse rate. Rarely the
hyperdynamic response exceeds the increase in oxygen consumption,
reflected in a ratio higher than 5:1 and venous saturation greater than
80%. Some patients cannot mount an increased oxygen delivery in
response to increased oxygen consumption because of the combination
of hypoxemia, anemia, and myocardial failure. If this occurs, then the
oxygen delivery/oxygen consumption ratio will be less than 5:1. The
patient will compensate for this by increased oxygen extraction,
however, and the patient will remain stable as long as the ratio is
greater than 2:1.

241 Which of the following statement(s) is/are true concerning the


treatment of pulmonary interstitial edema?

a. Diuresis and blood transfusion is a valuable step


b. Salt-poor albumin leaks through the capillaries and worsens the
condition
c. Mannitol is contraindicated as a diuretic in this clinical situation
d. Isoproterenol is a poor choice as an ionotropic agent

133 of 167 11-Aug-15 10:57 PM


MedCosmos Surgery: MCQ : General Surgery http://medcosmossurgery.blogspot.com/search/label/MCQ : General Surgery

Answer: a

Treatment of pulmonary edema has two important goals, the first is to


improve oxygenation if it is impaired, and the second is to minimize
fibrosis and bacterial infection, which often accompany pulmonary
edema caused by capillary injury. The treatment of interstitial edema is
to maintain the hydrostatic pressure as low as compatible with
adequate cardiac output and to raise the oncotic pressure selectively in
the vascular space. These measures, combined with fluid restriction and
diuresis, will decrease the amount of pulmonary edema. Since it is
desirable to maintain filling pressures of the left ventricle as low as
possible while maintaining a good cardiac output, inotropic drugs to
improve left ventricular contractility are helpful. Isoproterenol or
dopamine should be used, with serial cardiac output and filling pressure
measurements. The first step in decreasing pulmonary edema is to
decrease the pulmonary capillary hydrostatic pressure as low as is
compatible with an adequate cardiac output. This is done by diuresis
and fluid restriction. As the patient falls behind in blood volume, signs
of hypovolemia may appear. Blood volume is then replenished with a
fluid that stays in the vascular space. Packed red cells are ideal for this
application. When the hematocrit is normal, concentrated salt-poor
albumin should be used. This hyperoncotic fluid replenishes the blood
volume by attracting interstitial fluid from throughout the body into the
vascular space and supplements diuresis. This technique is useful even
in the septic patient who may have increased capillary permeability and
may loose albumin from the vascular space at a rapid rate. Even if
albumin “leaks out”, the short term effects of expanding blood volume
and decrease in edema will appear.

242 Which of the following statement(s) is/are true concerning the


pathophysiology of gas exchange?

a. Hypoventilation in relation to perfusion can result in an oxygen


saturation of less than 100%
b. Diffusion block and / mismatch can almost completely be overcome
by breathing 100% oxygen
c. Transpulmonary shunting does not occur under normal circumstances
d. The normal arterial oxygen saturation should be 100%
Answer: a, b

Under normal conditions, red blood cells in the pulmonary capillaries


become fully saturated and oxygen dissolves in plasma resulting in blood
PO2 of 100 and O2 saturation of 100%. This equilibration may be
disturbed by hypoventilation in relationship to the perfusion (/
mismatch), diffusion block caused by interstitial fibrosis, or perfusion of
nonventilated alveoli. Diffusion block and / mismatch can almost be
completely overcome by breathing 100% oxygen, hence hypoxemia
during exposure to high alveolar PO2 is caused by total / mismatch,
so-called transpulmonary shunting or venous ad mixture. Under normal
circumstances, about 5% of the blood entering the left atrium has been
shunted away from the pulmonary capillaries, either as the result of

134 of 167 11-Aug-15 10:57 PM


MedCosmos Surgery: MCQ : General Surgery http://medcosmossurgery.blogspot.com/search/label/MCQ : General Surgery

bronchial nutritive blood flow or through thebesian veins opening


directly into the left side of the heart. This phenomenon, combined
with a normal minor / mismatch associated with breathing at rest and
positional changes in pulmonary blood flow, result in the fact that
normal arterial PO2 is 90–100 mm Hg and the normal O2 saturation is
98%.

243 Which of the following statement(s) is/are true concerning CO2


transfer in the lung?

a. Carbon dioxide excretion is a direct function of alveolar ventilation


b. Normally end tidal CO2 should be identical to PaCO2
c. The gradient between end tidal and arterial CO2 can be an indirect
measure of nonperfused alveoli
d. Positive pressure ventilation under normal airway pressures creates a
significant end tidal PaCO2 gradient
Answer: a, b, c

The amount of carbon dioxide excretion is directly related to alveolar


ventilation. While oxygenation is a function of matching blood flow to
alveoli, carbon dioxide excretion is a direct function of ventilation or
hyperventilation of alveoli with some blood flow. Normally the end tidal
CO2 represents mixed alveolar gas which is at equilibrium with
pulmonary capillary blood, hence with arterial blood. Therefore, the
end tidal CO2 and the PaCO2 should be identical. End tidal CO2
measurement is a very useful continuous measurement of PaCO2 which
can be used as a monitor when the lung is normal, as in ventilator
weaning. Furthermore, the gradient between end tidal and arterial CO2,
when it is large, acts as an indirect measure of nonperfused alveoli
and/or compression volume. In patients who are ventilated with positive
pressure ventilation, a significant end tidal PaCO2 gradient occurs only
when peak airway pressures are very high (over 30 cm H2O) and the
compression volume is a significant component of each exhaled breath.

244 Which of the following statement(s) is/are true concerning the


assessment of protein reserve?

a. Conventional serum proteins such as albumin and globulin are early


indicators of malnutrition
b. The total lymphocyte count reflects immune status and not nutrition
c. Antigen skin testing reflects patient immunity and not nutrition
d. Measurement of urea excretion in urine can be used as a
measurement of protein breakdown
Answer: d

Since protein is the functional and structural chemical of the body, most
nutritional assessment techniques are estimates of protein reserves. The
actual nitrogen balance can be measured by measuring the amount of
nitrogen excreted. This is most conveniently done by measuring the
amount of urea excreted in the urine, assuming that urea constitutes

135 of 167 11-Aug-15 10:57 PM


MedCosmos Surgery: MCQ : General Surgery http://medcosmossurgery.blogspot.com/search/label/MCQ : General Surgery

85% of the total nitrogen excretion. Knowing nitrogen excretion, the


amount of protein catabolized can be estimated and compared with the
amount of protein ingested by the patient. Indirect assessments of
protein reserves are based on single measurements of body substances
that are dependent on rapid protein synthesis for maintenance of
normal levels. Conventional serum proteins such as albumin and globulin
are not affected by malnutrition until it is very severe. Proteins such as
prealbumen and transferrin, which turn over more rapidly, are better
indicators of protein status. Lymphocytes are rapidly destroyed and
protein is required for the formation of new cells. Consequently, the
absolute lymphocyte count is a useful measure of the status of protein
reserves. The lymphocyte count is considered by some the best single
“static” measurement characterizing nutritional status. Protein is also
required for synthesizing the cells and mediators involved in skin test
reactivity. Although skin test reactivity is a manifestation of
lymphocyte-mediated immunity, its usefulness in patient assessment is
probably that of assessment of the inflammatory response than
lymphocyte activity per se. Some chronically and acutely malnourished
patients convert from reactive to anergic, and reactivity can be
restored by nutritional repletion.

245 Useful steps to optimize systemic oxygen delivery include:

a. Maintaining mean arterial blood pressure between 50 and 90 mm Hg


b. Optimizing PEEP levels by monitoring mixed venous saturation
c. Turning the patient prone
d. Sedation or paralysis
Answer: a, b, c, d

Optimizing systemic oxygen delivery in relationship to oxygen


requirement is the primary goal of management. Improving oxygenation
of the blood itself by improving alveolar inflation is only one of the
steps in optimizing oxygen delivery. Equally or more important are
treating anemia and optimizing cardiac output. Cardiac output should
be optimized to maintain delivery of four to five times consumption. In
general, this means avoiding those factors which decrease cardiac
output, rather than actively trying to increase cardiac output. Blood
pressure should be maintained high enough to provide coronary
perfusion (over 50 mm Hg mean pressure) but not so high as to limit left
ventricular function (over 90 mm Hg mean arterial pressure). Alveolar
collapse is treated by cleaning the airways, avoiding 100% oxygen, and
moving fluid from the lung or chest, and finally by the use of positive
end exploratory pressure to hold open those alveoli which have been
opened by other measures. The optimal level of PEEP is that level that
maintains arterial oxygenation but does not decrease venous return or
cardiac output. This optimal level is best determined by monitoring
mixed venous saturation. Another step in optimizing lung function is to
take advantage of the gravitational effects on pulmonary blood flow by
turning the patient prone or to a full lateral position to direct blood
flow to areas of optimal alveolar function. This step will often result in
an opening in the closed posterior alveoli which have been compressed

136 of 167 11-Aug-15 10:57 PM


MedCosmos Surgery: MCQ : General Surgery http://medcosmossurgery.blogspot.com/search/label/MCQ : General Surgery

by the weight of the fluid in the lungs. At the same time that oxygen
delivery is optimized, oxygen consumption should be decreased to
normal or even below normal if necessary. Treating infection, providing
adequate sedation, and establishing muscular paralysis decrease oxygen
consumption, and decrease the need for oxygen delivery.

246 Phases of multiorgan failure will include:

a. Generalized increased capillary permeability


b. A hypermetabolic state
c. Organ malfunction
d. All of the above
Answer: d

Clinically the multiple-organ failure patient progresses through


well-defined phases. These phases include: Phase 1—a generalized
increased capillary permeability resulting in edema, weight gain, and
intravenous volume replacement, increased protein concentration in
urine and lymph. Although the pulmonary microvasculature has been
most thoroughly studied, it is apparent that the lung is simply the most
obvious end organ in a generalized permeability defect. Phase 2—A
hypermetabolic state, with increased oxygen consumption and a
compensatory increase in oxygen delivery characterized by tachycardia
and high cardiac output. This condition following systemic ischemic and
reperfusion is similar to hypermetabolism following endotoxemia,
localized sterile inflammation, and infusion of stress hormones,
suggesting a common mechanism. Phase 3—Organ malfunction due to
localized edema and cellular injury, particularly in the kidney, liver,
brain, and host defense system. Hemorrhagic shock predisposes to
bacterial translocation and endotoxin absorption from the intestine.
Phase 4—In the absence of systemic sepsis, organs may recover to
normalcy or may be irreversibly damaged, leading to a need for chronic
support. If the organ failure phases lead to systemic infection or
irreversible tissue damage in the lung or brain, the death of the entire
organ is likely.

247 Which of the following statement(s) is/are true concerning oxygen


consumption (O2)?

a. O2 is normally 100–120 cc2/m2/min


b. Resting O2 is controlled by the level of thyroid and catecholamine
hormones
c. Under steady state conditions the amount of oxygen consumed
exceeds the amount of oxygen taken up by the pulmonary capillaries
d. O2 is dependent on the status of pulmonary function
Answer: a, b

Oxygen consumed in the process of metabolism is expressed as the


volume of oxygen per minute (O2). O2 is normally 100–120 cc2/m2/min,
or 200 cc/min for a typical adult. Resting O2 is a function of

137 of 167 11-Aug-15 10:57 PM


MedCosmos Surgery: MCQ : General Surgery http://medcosmossurgery.blogspot.com/search/label/MCQ : General Surgery

metabolizing body cell mass, with fine tuning control provided by the
level of thyroid and catecholamine hormones. O2 decreases under
conditions of hypothermia, paralysis, and hypothyroidism. O2 increases
during exercise or muscular activity, hyperthermia, profound
hypothalamic injury, hyperthyroidism, catecholamines, and
inflammatory mediators, particularly the interleukin cytokines. Under
steady state conditions, the amount of oxygen consumed in systemic
metabolism is exactly equal to the amount of oxygen taken up by the
pulmonary capillaries via the airway. This is true regardless of the status
of pulmonary function or dysfunction, so we measure O2 across the lung
and assume that it is exactly the amount consumed in the systemic
metabolism.

248 Which of the following statement(s) is/are true concerning the


outcome in patients with acute renal failure?

a. Mortality for ischemic acute tubular necrosis without other organ


failure is approximately 6%
b. Multiple organ failure complicated with acute renal failure is
associated with mortality ranging from 50% to 90%
c. Recovery of renal function after six weeks is unlikely
d. There is no difference in survival between oliguric and nonoliguric
renal failure
Answer: a, b, c

Survival of patients with acute renal failure is a function of the


successful treatment of the primary disease from which the renal failure
was derived. The mortality for ischemic acute tubular necrosis without
organ failure has been reported at approximately 6%. By contrast,
mortality of multiorgan failure complicated by acute renal failure
ranges from 50% to 90%. In patients who survive the acute phase of
illness, recovery of renal function after acute renal failure is dependent
on the type and extent of injuries to the renal parenchyma. If renal
function is not returned after six weeks, recovery is unlikely. Nonoliguric
renal failure is usually limited in its extent and is almost always
reversible.

249 Which of the following statement(s) is/are true concerning oxygen


delivery?

a. The amount of oxygen delivered to peripheral tissues is dependent


upon the oxygen content in arterial blood and cardiac output
b. Oxygen content is commonly measured in arterial blood
c. The normal arterial-venous difference is 4 cc O2/dL
d. Normal systemic oxygen delivery for a typical adult is approximately
1000 cc/min
Answer: a, c, d

The amount of oxygen that is delivered to peripheral tissues is the


product of the oxygen content in arterial blood times the cardiac

138 of 167 11-Aug-15 10:57 PM


MedCosmos Surgery: MCQ : General Surgery http://medcosmossurgery.blogspot.com/search/label/MCQ : General Surgery

output. Normally, oxygen content of arterial blood is approximately


20 cc/dL, and the normal cardiac index is 5 L/min. Therefore, the
normal systemic oxygen delivery is approximately 1000 cc/min.
Although oxygen content is the most important measure of oxygen in
the blood, PO2 and oxyhemoglobin saturation is more commonly
measured in the Intensive Care Unit, hence it is necessary to convert
between these measurements. The normal oxygen content of venous
blood is 16 cc/dL. Hence, the normal arterial-venous difference is 4 cc
O2/dL.

250 Which of the following statement(s) is/are true concerning carbon


dioxide kinetics?

a. The amount of carbon dioxide produced is equivalent to the amount


of oxygen consumed
b. Carbon dioxide levels in blood, present mostly as a bicarbonate ion,
can quickly change
c. Normally the amount of carbon dioxide excreted through the lung is
exactly equal to the amount of carbon dioxide produced in peripheral
tissues
d. The amount of carbon dioxide excreted is a function of ventilation of
perfused alveoli
Answer: a, c, d

The total amount of carbon dioxide produced by systemic metabolism is


roughly equivalent to the amount of oxygen consumed (100–120
cc/m2/min, 200 cc/min in a typical adult). CO2 production is increased
or decreased by each of the factors that causes an increase or decrease
in oxygen consumption. Most of the carbon dioxide in blood is present as
bicarbonate ion which cannot be changed quickly. However, the
metabolically produced CO2 is mostly present as dissolved carbon
dioxide, added to the blood in the peripheral tissues and excreted in the
lung. In a steady state, the amount of carbon dioxide excreted through
the lung is exactly equal to the amount of carbon dioxide produced in
peripheral tissues. The amount of carbon dioxide excreted is a function
of ventilation of perfused alveoli (i.e. the alveolar ventilation/min).

251 Which of the following result in a decrease in functional residual


capacity?

a. Shallow breathing
b. Partial airway occlusion
c. Absorption atelectasis
d. Hemothorax
Answer: a, b, c, d

A decrease in functional residual capacity is caused by incomplete


alveolar inflation related to 1) shallow breathing; 2) partial or complete
airway occlusion, which may be generalized (as in bronchospasm) or
localized (as in gastric aspiration); 3) absorption atelectasis, which

139 of 167 11-Aug-15 10:57 PM


MedCosmos Surgery: MCQ : General Surgery http://medcosmossurgery.blogspot.com/search/label/MCQ : General Surgery

occurs when oxygen is substituted for nitrogen in the inspired gas; or 4)


conditions in which air or fluid is occupying a potential alveolar space in
the chest such as pneumothorax, hemothorax, or pulmonary edema.

252 Which of the following statement(s) meet the criteria for organ
failure?

a. Bilirubin greater than 5 mg/dl


b. Creatinine greater than 3 mg/dl
c. Alveoloarterial O2 gradient greater than 300 mm Hg
d. Glasgow Coma score less than 10
Answer: a, b, c, d

Multiple organ failure is defined by dysfunction of two or more of the six


vital organ systems: cardiovascular, respiratory, nervous system, renal,
liver, and host defenses.

253 Which of the following statement(s) is/are true concerning


pulmonary edema?

a. Pulmonary edema effectively narrows bronchi and increases


pulmonary vascular resistance
b. Ventilation and perfusion are decreased equally
c. Positive pressure ventilation improves gas exchange by decreasing
lung edema
d. The condition is frequently caused by decreased plasma protein
levels
Answer: a

The causes of pulmonary edema are: 1) increased hydrostatic pressure;


2) increased capillary permeability and 3) decreased plasma oncotic
pressure. The latter, however, is rarely a problem unless the
concentration of plasma protein is very low. When fluid begins to collect
in the lung interstitium, it migrates to the loose areolar portion of the
lung microanatomy that surround the small bronchioles and pulmonary
arteries. The edema in these areas has the effect of narrowing bronchi
and increasing resistance in the pulmonary vasculature. This will
decrease both ventilation and perfusion in the edematous area, but
ventilation is often affected more than blood flow, resulting in a
decreased / ratio, with all of its attendant effects on gas exchange.
Ventilator treatment of pulmonary edema which increases airway
pressure tends to hold the alveoli open, spreading out the space
available for water accumulation and overcomes the effect of small
bronchial occlusion. Positive pressure ventilation does not, therefore,
affect the amount of edema in the lung, only its manifestations.

254 Which of the following statement(s) is/are true concerning various


causes of acute renal failure?

a. Acute tubular necrosis is the most common pathologic finding of

140 of 167 11-Aug-15 10:57 PM


MedCosmos Surgery: MCQ : General Surgery http://medcosmossurgery.blogspot.com/search/label/MCQ : General Surgery

acute renal failure


b. Drug-induced renal failure is compounded in situations of
hypovolemia
c. Myoglobin-induced renal failure can be prevented using diuretics and
alkalization of urine
d. The incidence of radiographic contrast dye-induced renal failure
occurs independent of preexisting conditions
Myoglobin is a direct nephrotoxin
Answer: a, b, c

Acute tubular necrosis results from ischemia to the renal parenchyma


and is the most common pathologic finding of acute renal failure. In
conditions of diminishing renal blood flow, perfusion to the kidneys is
first maintained by vasomotor responses which dilate the afferent
arteriole and constrict the efferent arteriole. As continued hypotension
occurs, the renin-angiotensin system is activated and vasoconstriction of
the afferent arteriole occurs which exacerbates corticohypoperfusion.
Pigment nephropathy is a common cause of acute renal failure occurring
after trauma, burns, operations, or hemodynamic catastrophe. With
ischemia or blunt injury to large muscles, myoglobin is released into the
circulation. In the kidney, it is filtered from blood and reabsorbed by
the tubule. Although myoglobin is not a direct nephrotoxin, in the
presence of aciduria, myoglobin is converted to ferrihemate, which is
toxic to renal cells. Prevention of myoglobin-induced renal failure may
include the use of diuretics and alkalinization of urine. Drug-induced
acute renal failure is responsible for approximately 5% of all cases of
acute renal failure. Through normal reabsorption and secretion, the
kidney is exposed to high concentrations of drugs and solutes, which
may be toxic. This problem is compounded by hypovolemia, which
causes increased reabsorption of water and solutes and exposes the
lumen to even higher concentrations of toxins. The incidence of
radiographic contrast dye induced nephropathy is approximately 1 to
10% and may be predicted according to a number of risk factors which
include contrast load, age, preexisting renal insufficiency, and diabetes.
The incidence in patients with normal renal function is significantly
lower at 1% to 2%.

255 The patient requires renal replacement therapy. Which of the


following statement(s) is/are true concerning the differences between
hemodialysis and continuous arteriovenous hemodialysis (CAVHD)?

a. Anticoagulation is not required for CAVHD


b. Hemodynamic instability will be a particular problem with both
techniques
c. Both techniques will decrease serum urea ni+62trogen levels
d. CAVHD will likely result in better removal of excessive volume
Answer: c, d

256 Which of the following statement(s) is/are true concerning


continuous arteriovenous hemofiltration (CAVH)?

141 of 167 11-Aug-15 10:57 PM


MedCosmos Surgery: MCQ : General Surgery http://medcosmossurgery.blogspot.com/search/label/MCQ : General Surgery

a. The technique runs continuously


b. It is not associated with the hemodynamic instability
c. Systemic heparin anticoagulation is necessary
d. Fluid balance and correcting electrolyte abnormalities takes several
days
Answer: a, b

Continuous arteriovenous hemofiltration (CAVH) is an extracorporial


filtration technique that removes extracellular fluid across a synthetic
membrane via hydrostatic pressure gradient created between the
indwelling arterial and venous catheters. Arteriovenous access is
accomplished by percutaneous cannulation of femoral artery and vein
with a low incidence of complications. Although full systemic
anticoagulation is not necessary for CAVH, heparinization of the
extracorporial circuit is required. CAVH is run continuously for as many
days as renal replacement is required. Experience with CAVH has
demonstrated very little or no hemodynamic instability with treatment
of critically ill renal failure patients. The stable nature of this therapy is
attributed to a slow and continuous fluid and solute removal and to the
fact that the membrane does not induce compliment activation when in
contact with blood. Fluid balance and serum electrolyte concentrations
can be titrated to any level in a matter of hours by manipulating the
composition and rate of replacement solution. Solute clearance with
CAVH is limited by the ultrafiltration and replacement fluid exchange
rate. In patients with high urea generation rates, solute removal with
CAVH may be inadequate and variations of the technique may be used to
enhance clearance.

257 A 64-year-old diabetic patient develops acute renal failure following


an aortic aneurysm repair. Which of the following statement(s) is/are
true concerning his diagnosis and management?

a. Resting energy expenditure will likely be less than would be expected


for a patient with normal renal function
b. Maintenance of positive energy balance reduces protein catabolism
and makes the management of renal failure easier
c. Expected metabolic abnormalities include hyperkalemia,
hypercalcemia, and metabolic alkalosis
d. A nonoliguric renal failure is usually associated with a better outcome
Answer: b, d

In patients with nonoliguric renal failure, treatment may differ little


from that required for identical patients with normal renal function.
Management of fluids, solutes, and nutrition is usually unaffected by
nonoliguric renal failure, although BUN may be elevated. The extent of
renal dysfunction is limited and almost always reversible. The use of
renal replacement therapies is rarely necessary. Acute renal failure can
result in severe derangements in electrolyte and acid-based physiology.
Of all electrolyte abnormalities that might occur, hyperkalemia is the
most serious. Other electrolyte abnormalities such as hyponatremia,
hyperphosphatemia, hypocalcemia, and metabolic acidosis are common

142 of 167 11-Aug-15 10:57 PM


MedCosmos Surgery: MCQ : General Surgery http://medcosmossurgery.blogspot.com/search/label/MCQ : General Surgery

and must be monitored carefully.


The metabolic requirements of a patient with acute renal failure are
those of a critically ill hospitalized patient. The actual measurements of
resting energy expenditure has shown that caloric requirements for
multiorgan failure patients with renal failure are often 50% above
normal, healthy individuals. Although acute renal failure may require
fluid restriction, providing adequate nutrition is an important aspect of
their treatment. Positive energy balance may make management of
uremia and hyperkalemia less difficult. By providing adequate calories,
endogenous protein catabolism with resultant generation of urea and
release of potassium can be avoided. Maintenance of positive energy
balance with glucose and lipids should reduce protein catabolism, urea
generation, and hyperkalemia.

258 Which of the following statement(s) is/are true concerning various


energy sources?

a. Carbohydrate is the most efficient source of energy


b. Endogenous fat is the major source of energy during starvation
c. The respiratory quotient of carbohydrate is greater than either fat or
protein
d. Ketones can be used as a source of energy during starvation
Answer: b, c, d

The major sources of energy are carbohydrates and fats. Carbohydrates


are a major source of energy during normal, non-starving existence. The
brain, the red cells, and some other organs are obligate glucose users.
The brain and red cells can develop the capacity to use ketones as an
energy source, a process known as starvation adaptation. Fat is the
most efficient source of energy. Fat produces 9 calories of energy per
gram of substrate metabolized while carbohydrate produces only 4
calories. The respiratory quotient represents the number of molecules
of carbon dioxide for each molecule of oxygen consumed. For
carbohydrates it is 1.0, whereas for fat, this respiratory quotient is 0.7.
Endogenous fat is the major source of energy during starvation. The
glycogen storage is basically depleted after a day of fasting and fat
becomes a major energy source with protein breakdown supplying
glucose through the process of gluconeogenesis.

259 Which of the following statement(s) is/are true concerning the


treatment of multisystem organ failure?

a. Forced diuresis with negative fluid balance may improve survival and
acute respiratory failure
b. The titration of ionotropic drugs based on desired blood pressure
optimizes the results.
c. Nutritional support should be withheld for several days until the
patient’s condition stabilizes
d. Continuous arteriovenous hemofiltration is preferred to intermittent
hemodialysis for most critically ill patients

143 of 167 11-Aug-15 10:57 PM


MedCosmos Surgery: MCQ : General Surgery http://medcosmossurgery.blogspot.com/search/label/MCQ : General Surgery

e. Hepatic failure should be treated specifically with pharmacologic


manipulation
Answer: a, d

The important principles in the management of multiple organ failure


are to avoid further episodes of local or systemic ischemia and to keep
the brain viable by pharmacologic or mechanical support of the failing
organs until organ recovery occurs. Respiratory failure is treated by
mechanical assistance for lung inflation and ventilation and by
decreasing lung edema as much as possible. Airway intubation is usually
required. There is now good evidence that forced diuresis and negative
fluid balance is associated with improved survival and acute respiratory
failure. Cardiac failure is treated with inotropic drugs. Although
ionotropic drugs are usually titrated to achieve a desired arterial blood
pressure, it is more sensible to titrate ionotropes to achieve a normal
oxygen delivery/oxygen consumption ratio. Pulmonary artery pressure
and mixed venous saturation monitoring are essential for intelligent
management of the patient with severe respiratory or cardiac failure.
Adequate nutrition is also important for recovery from organ failure.
Renal failure is treated by mechanical substitution of renal function.
Although hemodialysis and peritoneal dialysis can serve this purpose,
each has a significant drawback in the critically ill, multiple organ
failure patient. Continuous arteriovenous hemofiltration (CAVH) and
continuous arteriovenous hemodialysis (CAVHD) are the methods of
choice for renal replacement therapy. Hepatic failure often occurs as
part of the multiple organ failure syndrome but unfortunately there is
no specific treatment.

260 Which of the following statement(s) is/are true concerning


pulmonary mechanics?

a. The standard compliance or volume pressure curve is measured


during lung inflation
b. The decreased compliance in acute respiratory failure occurs because
the lung is smaller not stiffer
c. In acute respiratory failure, higher pressures are required to achieve
the same level of inflation
d. Areas of normal lungs are more vulnerable to overdistention which
may lead to progressive lung dysfunction
Answer: b, c, d

The standard compliance or volume pressure curve is drawn by


measuring volume and pressure at stages of lung deflation after total
inflation. The decreased compliance in acute respiratory distress
syndrome occurs because the lung is smaller, not stiffer. In acute
respiratory failure, the cause of decreased compliance is almost always
associated with a decrease in functional residual capacity (FRC). The
decreased FRC represents lost alveoli which are either collapsed or
filled with fluid but still perfused with blood. Because the lung is
smaller, the compliance curve has shifted to the right and much higher
pressures are required to achieve the same level of inflation. Lung

144 of 167 11-Aug-15 10:57 PM


MedCosmos Surgery: MCQ : General Surgery http://medcosmossurgery.blogspot.com/search/label/MCQ : General Surgery

damage can be caused by high airway pressure, so that overdistension is


not merely inefficient but actually detrimental. Since the most normal
areas of lung have the best compliance, they are most vulnerable to
overdistension, contributing to the steady progression of lung
dysfunction in patients ventilated at high peak pressure.

261 Which of the following statement(s) is/are true concerning the use
of a ventilator in the treatment of respiratory failure?

a. The assist-control mode is appropriate in the paralyzed patient


b. Peak inspiratory pressure should be optimized at a level in excess of
40 cm H2O
c. A patient receiving excessive carbohydrate as a nutritional support
may have an elevated minute ventilation and may tire with spontaneous
breathing
d. In general, weaning requires an adequate inspiratory force, vital
capacity, and a minute ventilation less than 10 L/min
Answer: c, d

Most intensivists favor setting the ventilator on the assist-control mode


at a low sensitivity. In this fashion, the patient breathes at a rate that
regulates the PaCO2 to normal, but each breath is mechanically
assisted, providing maximal inflation. The volume of each breath is set
by limiting the maximal pressure or maximal volume of each breath.
Whichever method is used, the peak inspiratory pressure should not
generally exceed 40 cm H2O. If the patient is comatose or paralyzed,
the assist mode cannot be used and the rate is set in addition to the
volume.
Adequate weaning indices are: inspiratory force greater than 20 cm
H2O, vital capacity twice the tidal volume, adequate gas exchange at
assisted ventilation at FiO2 of 0.3 and 5 cm H2O of PEEP, and minute
ventilation less than 10 L/min. If the patient is hypermetabolic or is
receiving excessive carbohydrate as nutritional support, the minute
ventilation will be elevated, even during assisted mechanical
ventilation. If this is the case, the patient will tire rapidly on
spontaneous breathing.

262 Which of the following statement(s) is/are true concerning the


estimation and measurement of energy requirements in the critically ill
patient?

a. One can only estimate energy expenditure with actual measurement


not technically possible
b. The amount of oxygen absorbed through the lungs is equal to the
amount of oxygen consumed by metabolic processes
c. Metabolic rate, normalized to body surface area, may underestimate
metabolism in a fat person
d. To convert cc’s of oxygen per minute to calories per day, a conversion
factor of 10 kcal of energy per liter of oxygen should be used
Answer: b, c

145 of 167 11-Aug-15 10:57 PM


MedCosmos Surgery: MCQ : General Surgery http://medcosmossurgery.blogspot.com/search/label/MCQ : General Surgery

The actual metabolic rate of any patient can be estimated from the
predicted basal rate according to the clinical situation. The amount of
energy is most conveniently expressed in calories/day. The metabolic
rate is normalized to body surface area; however, the actively
metabolizing tissue is the lean body cell mass. Consequently, reporting
“per square meter” underestimates metabolism in a fat person and
overestimates in a very lean person. Although most of studies on
nutrition in critical illness have been based on estimated energy
expenditure, actual measurement is much more accurate and has
become an important aspect of critical care management. The most
commonly used method of measurement is indirect calorimetry. In this
method, the amount of oxygen absorbed across the lungs into the
pulmonary blood is measured over a given period of time. Assuming the
patient is at a metabolic steady state during this time, the amount of
oxygen absorbed across the lungs is equal to the amount of oxygen
consumed in the metabolic process. The metabolic rate, measured in
cubic centimeters of oxygen/minute, can be converted to calories/hour
or /day if the oxygenated substrates are known. For practical purposes,
a conversion factor of 5 kcal of energy/liter of oxygen consumed is a
reasonable approximation.

263 Which of the following statement(s) is/are true concerning the


response to a decrease in functional residual capacity percent (FRC)?

a. Supplying supplemental oxygen will always improve the situation


b. Respiratory alkalosis may occur
c. Decreasing compliance is a common occurrence
d. Respiratory rate and depth of breathing generally decrease
Answer: b, c

Pulmonary arterial spasm in response to local hypoxia autoregulates


pulmonary blood flow and maintains adequate gas exchange during
alveolar collapse—up to a point. However, when the loss in ventilation
exceeds the decrease in perfusion, a ventilation-perfusion mismatch
occurs, which results in incomplete oxygenation of blood perfusing that
area of the lung. The resultant hypoxemia stimulates an increased rate
and depth of breathing which may serve to reexpand the person’s
inflated area of lung. If it does not, hypoxemia will continue but
increased ventilation in other areas of the lung will result in excess CO2
excretion, hypocapnea and respiratory alkalosis. The blood gas picture,
hypoxemia with respiratory alklalosis, is the most common abnormality
of gas exchange in surgical patients and it is a hallmark of ventilation-
perfusion imbalance. Oxygenation of blood in the poorly ventilated area
of lung can be improved by increasing concentration of oxygen in the
inspired gas. The use of supplemental oxygen, however, treats the
symptom rather than the basic cause and may actually make the
problem worse by adding to absorption atelectasis, depriving the poorly
ventilated area of nitrogen to hold alveoli open. This may result in total
alveolar collapse. In this circumstance, blood perfusing the
nonventilated area will mix with blood from other areas of the lung,

146 of 167 11-Aug-15 10:57 PM


MedCosmos Surgery: MCQ : General Surgery http://medcosmossurgery.blogspot.com/search/label/MCQ : General Surgery

resulting in hypoxemia that does not improve significantly in response of


administration of oxygen. Aside from the effects on gas exchange, loss
of alveolar space results in changes in the volume-pressure relationships
in the lung. A decrease in functional residual capacity always results in
a shift in the volume-pressure relationship toward a condition of
decreasing compliance.

264 Which of the following statement(s) is/are correct concerning the


body fluid compartments?

a. Both the extracellular and intracellular components of total body


water can be directly measured
b. The intravascular space accounts for the majority of extracellular
fluid
c. All water in the interstitial space is freely exchangeable
d. Transcellular fluid, separated from other compartments by both
endothelial and epithelial barriers, constitute about 4% of total body
water
Answer: d

Total body water (TBW) is distributed within the intracellular and


extracellular compartments. Intracellular fluid cannot be measured
directly but is calculated as the difference between TBW and the
measured extracellular water. Extracellular fluid can be measured
directly. The extracellular fluid compartment can be further simplified
into the intravascular and interstitial spaces. Intravascular space, which
accounts for 20% of the extracellular fluid, contains the plasma volume
which is approximately 8% of TBW or 5% of body weight. The interstitial
space extends from the blood vessels to the cells themselves and
includes the complex ground substance making up the acellular matrix
of tissue. Although the water within the space is thought to be freely
exchangeable, this water exists in two phases. The free phase contains
water that is generally freely exchangeable and in a constant state of
flux. The bound or gel phase is composed of water that is closely
associated with glycosaminoglycans, mucopolysaccharides, and other
matrix components. This water is much less freely exchangeable. An
additional extracellular fluid compartment, the transcellular
compartment, consists of water that is poorly exchangeable under
normal circumstances. This fluid is separated from other compartments
by both endothelial and epithelial barriers and includes cerebrospinal
fluid, synovial fluid, water within cartilage and bone, fluids of the eye,
and the lubricating fluids of the serous membranes. Together, these
fluids constitute about 4% of TBW.

265 Which of the following statements(s) is/are true concerning


metabolic alkalosis?

a. Either increased extracellular bicarbonate concentration or inhibited


renal excretion of bicarbonate can cause metabolic alkalosis
b. In metabolic alkalosis secondary to prolonged gastric outlet
obstruction, the urine pH is usually acidic

147 of 167 11-Aug-15 10:57 PM


MedCosmos Surgery: MCQ : General Surgery http://medcosmossurgery.blogspot.com/search/label/MCQ : General Surgery

c. Hypokalemia can lead to metabolic alkalosis


d. The respiration compensatory mechanisms for metabolic alkalosis are
quite ineffective
Answer: b, c, d

Sustained metabolic alkalosis occurs only if extracellular bicarbonate


concentration is increased and renal excretion of excess bicarbonate is
inhibited. Alone, neither is sufficient to result in metabolic alkalosis.
Extracellular bicarbonate concentration is increased by numerous
mechanisms. Loss of HCl is the leading cause of metabolic alkalosis in
surgical patients. External loss of gastric acid results in net gain in
bicarbonate, which causes metabolic alkalosis. Although the kidney can
excrete excess bicarbonate, this must be accompanied by excretion of
sodium. Renal excretion of sodium is limited in the face of volume
depletion, which also occurs with external losses of gastric secretion. As
volume depletion progresses, sodium is conserved in exchange for
hydrogen. Thus, in metabolic alkalosis secondary to prolonged gastric
outlet obstruction, the urine, although initially alkalotic, becomes
paradoxically acidotic in prolonged or uncorrected cases. Hypokalemia
and cellular exchange of potassium for hydrogen can also lead to
metabolic alkalosis. Hypokalemia results in enhanced proximal tubular
bicarbonate reabsorption and distal tubular acid secretion. The major
compensatory mechanism in metabolic alkalosis is respiratory, since the
presence of metabolic alkalosis implies renal dysfunction in either
generating or failing to excrete increased amounts of bicarbonate.
Hypoventilation is limited by the development of hypoxemia, which
stimulates ventilation. Among the four major types of acid-base
disorders, this compensatory mechanism is the least effective.

266 Which of the following statement(s) is/are true concerning


respiratory acidosis?

a. Respiratory acidosis is associated with chronic pulmonary disease far


more commonly than is hypoxemia
b. The initial buffering effect occurs at the cellular level
c. Renal compensation occurs within 24 hours
d. Correction of hypoxemia in patients with chronic lung disease may
worsen respiratory acidosis
Answer: b, d

Respiratory acidosis, the decrease in extracellular pH from a primary


increase in PCO2, is due to inadequate ventilation. Although pulmonary
disease commonly causes hypoxemia, respiratory acidosis is far less
common, since defusion of O2 is more readily impaired than diffusion of
CO2. Increased PCO2 results in increased H2CO3 which disassociates into
H+ and HCO3–. Cellular exchange of Na+ and K+ for H+ allows the
reaction to continue in this direction with increased extracellular
bicarbonate. This tissue buffering is accomplished within minutes.
Persistently elevated PCO2 also stimulates increased renal acid
excretion. Full renal compensation occurs over 3 to 5 days. The
treatment of chronic compensated respiratory acidosis may be
complicated by accompanying hypoxemia. In chronic hypercapnia, the

148 of 167 11-Aug-15 10:57 PM


MedCosmos Surgery: MCQ : General Surgery http://medcosmossurgery.blogspot.com/search/label/MCQ : General Surgery

chemical chemoreceptors may be insensitive and the accompanying


hypoxemia may supply the main respiratory drive through the
stimulation of peripheral chemoreceptors. In such patients, complete
correction of hypoxemia may further depress respiration and worsen the
respiratory acidosis.

267 Which of the following(s) is/are true concerning the control of the
volume of body water?

a. Osmoreceptors and baroreceptors work equally to control fluid


balance during normal conditions
b. The cardiac atrium regulates volume only by means of its
sympathetic and parasympathetic connections
c. The kidney is the primary effector organ in controlling water balance
d. The conversion of angiotensin I to angiotensin II is dependent on the
amount of the enzyme, renin, available
e. Nitric oxide plays a number of important roles in regulation of renal
hemodynamics
Answer: c, d, e

Changes in volume are detected both by osmoreceptors, which detect


changes in plasma osmolality and baroreceptors, which are sensitive to
changes in pressure. The osmoreceptors are responsible for day-to-day
fine-tuning of volume whereas the baroreceptors contribute relatively
little to the control of fluid balance under normal conditions. Changes in
effective circulating volume are sensed by the volume receptors of the
intrathoracic capacitance vessels and atria, the pressure receptors of
the aortic arch and carotid arteries, the intrarenal baroreceptors, and,
to a lesser extent, by the hepatic and cerebrospinal volume receptors.
These baroreceptors control volume by means of sympathetic and
parasympathetic connections. The atria also appear to serve as
endocrine organs capable of directing responses to volume changes with
the elaboration of the hormone, atrial natriuretic peptide. The major
hormonal mediator of baroreceptor modulation of volume control is the
renin-angiotensin system. The end result of this complex system of
receptors or messengers is a change in sodium and water balance
mediated by the kidneys. It is through changes in sodium and water
reabsorption that volume and pressure ultimately normalize. Renin is a
proteolytic enzyme that is released in response to changes in arterial
pressure, changes in delivery of sodium to the macula densa of the
distal convoluted renal tubule, increases in beta adrenergic activity and
increases in cellular cAMP. Renin cleaves angiotensin I from circulating
angiotensinogen. Angiotensinogen is abundant, so this reaction is
enzyme dependent rather than substrate dependent. Angiotensin I is
further cleaved to angiotensin II which acts with locally and systemically
to increase vascular tone. Angiotensin II affects sodium reabsorption by
decreasing renal plasma flow and the glomerular filtration coefficient.
Finally, angiotensin II increases sodium reabsorption by direct tubular
action as well as by stimulation of aldosterone release from the adrenal
cortex.
The importance of nitric oxide and its many biologic functions has

149 of 167 11-Aug-15 10:57 PM


MedCosmos Surgery: MCQ : General Surgery http://medcosmossurgery.blogspot.com/search/label/MCQ : General Surgery

recently been recognized. Nitric oxide participates in the regulation of


renal hemodynamics and renal handling of water and electrolytes.

268 Which of the following statement(s) is/are true concerning


maintenance intravenous fluid therapy?

a. The total daily water requirement for a 70-kg man is about 2500
mL/day
b. Normal maintenance IV therapy requires administration of sodium,
potassium, calcium, phosphate, and magnesium
c. Fluid volume calculations for elderly patients generally are decreased
compared to their younger counterparts
d. A child requires a lesser amount of maintenance fluid per kilogram
than a larger individual
Answer: a, c

Maintenance fluid replacement is aimed at replacing fluids normally lost


during the course of a day. Calculation of maintenance fluid
replacement does not include replacement of preexisting deficits or
ongoing additional losses. Formulas exist for calculating maintenance
fluid requirements which adjust for differences in body weight and for
changes in TBW content. A smaller (or younger) individual who has a
high percentage of TBW in relation to body weight requires a greater
amount of maintenance fluid per kilogram than a larger individual. The
total daily water requirement for a 70-kg man is about 2500 mL/day.
Because hypervolemia is poorly tolerated in older individuals and in
patients with cardiac disease, the volume calculated is generally
diminished in this age group. Normal maintenance therapy requires
administration of sodium and potassium. Replacement of calcium,
phosphate or magnesium are generally not necessary in patients
requiring short-term therapy. In critically ill patients, however, critical
deficits in these electrolytes may occur and must be replaced.

269 Which of the following statement(s) is/are true concerning


abnormalities in calcium concentration?

a. Parathyroid hormone affects calcium homeostasis only at the


exchange of calcium between bone and extracellular fluid
b. About 45% of total plasma calcium is in the ionized state and is
responsible for most physiologic actions
c. Changes in plasma protein levels or pH can alter the proportion of
calcium in the ionized state
d. Intravenous normal saline administration is the first step in treatment
of hypercalcemia
e. Classic signs of hypocalcemia include hyperactive deep tendon
refluxes, Cvostek’s sign and Trousseau’s sign
Answer: b, c, d, e

Calcium is a divalent cation found in abundance in the human body.


About 99% of total body calcium is located in bone in the form of

150 of 167 11-Aug-15 10:57 PM


MedCosmos Surgery: MCQ : General Surgery http://medcosmossurgery.blogspot.com/search/label/MCQ : General Surgery

hydroxyapatite crystals. Calcium homeostasis depends on the exchange


of calcium between bone and extracellular fluid, renal excretion, and
intestinal absorption. These three processes are controlled to a great
extent by parathyroid hormone. In extracellular fluid, calcium exists in
three forms: ionized calcium, non-ionized calcium, and protein-bound
calcium. Ionized calcium, which comprises about 45% of total calcium is
responsible for most physiologic actions of calcium in the body, and its
level is tightly controlled by a regulatory mechanisms. Some nonionized
calcium is complexed with non-protein anions, including phosphate and
citrate, and does not easily disassociate. These molecular forms make
up only about 15% of total calcium present in plasma. About 40% of
extracellular nonionized calcium is bound to proteins, with most being
bound to albumin. Changes in either plasma protein levels or pH can
alter the proportion of calcium in the ionized state. The most common
cause of hypercalcemia is primary hyperparathyroidism. Hypercalcemia
can also occur secondary to malignant disease, caused either by a
metastasis to bone or by autonomous tumor secretion of hormone-like
substances that alter calcium homeostasis. Neuromuscular effects may
be the earliest manifestations and include muscle fatigue, weakness,
personality disorders, psychosis, confusion, and coma. Elevation of total
serum calcium concentrations to greater than 14mg/dL requires prompt
treatment to prevent any serious and potentially lethal complications.
Immediate measures are directed toward maximizing renal excretion of
calcium. Vigorous hydration with 0.9% saline solution to prompt diuresis
should be the initial step in treatment. The addition of potassium to the
resuscitation fluid as well as the use of furosemide can also be used for
treatment.
Serum calcium levels below 8 mg/dL may be associated with symptoms
and signs that are primary manifestations of neuromuscular
abnormalities. These include muscle cramps, perioral tingling,
parastesias, laryngeal stridor, tetany, seizures and psychotic behavior.
Classic signs of hypocalcemia include hyperactive, deep tendon
reflexes, Cvostek’s sign, and Trousseau’s sign. Symptomatic
hypocalcemia is best treated with intravenous infusion of calcium in the
form of calcium gluconate or calcium chloride.

270 Which of the following statement(s) is/are true concerning total


body water?

a. Total body water in men represents a higher percent body weight


than in women
b. In infants, water comprises up to 80% of body weight
c. Total body water content decreases with increasing age
d. Total body water is equally distributed within the intra-and
extracellular compartments
Answer: a, b, c

The total volume of water within the body is termed total body water.
The relationship between total body water (TBW) and body weight is
relatively consistent for any given individual and depends on the amount
of fat within the body. Because fat contains little water, TBW as a

151 of 167 11-Aug-15 10:57 PM


MedCosmos Surgery: MCQ : General Surgery http://medcosmossurgery.blogspot.com/search/label/MCQ : General Surgery

percentage of body weight decreases with increasing body fat. The


estimated TBW in men is 60% of body weight, whereas in women, who
typically have more adipose tissue, the average TBW is 50% of body
weight. The percentage of body weight accounted for by water also
varies with age. In infants, water comprises about 80% of body weight.
Throughout adult life, a gradual decrease occurs in TBW content
because of the amount of fat within the body usually increases with
age. In obese patients, estimates of TBW should be decreased by 10% to
20% whereas in lean patients, estimates should be increased by about
10%.
TBW is distributed within the intra and extracellular compartments.
Intracellular fluid makes up about 2/3 of the TBW, or 40% of body
weight.

271 Which of the following statement(s) is/are true concerning the


clinical presentation and treatment of severe metabolic alkalosis?

a. In most cases clinical signs are obvious


b. Correction of potassium and volume depletion corrects most cases of
metabolic alkalosis
c. Acetazolamide can enhance renal excretion of bicarbonate
d. Acid replacement should be provided at a molar equivalent basis for
excess serum bicarbonate
Answer: b, c

Clinical signs of metabolic alkalosis may not be prominent, since the


condition usually develops relatively slowly. Correction of the underlying
cause is the mainstay of treatment in this disorder. In general,
correction of potassium and volume depletion corrects the metabolic
alkalosis. In patients without intravascular volume deficits, renal
excretion of bicarbonate can be enhanced by administration of the
carbonic acid anhydrase inhibitor acetazolamide. If renal excretion of
bicarbonate cannot be increased because of underlying renal
insufficiency or if the metabolic alkalosis is severe, acid may be
administered to directly titrate the excess extracellular bicarbonate.
Acids that can be used include ammonium chloride, arginine
hydrochloride, or dilute hydrochloric acid. Partial correction of alkalosis
is the initial goal. A general guide is that 2.2 mEq/kg decreases serum
bicarbonate by about 5 mEq/L.

272 Which of the following statement(s) is/are true concerning


abnormalities in serum sodium?

a. The most common cause of hyponatremia is a deficit in total body


sodium
b. Hyponatremia can occur in situations of excessive solute
c. Most surgical patients with hyponatremia are best treated by free
water restriction
d. Central nervous system effects are the predominant symptom of
hypernatremia

152 of 167 11-Aug-15 10:57 PM


MedCosmos Surgery: MCQ : General Surgery http://medcosmossurgery.blogspot.com/search/label/MCQ : General Surgery

e. Hypernatremia should be rapidly corrected with free water


administration
Answer: b, c, d

The most common cause of hyponatremia is an excess of free water


rather than a deficit of total body sodium. Hyponatremia is frequently
seen in the postoperative or post-trauma patients because increased
ADH secretion acts on the collecting tubules of the kidney to increase
free water reabsorption. Although hyponatremia most often results from
excess free water, it can occur in the presence of excess solute. In this
situation, TBW content is either normal or diminished but the plasma
osmolality is increased. An example of this hyperosmolar-hyponatremic
state is untreated hyperglycemia. Excess solute may also be due to
exogenous administration or ingestion of mannitol, ethanol, methanol,
or ethylene glycol. Most surgical patients with hyponatremia are
euvolemic or hypervolemic. Such patients, if asymptomatic, are best
treated by free water restriction, since free water overload is the cause
of the condition. Hypernatremia is a less common problem in surgical
patients than hyponatremia and is usually the result of excess free
water loss associated with hypovolemia. Hypernatremia may also be
secondary to increased total body content of sodium, which is usually
related to exogenous administration of sodium. The symptoms of
hypernatremia are related to the hyperosmolar state. CNS effects
predominate because of cellular dehydration as water passes into the
extracellular space. Once hypernatremia becomes symptomatic, it is
associated with significant morbidity and mortality. Prompt treatment of
hypernatremia is essential. Rapid correction, however, of hypernatremia
is associated with significant risk of cerebral edema and herniation.
Because chronic hypernatremia is relatively well tolerated, there are
few advantages to rapidly correcting the free water deficit. Moderate
degrees of hypernatremia are tolerated well, and symptoms rarely
develop unless serum sodium levels exceed 160 mEq/liter. The
development of symptoms also depends on the rapidity at which
hypernatremia develops.

273 Which of the following statement(s) is/are true concerning


abnormalities in serum potassium?

a. Hyperkalemia can occur in the otherwise normal surgical patient due


to excessive intravenous potassium administration
b. The primary EKG change associated with severe hyperkalemia is
peaked T-waves
c. Temporary treatment of hyperkalemia includes administration of
calcium, sodium bicarbonate, or glucose and insulin
d. Alterations in membrane potentials reflected in cardiac and skeletal
muscle are common results of both hypo-and hyperkalemia
e. A reduction in serum potassium of 1mEq/liter requires replacement
of 40mEq of potassium
Answer: c, d

Potassium is the major intracellular cation and is a major determinant

153 of 167 11-Aug-15 10:57 PM


MedCosmos Surgery: MCQ : General Surgery http://medcosmossurgery.blogspot.com/search/label/MCQ : General Surgery

of intracellular osmolality. Because of the large differences between


intracellular and extracellular potassium concentrations, a
transmembrane potential is generated. Alterations in potassium
concentration gradient (both hyper- and hypokalemia) have profound
effects on transmembrane potential and consequently on cellular
function. This is especially true for cardiac, skeletal, and smooth
muscle. Extracellular potassium concentration is primarily determined
by renal excretion. About 90% of ingested potassium is secreted by the
urine. Hyperkalemia therefore rarely develops from excessive potassium
intake in the absence of renal insufficiency, since the capacity for renal
potassium excretion is large. In the surgical patient, diminished renal
function is perhaps the most common problem leading to hyperkalemia.
Both chronic and acute renal failure result in the deficit in potassium
excretion. Hyperkalemia can also be associated with cellular disruption,
such as with crush injuries or lysed erythrocytes in large hematomas or
after massive blood transfusion. The clinical manifestations of
hyperkalemia are primarily related to membrane depolarization. The
most life-threatening manifestations are related to the cardiac effects
of membrane depolarization. Mild hyperkalemia results in peaked
T-waves on the EKG and may cause parethesia and weakness. More
severe forms of hyperkalemia cause flattened P-waves, prolongation of
the QRS complex, and deep S-waves on EKG. Ventricular fibrillation and
cardiac arrest may follow. Severe hyperkalemia with EKG abnormalities
requires urgent treatment. Rapid infusion of 10% to 20% calcium
gluconate may reduce the effects of hyperkalemia on membrane
potentials. Administration of sodium bicarbonate is another temporary
measure. The increase in serum sodium antagonizes the effects of
hyperkalemia on the membrane potential, whereas the increase in
extracellular pH shifts potassium into the cells. Movement of potassium
into the intracellular compartment can also be achieved by giving
insulin and glucose.
Hypokalemia is usually caused by total body potassium depletion
secondary to the decreased potassium intake, increased extra-renal
potassium losses, or increased renal potassium losses. Decreased serum
potassium levels may also be secondary to redistribution of potassium
into the intracellular space. Symptoms of hypokalemia, like those of
hyperkalemia, are manifested by disturbances in membrane potentials.
As potassium levels fall below 2.5mEq/L, muscle weakness is common.
The primary treatment of hypokalemia is potassium replacement. The
route and rate of potassium replacement depends on the presence and
severity of symptoms. A reduction in serum potassium of 1mEq/L
represents a total body potassium deficiency of 100 to 200 mEq.

274 Which of the following statement(s) is/are true concerning the


derangement of metabolic acidosis?

a. A major source of acid production of the body is sulfuric acid


b. Excessive loss of bicarbonate can occur with intestinal or pancreatic
fistulas
c. Ketoacidosis can occur in conditions of either hyper-or hypoglycemia
d. Lactic acidosis is present when serum lactate concentration is > 2

154 of 167 11-Aug-15 10:57 PM


MedCosmos Surgery: MCQ : General Surgery http://medcosmossurgery.blogspot.com/search/label/MCQ : General Surgery

mEq/L
e. Lactic acidosis can be associated with ethanol toxicity
Answer: a, b, c, e

Most clinically significant metabolic acidosis is related to the net loss of


bicarbonate, which occurs when consumption due to either loss or
titration is greater than bicarbonate generation. Under normal
circumstances of ingestion of the average amount of protein in an
American diet, about 70 mEq acid is generated daily. The major source
of acid production is sulfuric acid from the metabolism of sulphur-
containing amino acids. Increased protein intake and tissue catabolism
resulting in greater metabolism of sulphur containing amino acids may
lead to a generation of increased amounts of sulfuric acid. This excess
acid utilizes excess bicarbonate for neutralization. Diarrhea, intestinal
or pancreatic fistulas, and burns can cause metabolic acidosis secondary
to loss of bicarbonate. The two most common types of organic acidosis
are ketoacidosis and lactic acidosis. The abnormality primarily
responsible for ketoacidosis is deficiency of insulin whether primary, as
in diabetic ketoacidosis, or secondary to hypoglycemia. Under normal
conditions a small amount of ketoacids is produced. During prolonged
starvation, production of ketoacids increase to modest levels, providing
an important source of energy to nonhepatic tissues, particularly the
brain. In ketoacidosis, the ketoacid production is excessive because of
insulin deficiency. In diabetic acidosis, insulin deficiency also
contributes to hyperglycemia by decreasing the metabolism of glucose
by extrahepatic tissue and increasing hepatic production of glucose.
Lactic acidosis can be divided into type A, caused by tissue hypoxia, and
type B, caused by other mechanisms. Hypoxia, the most common cause
of lactic acidosis, impairs the mitochondrial oxidation of NADH to NAD
that is necessary for glycolysis. Normal serum lactate concentration is
below 2 mEq/L. Lactate acidosis is secondary to hypoxemia, usually due
to an increased production of lactate as well as decreased use, and
serum lactate concentrations greater than 6 mEq/L. The most common
cause of type B lactate acidosis is ethanol intoxication.

275 Which of the following statement(s) is/are true concerning the


osmotic activity of body fluids?

a. Urea contributes to the osmolality of a solution but not its tonicity


b. The osmolality of the body remains fairly constant at approximately
289 mOsm/kg H2O
c. The two primary regulators of water balance are antidiuretic
hormone and aldosterone
d. Serum sodium is the most valuable laboratory indicator of abnormal
total body water content
Answer: a, b, d

Body fluids are aqueous solutions composed primarily of water and


contained in different compartments of the body. The movement of
water from these compartments depends on a number of physical
properties, the most important of which is osmosis. According to the

155 of 167 11-Aug-15 10:57 PM


MedCosmos Surgery: MCQ : General Surgery http://medcosmossurgery.blogspot.com/search/label/MCQ : General Surgery

principles of osmosis, if two solutions are separated by semipermeable


membrane, water moves across the membrane to equalize the
concentration of the osmotically active particles. The osmotic activity
across a semipermeable membrane is determined by the concentration
of solutes on each side of the membrane. The body is capable of fine
regulation of solute and water concentrations, so that osmolality
remains fairly constant at an average of 289 mOsm/kg H2O. In response
to small changes in cell volume, osmoreceptors in the paraventricular
and supraoptic nuclei of the hypothalamus send signals to the neuronal
centers that control the two primary regulators of water balance, thirst
and antidiuretic hormone secretion. Changes in TBW are reflected by
changes in extracellular solute concentration. Because sodium is the
primary extracellular cation and potassium is the predominant
intracellular cation, the serum sodium approximates the sum of the
exchangeable total body sodium and exchangeable total body potassium
divided by the TBW. Because total body solute content remains
relatively stable over time, changes in TBW content result in inversely
proportional changes in serum sodium. Thus, abnormalities in serum
sodium are the indication of abnormal TBW content. In contrast to
impermeable solutes that are excluded from the intracellular space,
such as sodium, permeable solutes such as urea can freely cross the cell
membranes. Although urea contributes to the osmolality of a solution, it
has no effect on tonicity because it distributes equally across
membranes, and as such does not contribute to the osmols that affects
cell volume.

276 Which of the following statement(s) is/are true concerning the


compensatory mechanisms and treatment of metabolic acidosis?

a. Maximal renal compensation for metabolic acidosis occurs before full


respiratory compensation can occur
b. All patients with lactic acidosis should receive prompt treatment with
bicarbonate
c. Potassium replacement is essential even in the face of normal or high
serum potassium when treating diabetic ketoacidosis
d. Sodium bicarbonate administration should begin simultaneous with
volume resuscitation in patients with hypoxia secondary to shock
Answer: c

The kidney is extremely sensitive to changes in serum bicarbonate


concentration and responds by increasing net acid excretion primarily
by increasing ammonia excretion. Maximal renal compensation requires
2 to 4 days. Delay in achieving maximal renal response to an increased
acid load causes blood pH to decline, which stimulates hyperventilation.
Although effective in promptly raising blood pH, ventilatory
compensation is only partial, and full respiration compensation requires
12 to 24 hours. The major principal of treatment for mild to moderate
acute metabolic acidosis is correction of the underlying cause. In
surgical and trauma patients, metabolic acidosis is often the result of
hypoxia secondary to inadequate tissue perfusion and subsequent lactic
acidosis. Volume and/or blood resuscitation alone may be enough to

156 of 167 11-Aug-15 10:57 PM


MedCosmos Surgery: MCQ : General Surgery http://medcosmossurgery.blogspot.com/search/label/MCQ : General Surgery

correct the acidosis. Attempts to correct acidosis with exogenous


bicarbonate before correction of inadequate tissue perfusion are usually
unsuccessful. The use of bicarbonate for the treatment of lactic acidosis
is controversial at best. In several studies the use of bicarbonate in
patients with lactic acidosis does not improve clinical parameters or
outcome. The correction of both acidosis and hypoglycemia of diabetic
ketoacidosis is best achieved by the administration of insulin. Volume
resuscitation is also required. Potassium replacement is essential, even
in the face of normal or high serum potassium, and as hypokalemia
develops as acidosis in hyperglycemia are corrected.

277 Which of the following statement(s) is/are true concern renal


tubular acidosis?

a. Renal tubular acidosis is primarily caused by reduction in ammonia


excretion
b. The renal tubular defect in renal tubular acidosis can either be at the
distal or proximal renal tubule
c. In distal renal tubular acidosis associated with hyperkalemia, the
defect involves increased tubular permeability with backleak of
secreted sodium and potassium into the tubular cell
d. Uremic acidosis occurs independently of protein intake
Answer: a, b

The impaired ability of the kidney to excrete acid and hence generate
bicarbonate may be secondary to a decrease in the number of
functioning nephrons and is termed uremic acidosis or renal tubular
acidosis. Renal tubular acidosis, which can occur both in acute and
chronic renal failure, is primarily caused by reduction in ammonia
excretion secondary to a reduction in the number of functioning
proximal tubular cells. In addition, decreased proximal tubular
bicarbonate reabsorption contributes to the development of acidosis.
Although the onset of uremic acidosis is related to declining renal
function, its appearance may be influenced by diet-dependent protein
and organic anion ingestion. Renal tubular acidosis may be classified as
distal or proximal, depending on the primary site of the renal tubular
defect leading to acidosis. In renal tubular acidosis with hyperkalemia,
the mechanism is decreased luminal negativity secondary to impaired
sodium reabsorption. In distal renal tubular acidosis with hypokalemia,
mechanisms including increased tubular permeability with backleak of
secreted H+ into the tubular cell and reduced H+ pump activity are
proposed mechanisms.

278 Which of the following statement(s) is/are true concerning the


postoperative fluid management in a surgical patient?

a. Standard formulas are available that essentially can direct the


therapy for all patients
b. Isotonic solutions containing potassium should be used throughout the
entire postoperative period

157 of 167 11-Aug-15 10:57 PM


MedCosmos Surgery: MCQ : General Surgery http://medcosmossurgery.blogspot.com/search/label/MCQ : General Surgery

c. Urine output should be maintained at a level greater than 0.5


ml/kg/h
d. A urine specific gravity of greater than 1.012 may indicate that the
patient is dehydrated
Answer: c, d

Fluid therapy during the postoperative period should be tailored to each


patient and depends on the adequacy of patient’s volume status at the
completion of the operative procedure, as well as ongoing fluid losses.
Maintenance fluid should be supplemented by replacement of the
additional fluids needed to replace the ongoing third space loss as well
as losses from various tubes and drains. In general, isotonic solution
should be used for volume resuscitation during the early postoperative
period. It is best not to give potassium supplements during this period
unless they are specifically required as indicated by serum electrolyte
measurements. Monitoring fluid status during the postoperative period is
best accomplished by careful monitoring of vital signs, urine output, and
central venous pressure, if necessary. Urine output is maintained at a
level greater than 0.5 mL/kg/h. A urine specific gravity of greater than
1.010 to 1.012 indicates that urine is being concentrated and the
patient may not be receiving adequate hydration.

279 Which of the following statement(s) is/are true concerning


parenternal electrolyte solutions?

a. Lactated Ringer’s solution contains physiologic concentrations of all


important electrolytes
b. Glucose is added to hypotonic saline solutions to increase their
tonicity
c. About 1/2 of all exogenously administered albumin ends up in the
extravascular space
d. Normal saline solution provides excessive sodium and chloride which
may lead to body sodium overload
Answer: b, c, d

A number of electrolytes solutions are available for parenteral


administration. Lactated Ringer’s solution is a physiologic solution
containing many of the electrolytes found in plasma. The disadvantage
of this solution is the relatively low sodium content (130 mEq/L) as
compared to plasma. Hyponatremia can occur with extended use of
lactated Ringer’s solution. Isotonic saline (0.9% or normal saline)
contains 154 mEq of both sodium and chloride. The excess of both
sodium and chloride can lead to electrolyte and acid-base disturbances.
Infusion of large volumes of 0.9% saline can lead to total body sodium
overload and hyperchloremia. The less-concentrated saline solutions are
hypo-osmotic and have excess free water. In addition, 0.2% saline
solution is hypotonic with respect to plasma and can result in red blood
cell lysis if rapidly infused. For this reason, 5% dextrose is added to
these solutions to increase the tonicity. Plasma expanders are commonly
used in surgical patients. Plasma protein solutions such as 5% and 25%
albumin act initially by increasing plasma oncotic pressures.

158 of 167 11-Aug-15 10:57 PM


MedCosmos Surgery: MCQ : General Surgery http://medcosmossurgery.blogspot.com/search/label/MCQ : General Surgery

Abnormalities in microvascular permeability such as those found in the


pulmonary circulation in adult respiratory distress syndrome, in regional
circulatory bed burns or infections, and in the systemic circulation in
sepsis, may result in extravasation of these proteins into the interstitial
space. About half of all exogenously administered albumin eventually
ends up in the extravascular space. The half life of exogenously
administered albumin is about 11 days.

280 An 11-year-old boy has experienced severe diarrhea for 10 days. He


presents with decreased skin tungor, sunken eyes, orthostatic
hypotension, and tachycardia. Which of the following statement(s) may
be true concerning his diagnosis and treatment?

a. His hematocrit will likely be elevated


b. His BUN may be elevated out of proportion to serum creatinine
c. His serum sodium will be elevated
d. Fluid resuscitation should begin with D5/.2 normal saline because of
the expected high serum sodium associated with excessive fluid loss
Answer: a, b

Chronic volume deficits may be manifested by decreased skin turgor,


weight loss, sunken eyes, hypothermia, oliguria, orthostatic hypotension
and tachycardia. Serum BUN and creatinine may be elevated, with a
high BUN/creatinine ratio. The hematocrit may be elevated as well.
Plasma sodium is not an indicator of intravascular volume, and if the
loses have been isotonic, plasma sodium concentration remains normal.
Fluid resuscitation for hypovolemia is initiated with an isotonic solution
such as lactated Ringer’s solution. Urine flow in critically ill patients is
monitored with an indwelling Foley catheter, with the goal of a urine
output 0.5mL/kg/h desirable.

281 Clinical manifestations of acute metabolic acidosis include:

a. Decreased cardiac contractility


b. Decreased catecholamine secretion
c. Peripheral arteriolar dilitation
d. Shift of the oxygen-hemoglobin disassociation curve to the left
Answer: a, b, c

The major cardiovascular effects of acute metabolic acidosis are


peripheral arteriolar dilitation, decreased cardiac contractility, and
central venous constriction. These may lead to cardiovascular collapse
and pulmonary edema. Catecholamine secretion is stimulated by
metabolic acidosis and in mild cases, heart rate may be increased. In
addition to these cardiovascular effects, metabolic acidosis may also
affect oxygen delivery by shifting the oxygen-hemoglobin disassociation
curve to the right.

282 Which of the following statement(s) is/are true concerning

159 of 167 11-Aug-15 10:57 PM


MedCosmos Surgery: MCQ : General Surgery http://medcosmossurgery.blogspot.com/search/label/MCQ : General Surgery

respiratory alkalosis?

a. Exposure to high altitudes can result in respiratory alkalosis


b. Renal compensation for respiratory alkalosis is obtained by increasing
excretion of bicarbonate
c. Symptoms of respiratory alkalosis may mimic those of hypocalcemia
d. The treatment of acute respiratory alkalosis may involve a brown
paper bag
Answer: a, c, d

A primary decrease in PCO2 resulting in an increase extracellular pH is


referred to as respiratory alkalosis. Hyperventilation and the ensuing
fall in PCO2 may be secondary to hypoxia, reflux simulation from
decreased pulmonary compliance, drugs, mechanical ventilation, and
other causes. The two most common causes of hypoxia resulting in
respiratory alkalosis are pulmonary disease and exposure to high
altitudes. Renal compensation for respiratory alkalosis is not achieved
by increasing excretion of bicarbonate but by decreasing net acid
excretion, primarily through the reduction in ammonia excretion and
increases in organic anion excretion. Chronic respiratory alkalosis is
generally asymptomatic. Acute respiratory alkalosis may cause
sensations of breathlessness, dizziness, and nervousness and can result
in circumoral and extremity parathesias, altered levels of consciousness,
and tetany. These signs are related to decreased cerebral blood flow
secondary to decreased PCO2 and decreased ionized calcium
concentration secondary to increased blood pH. In acute symptomatic
respiratory alkalosis rebreathing, by breathing in and out of a paper
bag, can temporarily relieve the symptoms.

283 Which of the following statement(s) is/are true concerning


alterations in serum magnesium?

a. Renal failure is the primary cause of hypermagnesemia


b. Hypomagnesemia may occur during prolonged periods of intravenous
fluid replacement
c. Symptoms of hypomagnesemia may mimic symptoms of hypocalcemia
d. Intravenous administration of magnesium sulfate is usually the most
efficient method of correction of magnesium deficiency
Answer: a, b, c, d

Renal failure is the primary cause of hypermagnesemia. Because of the


kidneys ability to excrete large magnesium loads, hypermagnesemia
rarely occurs if renal function remains normal. Because the kidneys are
able to conserve magnesium well in states of magnesium depletion,
hypomagnesemia rarely occurs from poor intake alone. The combination
of low intake and increased gastrointestinal loss may lead to
hypomagnesemia. Prolonged periods of intravenous fluid replacement
without magnesium replacement and the chronic use of loop diuretics or
other medications such as cyclosporine or aminoglycosides can also
result in hypomagnesemia. Deficiencies of magnesium may present signs
and symptoms similar to hypocalcemia. Hypomagnesemia may be

160 of 167 11-Aug-15 10:57 PM


MedCosmos Surgery: MCQ : General Surgery http://medcosmossurgery.blogspot.com/search/label/MCQ : General Surgery

treated by the oral administration of magnesium however large doses


frequently leads to diarrhea. Correction of major deficits is therefore
best managed by intravenous administration of magnesium sulfate at a
dose of 50 to 100 mEq/d.

284 Muscle relaxants are a class of anesthetic agents used to prevent


movement and facilitate surgical exposure. Which of the following
statement(s) is/are true concerning the use of muscle relaxants in
surgical procedures.

a. Succinylcholine produces rapid obvious muscle fasciculations


b. Pancuronium can be reversed by increasing the acetylcholine
concentration using an anticholinesterase inhibitor (neostigmine)
c. Prolonged periods of muscle relaxation in patients requiring
prolonged ventilation should be used in conjunction with analgesics and
amnesic agents
d. The best clinical test for complete reversal of neuromuscular
blockade is the ability of the patient to produce a large negative
inspiratory force
Answer: a, b, c

Neuromuscular blocking agents can be classified as depolarizing or


nondepolarizing inhibitors of the neurotransmitter, acetylcholine at the
neuromuscular junction. The only noncompetitive inhibitor employed
clinically is succinylcholine. This drug rapidly binds to the
neuromuscular junction and produces depolarization, clinically obvious
as fine muscle fasciculations occurring approximately 60 seconds after
injection. All other clinically useful muscle relaxants are termed
competitive inhibitors and do cause depolarization when they attach to
the neuromuscular junction. Since these agents compete with
acetylcholine, the block produced is in direct proportion to the
concentration of the agent relative to the concentration of
acetylcholine. If the concentration ratio is low enough, competitive
relaxants can be “reversed” if the concentration of acetylcholine is
artificially elevated. Increase of acetylcholine concentration can be
achieved by giving a drug which blocks metabolism of anticholinesterase
(neostigmine).
Nondepolarizing relaxants are frequently used in critically ill patients
who are difficult to manage otherwise during prolonged periods of
mechanical ventilation. It is imperative that these drugs be given in
conjunction with analgesics and amnesic agents, since neuromuscular
blocking agents have no analgesic or amnestic properties and only
prevent motion of voluntary muscles. Patients may therefore be totally
aware and in pain and unable to communicate. All muscles of the body
do not have equal sensitivity in muscle relaxants. The diaphragm is both
resistant to neuromuscular blockade while the neck and pharyngeal
muscles that support the airway are most sensitive. It is possible for an
intubated patient to spontaneously ventilate and even to produce a
large negative inspiratory force and yet have complete airway
obstruction when extubated due to effects of residual muscle relaxants
on upper airway muscles. The definitive clinical test for complete
reversal of neuromuscular blockade is the ability of the patient to

161 of 167 11-Aug-15 10:57 PM


MedCosmos Surgery: MCQ : General Surgery http://medcosmossurgery.blogspot.com/search/label/MCQ : General Surgery

sustain a head lift from the bed for five seconds.

285 Local anesthetics are essential agents used in current surgical


practice. Which of the following statement(s) is/are true concerning the
use of local anesthetic agents.

a. Complications due to excessive plasma concentration can result only


from inadvertent intravascular injection of the agent
b. Bupivacaine is noted for a slow onset but long duration
c. The addition of epinephrine to a local anesthetic agent will both
lower the toxicity and increase the duration of local anesthesia
d. Hypotension observed when a local anesthetic is administered in the
form of a spinal epidural block, is the result of myocardial depression
Answer: b, c

Local anesthetics constitute a class of drugs which produce temporary


blockage of nerve conduction by binding to neuronal sodium channels.
Adverse consequences associated with the use of local anesthetics fall
into three categories: acute central nervous system toxicity due to
excessive plasma concentration, hemodynamic and respiratory
consequences due to excessive conduction block of the sympathetic or
motor nerves, and allergic reactions. Whenever a local anesthetic has
been injected, there may be inadvertent intravascular injection or an
overdose of the drug due to rapid uptake from the tissues. All may
produce seizures. Complications can be minimized by aspirating prior to
injection to avoid intravascular injection and limiting the doses to the
safe range. When local anesthetics are administered for a spinal or
epidural block, there will be a progressive blockade of the sympathetic
nervous system which will produce systemic vasodilatation. If the block
travels along the thoracolumbar region, a sympathetic blockade will
result in profound systemic vasodilatation and bradycardia with
resultant hypotension.
Local anesthetics are divided into two groups: esters and amides. Most
commonly used agents, the amides, include lidocaine and bupivacaine.
Lidocaine is noted for a fast onset of action but a short duration
whereas bupivacaine has a slower onset with the duration lasting for
four to 12 hours. The addition of epinephrine (100 µg) will lower the
toxicity and increase the duration of the local anesthetic.

286 A 65-year-old gentleman with a history of coronary artery disease


and a recent myocardial infarction requires an elective colon resection
for a nonobstructing neoplasm. Which of the following statement(s)
is/are true concerning the risks of general anesthetic in this patient?

a. The age of the previous infarct has no effect on the perioperative


reinfarction risk
b. The incidence of reinfarction appears to stabilize after six months
c. Invasive hemodynamic monitoring has no effect on perioperative
reinfarction rates
d. Reinfarction has minimal effect on mortality

162 of 167 11-Aug-15 10:57 PM


MedCosmos Surgery: MCQ : General Surgery http://medcosmossurgery.blogspot.com/search/label/MCQ : General Surgery

e. Perioperative infarction most frequently occurs after the first 72


hours from surgery
Answer: b, d

The history of myocardial infarction is an important risk factor for


general anesthesia. Large retrospective studies have found that the
incidence of reinfarction is related to the time elapsed since the
previous myocardial infarction. The incidence of reinfarction appears to
stabilize at approximately 1% after six months, with the highest rate of
reinfarction occurring in the first three months after the infarct.
Mortality from reinfarction, for patients undergoing non-cardiac surgery,
has been reported to be between 20–50% and usually occurs within the
first 48 hours after surgery. Invasive hemodynamic monitoring with
pulmonary artery catheters and aggressive pharmacologic intervention
has been demonstrated to reduce reinfarction rates.

287 General anesthesia is not without risks. Which of the following


statement(s) is/are true concerning the risk associated with general
anesthesia.

a. Current estimates for mortality due to anesthesia alone are 1:10,000


b. Human error accounts for between 50 and 75% of anesthetic-related
deaths
c. Most anesthetic-related deaths are associated with overdose of
analgesic agents
d. The most common problems associated with adverse anesthetic
outcomes are related to the airway
Answer: b, d

Anesthetic agents effectively obtund or completely block nearly all


physiologic protective mechanisms, therefore, there is an associated
risk even without a surgical procedure. Fortunately, with the advent of
newer agents and monitoring techniques, it is estimated the mortality
due to anesthesia alone has decreased from approximately 1:10,000 in
the 1950s to as low as 1:100,000 or less for healthy patients today. It has
been estimated that between 50–75% of anesthetic-related deaths are
due to human error and are preventable. The most common problems
associated with adverse outcomes are related to the airway: inadequate
ventilation, unrecognized esophageal intubation, unrecognized
extubation, and unrecognized disconnection from the ventilator.

288 Over the last decade, the routine use of both invasive and
noninvasive monitoring devices has been instituted for the
administration of most anesthetics. The following statement(s) is/are
true concerning monitoring of the surgical patient.

a. A pulse oximeter reading will reflect changes in PaO2 only below 80


mm Hg
b. Monitoring of end tidal CO2 will reflect changes in ventilation but not
cardiac output

163 of 167 11-Aug-15 10:57 PM


MedCosmos Surgery: MCQ : General Surgery http://medcosmossurgery.blogspot.com/search/label/MCQ : General Surgery

c. Intermittent, noninvasive systemic blood pressure monitoring using an


oscillometric blood pressure cuff has essentially replaced clinical
measurement by auscultation
d. Pulmonary arterial catheter monitoring is generally reserved for
critically ill patients with significant left ventricular dysfunction
Answer: a, c, d

Pulse oximetry continuously, noninvasively and inexpensively provides


arterial hemoglobin saturation and peripheral pulse determination. It
must be remembered, however, that a pulse oximeter measures oxygen
saturation and not arterial oxygen tension (PaO2). The PaO2 must drop
below 80 mm Hg before any significant change in oxygen saturation will
occur. End tidal CO2 monitoring reflects metabolism (the production of
CO2), circulation (blood flow to the lungs), and ventilation (respiratory
rate in an intact ventilatory circuit). It can be used as a surveillance
monitor for both the respiratory circuit and the cardiovascular system.
Any acute decrease in cardiac output will decrease output to the lung
and increase alveolar dead space, causing an acute drop in end tidal
CO2.
Hemodynamic stability can be monitored in a variety of methods, the
most basic of which is systemic arterial blood pressure measure.
Intermittent, noninvasive measure of systemic blood pressure with an
oscillometric blood pressure cuff has become the standard in the
operating room with an accuracy equal to that of clinical measurement
by auscultation. When tighter control is required in patients with
significant hypertension, serious heart disease, or in patients who may
suffer acute blood loss, invasive arterial monitoring is employed. In
patients with left ventricular dysfunction who are undergoing extended
surgical procedures with significant fluid shifts and potential blood loss,
central venous pressure monitoring is frequently used, with pulmonary
arterial catheter monitoring reserved for more critically ill patients and
for those with significant left ventricular dysfunction.

289 Correct statement(s) concerning complications occurring in the


post-anesthetic care unit include which of the following?

a. The use of nitrous oxide has been well documented to increase the
incidence of postoperative nausea
b. Perioperative myocardial ischemia is usually easily diagnosed in the
early postoperative period
c. Hypothermia results in a deleterious effect on drug metabolism
therefore delaying recovery from anesthesia
d. The serotonin antagonist, odansetron, holds promise as the superior
antiemetic agent in the perioperative period
Answer: c, d

Twenty-four percent of patients experience a post-anesthetic care unit


complication. Nausea, vomiting and airway support comprise 70% of
these complications. The need to maintain airway support is by far the
most common respiratory complication. Hypothermia has a deleterious
effect on altering drug metabolism and delaying recovery. Nausea and
vomiting are rarely unifactorial and cause considerable discomfort to

164 of 167 11-Aug-15 10:57 PM


MedCosmos Surgery: MCQ : General Surgery http://medcosmossurgery.blogspot.com/search/label/MCQ : General Surgery

the patient. There is little evidence to favor one anesthetic or


anesthetic technique over another. Nitrous oxide does not appear to
increase incidence of nausea in well documented studies. The new
serotonin antagonist, odansetron, has been shown in several studies to
be superior to other agents as a perioperative antiemetic agent.
Perioperative myocardial ischemia is an extremely important
complication but difficult to recognize. Diagnosis is complicated by the
fact that only 10—30% of patients suffering documented myocardial
infarction will have pain and that postoperative EKG changes are often
nonspecific. One must therefore look for secondary indications of
on-going ischemia such as hypotension, arrhythmias, elevated filling
pressures, or postoperative oliguria.

290 Patient-controlled analgesia ( PCA) is a commonly used technique


for postoperative analgesia. The following statement(s) is/are true for
the use of PCA.

a. Satisfactory pain relief is provided by the administration of higher


narcotic doses
b. The technique is not applicable in the semiconscious or
uncooperative patient
c. PCA is as safe as conventional intramuscular administration of pain
medication
d. Excessive administration of narcotic medication can be limited by a
lockout duration which controls administration of the narcotic
Answer: b, c, d

The technique of patient-controlled analgesia is based on investigations


that small intravenous bolus doses of narcotic on demand can provide
patients with improved pain relief at the same or less total narcotic
dose. The system requires some degree of sophistication and a conscious
patient who has been instructed in the technique. Numerous studies
have demonstrated that PCA is as safe as conventional IM medication.
The patient can be restricted from receiving excessive agents via setting
a lockout interval duration of several minutes during which time a dose
of narcotic cannot be successfully administered. In addition, limits to
the total hourly dose can be set.

291 Narcotics are commonly used in the administration of general


anesthesia. Which of the following statement(s) is/are true concerning
this class of agents.

a. Narcotics have both profound analgesic and amnestic properties


b. Narcotics can cause hypotension by direct myocardial depressive
effects
c. Naloxone should be used routinely for the reversal of narcotic
analgesia
d. Acutely injured hypovolemic patients are at significant risk for
decreased blood pressure with the use of narcotic analgesics
e. Propofol is a new intravenous short-acting narcotic used frequently in
the outpatient setting

165 of 167 11-Aug-15 10:57 PM


MedCosmos Surgery: MCQ : General Surgery http://medcosmossurgery.blogspot.com/search/label/MCQ : General Surgery

Answer: d

Narcotics and synthetic analogues belong in the class of drugs called


opioids. Narcotics produce profound analgesia and respiratory
depression. They have no amnesic properties, no myocardial depressive
effects, and no muscle relaxant properties. Narcotics may produce
significant hemodynamic effects indirectly through the release of
histamine and/or blunting of the patient’s sympathetic vascular tone
due to analgesic properties. Acutely injured patients may be
hypovolemic and in pain, with high sympathetic tone and peripheral
resistance. Therefore, such patients can experience a dramatic drop in
systemic blood pressure with minimal doses of opioids. All opioids can
be reversed with naloxone. Naloxone reversal, however, can be
dangerous because the agent acutely reverses not only the analgesic
effects of the opioid but also analgesics effects of native opioids.
Naloxone treatment has been associated with acute pulmonary edema
and myocardial ischemia and should not be used electively to reverse
the effects of narcotic. Propofol is a lipid-soluble substitute isopropyl
phenol non-narcotic agent that produces rapid induction of anesthesia
followed by awakening in four to eight minutes.

292 Anesthetic techniques used in the management of patients with


significant pulmonary disease include:

a. Intubation at a deep level of anesthesia


b. Choice of an anesthetic agent which produces bronchodilatation
c. The use of epidural analgesia for postoperative pain control
d. Perioperative use of intermittent positive pressure breathing
Answer: a, b, c

Patients with significant pulmonary diseases require special anesthetic


techniques. Obstructive pulmonary disease can either be chronic (COPD)
or acute (asthma). In either case, the reversible component of
obstruction should be reversed prior to elective surgery. In patients with
reactive airway disease, the endotracheal tube may induce severe
bronchospasm. Even in patients who are well treated preoperatively,
reactive bronchospasm may complicate anesthetic induction and
emergence from anesthesia. The principal method used to prevent or
diminish this “foreign body” induced bronchospasm is intubation of the
patient at a deep level of anesthesia when reflexes are blunted. The
classic way of managing a patient with severe asthma is to induce with
an agent that produces bronchodilatation and to ventilate the patient
with an inhalation agent until deeply anesthetized prior to laryngoscopy
and intubation. The patient should be extubated while spontaneously
ventilating, but with the inhalation agent still in effect, bringing the
patient to consciousness while ventilating by mask.
Because of the potential adverse effects of systemic narcotics on
respiratory drive, the use of epidural narcotics and local anesthetics for
postoperative pain control has become very popular. These techniques
allow the patient to be extubated earlier, and patients with
intrathoracic and upper abdominal surgery, help restore pulmonary

166 of 167 11-Aug-15 10:57 PM


MedCosmos Surgery: MCQ : General Surgery http://medcosmossurgery.blogspot.com/search/label/MCQ : General Surgery

function toward preoperative values. Preoperative use of intermittent


positive pressure breathing has not been demonstrated to decrease the
incidence of postoperative pulmonary complications.

293 Which of the factors listed below will adversely affect the risk of
perioperative cardiac complications and reinfarction in the patient
described above?

a. Greater than five premature ventricular beats per minute on EKG


rhythm strip
b. The anesthetic technique used
c. Withdrawal of medical therapy with beta blockers and topical
nitrates
d. Length of surgical procedure less than three hours
e. Known three vessel coronary artery disease
Answer: a, c, e

The incidence of reinfarction is increased in patients undergoing


intrathoracic or intra-abdominal procedures lasting longer than three
hours. The site of surgery or anesthetic technique have not been shown
to change the incidence of reinfarction if the procedure is less than
three hours in duration. Patients with known three-vessel or left main
coronary artery disease are at increased risk, while those who have
undergone prior coronary artery bypass grafting are of substantially
decreased risk of reinfarction. Prophylactic therapy with beta blockers,
calcium channel agents, and nitrates has not been proven beneficial;
however, withdrawal of these agents has been associated with
perioperative ischemia, myocardial infarction, and death. CHF is the
single most important factor predicting postoperative cardiac morbidity.
Rhythm disturbances, particularly frequent premature ventricular beats,
more than five beats/minute, are also independently associated with an
increased risk of perioperative cardiac complications.

Posted by MedCosmos at 6:08 PM No comments:


Labels: MCQ : General Surgery

Home Older Posts

Subscribe to: Posts (Atom)

167 of 167 11-Aug-15 10:57 PM

You might also like